Вы находитесь на странице: 1из 514

Financial Accounting

Paper F3 (International)
Integrated Course Notes
ACF3CN09(J) (INT)
Improving study material and removing errors
There is a constant need to update and enhance our study materials in line with both regulatory changes and new insights
into the exams. BPP appoints, from one of our experienced tutor team, a subject expert to update and improve these
course notes regularly. These updates are technically checked by another experienced tutor and frequently proof read.
We always aim to leave no numerical errors and narrative typos. However, given the volume of detailed information being
changed in a short space of time, it is regrettable that despite our control procedures an error may slip through. We
apologise sincerely for any inconvenience that this might cause.
If you find a specific error or typo please let us know at ACCAcoursesfeedback@bpp.com so we can correct it
immediately. In addition we would welcome any suggestions you may have to further improve these study materials.

Blank
F3 Financial Accounting (INT)
Contents page
Page
Introduction to the paper and the course................................................................................................................. 5
Skills bank ............................................................................................................................................................. 13

1 Introduction to accounting ............................................................................................................................ 31


2 Home study chapter – The regulatory framework ........................................................................................ 43
3 Accounting conventions................................................................................................................................ 53
4 Sources, records and books of prime entry.................................................................................................. 73
5 Ledger accounts and double entry ............................................................................................................... 91
6 From trial balance to financial statements .................................................................................................. 107
7 Sales tax..................................................................................................................................................... 135
8 Inventory..................................................................................................................................................... 149
9 Tangible non-current assets....................................................................................................................... 169
10 Intangible non-current assets ..................................................................................................................... 201
11 Accruals and prepayments......................................................................................................................... 213
12 Irrecoverable debts and allowances ........................................................................................................... 237
13 Provisions and contingencies..................................................................................................................... 261
14 Control accounts ........................................................................................................................................ 273
15 Bank reconciliations.................................................................................................................................... 301
16 Correction of errors..................................................................................................................................... 315
17 Home study chapter – Preparation of financial statements for sole traders ............................................... 331
18 Incomplete records..................................................................................................................................... 343
19 Partnerships ............................................................................................................................................... 359
20 Introduction to company accounting........................................................................................................... 385
21 Preparation of financial statements for companies..................................................................................... 405
22 Events after the reporting period ................................................................................................................ 427
23 Statements of cash flows............................................................................................................................ 437
24 Home study chapter – Information technology ........................................................................................... 459
25 Answers to Lecture Examples ................................................................................................................... 471

3
INTRODUCTION

F3 Financial Accounting (INT)


Study Programme
Page
Introduction to the paper and the course................................................................................................................. 5
Skills bank ............................................................................................................................................................. 13

1 Introduction to accounting ............................................................................................................................ 31


2 Home study chapter – The regulatory framework ........................................................................................ 43
3 Accounting conventions................................................................................................................................ 53
4 Sources, records and books of prime entry.................................................................................................. 73
5 Ledger accounts and double entry ............................................................................................................... 91
End of Day 1 – refer to Course Companion for Home Study
6 From trial balance to financial statements .................................................................................................. 107
8 Inventory..................................................................................................................................................... 149
9 Tangible non-current assets....................................................................................................................... 169
10 Intangible non-current assets ..................................................................................................................... 201
End of Day 2 – refer to Course Companion for Home Study
11 Accruals and prepayments......................................................................................................................... 213
12 Irrecoverable debts and allowances ........................................................................................................... 237
13 Provisions and contingencies..................................................................................................................... 261
15 Bank reconciliations.................................................................................................................................... 301
End of Day 3 – refer to Course Companion for Home Study
7 Sales tax..................................................................................................................................................... 135
14 Control accounts ........................................................................................................................................ 273
16 Correction of errors..................................................................................................................................... 315
17 Home study chapter – Preparation of financial statements for sole traders ............................................... 331
18 Incomplete records..................................................................................................................................... 343
End of Day 4 – refer to Course Companion for Home Study
19 Partnerships ............................................................................................................................................... 359
20 Introduction to company accounting........................................................................................................... 385
22 Events after the reporting period ................................................................................................................ 427
End of Day 5 – refer to Course Companion for Home Study
21 Preparation of financial statements for companies..................................................................................... 405
23 Statements of cash flows............................................................................................................................ 437
24 Home study chapter – Information technology ........................................................................................... 459

End of Day 6 – refer to Course Companion for Home Study


25 Answers to Lecture Examples ................................................................................................................... 471

Prepare for and book your CBE!


You should plan to sit your CBE within the next couple of weeks whilst the knowledge from this course is still fresh in your
mind. In preparation use the Learning Media Practice & Revision Kit and i-Pass to test yourself on as many questions as
you can, revising from the Course Notes and Passcards any areas of the syllabus that cause you problems.
One of the criteria for Pass Assurance is that you book your CBE with BPP, so please contact your local BPP centre as
early as you can to book your CBE and good luck!

4
INTRODUCTION

Introduction to Paper F3 Financial Accounting (INT)


Overall aim of the syllabus
To develop knowledge and understanding of the underlying principles and concepts relating to financial
accounting and technical proficiency in the use of double-entry accounting techniques including the preparation
of basic financial statements.

The syllabus
The broad syllabus headings are:

A The context and purpose of financial reporting


B The qualitative characteristics of financial information and the fundamental bases of accounting
C The use of double entry and accounting systems
D Recording transactions and events
E Preparing a trial balance
F Preparing basic financial statements

Main capabilities
On successful completion of this paper, candidates should be able to:
• Explain the context and purpose of financial reporting
• Define the qualitative characteristics of financial information and the fundamental bases of accounting
• Demonstrate the use of double entry and accounting systems
• Record transactions and events
• Prepare a trial balance (including identifying and correcting errors)
• Prepare basic financial statements for incorporated and unincorporated entities

Links with other papers

Corporate Reporting
(P2)

Financial Reporting
(F7)

Accountant in Financial Accounting


Business (F1) (F3)

This diagram shows where direct (solid line arrows) and indirect (dashed line arrows) links exist between this
paper and other papers that may precede or follow it.
Paper F7 Financial Reporting, assumes knowledge acquired in paper F3 Financial Accounting, and develops and
applies this further and in greater depth. Paper P2 Corporate Reporting, assumes knowledge acquired at the
Fundamentals level including core technical capabilities to prepare and analyse financial reports for single and
combined entities.

5
INTRODUCTION

Assessment methods and format of the exam


Examiner: Nicola Ventress
Students have the option to take either a paper-based or a computer-based exam.
Both examinations are a two hour paper and all questions are compulsory. Questions will assess all parts of the
syllabus and will contain both computational and non-computational elements.

Format of the Exam Marks


40 two mark questions 80
10 one mark questions 10
90

6
INTRODUCTION

Study Programme Aims


Achieving ACCA's Study Guide Outcomes

A The context and purpose of financial reporting


A1 The reasons for and objectives of financial reporting Chapter 1
A2 Users’ and stakeholders’ needs Chapter 1
A3 The main elements of financial reports Chapter 1
A4 The regulatory framework Chapter 2

B The qualitative characteristics of financial information and the


fundamental bases of accounting
B1 The qualitative characteristics of financial reporting Chapter 3
B2 Alternative bases used in the preparation of financial information Chapter 3

C The use of double entry and accounting systems


C1 Double entry bookkeeping principles including the maintenance of accounting records Chapters 4 & 5
and sources of information
C2 Ledger accounts, books of prime entry and journals Chapters 4 & 5
C3 Accounting systems and the impact of information technology on financial reporting Chapter 24

D Recording transactions and events


D1 Sales and purchases Chapters 4, 5, 7
& 14
D2 Cash Chapters 4 & 5
D3 Inventory Chapter 8
D4 Tangible non-current assets Chapter 9
D5 Depreciation Chapter 9
D6 Intangible non-current assets and amortisation Chapter 10
D7 Accruals and prepayments Chapter 11
D8 Receivables and payables Chapter 12
D9 Provisions and contingencies Chapter 13
D10 Capital structure and finance costs Chapters 20 &
21

7
INTRODUCTION

E Preparing a trial balance


E1 Trial balance Chapter 6
E2 Correction of errors Chapter 16
E3 Control accounts and reconciliations Chapter 14
E4 Bank reconciliations Chapter 15
E5 Suspense accounts Chapter 16

F Preparing basic financial statements


F1 Statements of financial position Chapter 17
F2 Income statements Chapter 17
F3 Events after the reporting period Chapter 22
F4 Accounting for partnerships Chapter 19
F5 Statements of cash flows (excluding partnerships) Chapter 23
F6 Incomplete records Chapter 18

8
INTRODUCTION

Analysis of the pilot paper


Please note that the ACCA will not publish past exam papers for the Knowledge modules. The analysis of the
Pilot Paper should therefore be used as a guide to both the areas that will be examined and the mix between
narrative and computational questions.

Computational
Narrative
The context and purpose of financial reporting
The reasons for and objectives of financial reporting
Users’ and stakeholders’ needs
The main elements of financial reports 1
The regulatory framework 1

The qualitative characteristics of financial information and the fundamental bases of


accounting
The qualitative characteristics of financial reporting 1
Alternative bases used in the preparation of financial information 4

The use of double entry and accounting systems


Double entry bookkeeping principles including the maintenance of accounting records and
sources of information
Ledger accounts, books of prime entry and journals 1

Accounting systems and the impact of information technology on financial reporting 1

Recording transactions and events


Sales and purchases 1
Cash 1
Inventory 1 1
Tangible non-current assets and depreciation 1 4

Intangible non-current assets and amortisation 1


Accruals and prepayments 2

Receivables and payables 2

Provisions and contingencies 1


Capital structure and finance costs 1 2

9
INTRODUCTION

Computational
Narrative
Preparing a trial balance
Trial balance 1
Correction of errors 1 2

Control accounts and reconciliations 4

Bank reconciliations 1 1

Suspense accounts 1

Preparing basic financial statements


Statements of financial position 1 1
Income statements
Events after the reporting period 2
Accounting for partnerships 1 2

Statements of cash flows (excluding partnerships) 3


Incomplete records 2

10
INTRODUCTION

Key to icons

Question practice from the Study Text


This is a question we recommend you attempt for home study.

Section reference in the Study Text


Further reading is needed on this area to consolidate your knowledge.

11
INTRODUCTION

12
1

Skills bank

This section explains and demonstrates the key skills


required to enable you to maximise your chance of
exam success. Knowledge of the syllabus is insufficient
on its own. Through question practice you will develop a
set of skills that will enable you to pass this paper.

13
14
1
SKILLS BANK

Key skills required to pass


Our analysis of the examiner’s comments on past exams, together with our experience of preparing students for
this type of exam, suggests that to pass Paper F3 you will need to develop a number of key skills.

1 Learning and 2 Time


understanding management and
the syllabus approaching
content questions logically

3 Answering
questions
efficiently

Each of these key skills is analysed on the following pages. Examples from past exam questions are
included to illustrate the importance of these skills and how these skills should be applied.

15
SKILLS BANK

Skill 1 – Learning and understanding the syllabus content

1 What do I need to know to attempt the paper?


F3 is a broad syllabus that can be tested by objective testing including multiple choice questions and other
means such as data entry type questions. This means that you need a broad and yet quite detailed knowledge
of the syllabus as well as an understanding of a variety of calculations to apply the theory.
The type of knowledge that you have to acquire includes the following:
Practical application – e.g. calculation of numbers to be included in the financial statements; identifying errors
and correcting them; producing figures to go into a statement of cash flows.
Theoretical knowledge – e.g. how accounts fit together and their key components; the purpose and roles of the
regulatory bodies; the underlying concepts and assumptions that govern accounts preparation.
In this section we will look at approaches that you can take to help you learn the key elements of the knowledge
in the syllabus.

2 Practical application
Practical application requires you to do two main things:
1. Understand the rationale behind a topic and be able to explain it; and
2. Apply your understanding to generate figures that may be included in a set of accounts.
In this way you should be in a good position to answer most questions. They will either ask you to calculate a
number from some information provided, or to use the numerical information provided to demonstrate your
knowledge of the topic in some way. You should ensure that you read the requirement carefully for these
questions; further tips on question approach will be covered under skill 2 [page 20].
For example, in the pilot paper Q32 asks:

Alpha received a statement of account from a supplier, Beta,


showing a balance to be paid of $8,950. Alpha’s payables
ledger account for Beta shows a balance due to Beta of $4,140.

Investigation reveals the following:


(1) Cash paid to Beta $4,080 has not been allowed for by Beta
(2) Alpha’s ledger account has not been adjusted for $40 of
cash discount disallowed by Beta.

What discrepancy remains between Alpha’s and Beta’s records


after allowing for these items?

A $690
Application

B $770
C $9,850
D $9,930 (2 marks)

This question requires you to have a good understanding of how to account for credit purchase transactions and
the way control accounts work in order to be able to answer it.

16
SKILLS BANK

Another question type is Q46 from the pilot paper. It requires you to apply your understanding of a specific area
of the syllabus.

Gareth, a sales tax registered trader purchased a computer for


use in his business. The invoice for the computer showed the
following costs relating to the purchase:
$
Computer 890
Additional memory 95
Delivery 10
Installation 20
Maintenance (1 year) 25
1,040
Sales tax (17.5%) 182
Total 1,222
How much should Gareth capitalise as a non-current asset in
relation to the purchase?

A $1,222

B $1,040
Principles

C $890

D $1,015 (2 marks)

Note that to find the correct figure you need to understand both the principle behind what can be included in the
cost of a non-current asset and what effect the sales tax may have on the cost of the asset to be included in the
accounts.

Therefore you need to ensure that you understand the context of the principles of what you are learning and are
able to apply them to numerical examples. To do this you should consider:

Detail – for practical application of the topics covered in the syllabus, you must know the rules for each area and
understand where they have been derived from. By reviewing the overview at the beginning and the summary at
the end of the relevant chapter in your course notes before attempting questions, you should be able to pick up
the key points.

Application – this is where question practice is key. The more practice you have in working through the
questions, the more confident you will become on using and applying the theory.

3 Theory
You will also have to answer narrative questions about theory.

You can expect questions about:

1. Fact – what are the rules and requirements of the accounting standards? What are the similarities and
differences between sole traders, partnerships and companies?

2. Application – how are the accounting concepts applied to different areas of the syllabus?

Here is an example from the ACCA F3 examiner on an area that students struggled with in the December 2007
paper based exam:

17
SKILLS BANK

Which of the following provides advice to the International


Accounting Standards Board (IASB) as well as informing the IASB of
the implications of proposed standards for users and preparers of
financial statements?
A The Standards Advisory Council
B The International Financial Reporting Interpretations Committee
Fact

(1 mark)

This question is testing you on facts regarding the roles of the different regulatory bodies. To be able to answer
this question you need to know what each body does and how they fit together.

Here is a further example from the Pilot Paper

Q3

In times of rising prices, what effect does the use of the


historical cost concept have on a company’s asset values and profit?
A Asset values and profit both understated
B Asset values and profit both overstated
Application

C Asset values understated and profit overstated


D Asset values overstated and profit understated
(2 marks)

This type of question is about applying the theory to a theoretical situation. This can seem tricky if there are no
numbers involved – the key here is to think of some simple numbers for an asset and its depreciation policy (e.g.
10% straight line) both at cost (e.g. $100) and revalued amount (e.g. $300) and see what effect they have on
asset values and profits:

Cost ($) Revalued


amount ($)
Asset (cost – 1 year’s depreciation) (100 – 10) 90 270 (300- 30)

Profit before depreciation 50 50


Depreciation (10) (30)
Profit after depreciation 40 20

It is therefore important to make sure that you actively learn the content of the syllabus in such a way that you
can answer all of the types of question that you may face in the exam.

18
SKILLS BANK

There are various ways to build up this level of knowledge. Here are some suggestions:

Read Passcards Get a colleague or


regularly friend to set you
quizzes
Practice as many
MCQs as possible
Write your own Produce a list of your
glossary of definitions common mistakes and
review it before doing
question practice!

Skills practice
Learn the content of the syllabus actively by:
1. Reviewing the key overviews for each chapter
2. Practising as many questions as possible, moving from using your notes to completing them
without any help
3. Using the study text to help only on areas you’re struggling with and to fill in gaps in your
background knowledge

19
SKILLS BANK

Skill 2 – Time management and approaching questions


logically
The F3 exam contains objective tests; it is important that you use your time wisely to gain maximum marks. Your
approach will differ depending on whether you are doing a computer based exam (CBE) or a paper based exam.
In the paper based exam you will have mainly multiple choice questions with some yes/no questions. The
computer based exam (CBE) may contain a wider range of question styles including selecting a number of
possible correct answers from a list and entering numbers into a blank box.
You need to ensure that you have a logical approach to these questions to maximise your chances of getting the
correct answers.
Below we look at how to approach each style of paper, and within that how to deal with the different types of
questions you may come across.

Time management

1 Paper based exam and computer based exam


: What you SHOULD NOT do
Panic! You have two hours to answer 50 questions. This equates to approximately 2.6 minutes per question or
1.3 minutes for the small 1 mark questions. For many questions you will get the answer straight away and so
you are likely to have a bit more time to think about some of the others. Therefore don’t worry about your timing
on each individual question, just keep track over a few (e.g. 5).

; What you SHOULD do


It is important to start the exam positively.

Firstly:
Work through questions systematically
Start at question 1 and begin answering from there working through questions in order.
If you find a question that you don’t know the answer to and want to come back to it later then put
an answer in for the moment, make a note of it and go onto the next question. If you take the
paper based exam, this means you won’t fill out your answer sheet incorrectly by having left a
gap on the sheet.
Try not to jump around questions otherwise you may leave some unanswered by the end.

Then:
Check your paper before the end of the exam
Having answered all of the questions you should look through your answers to make sure:

1. You are happy with the options selected; and

2. You have answered all questions

If you have taken this logical and systematic approach you should have given yourself the best chance of doing
well in the exam.

20
SKILLS BANK

2 Computer based exams (CBE)


If you are doing the computer based exam you need be systematic in your approach as with the paper based
exam. There are however some other guidelines to ensure that you make the most of the CBE format of the
exam.

STEP Before the exam


1 Make sure that you are registered with the ACCA and that your exam centre has your exam
booking. This is important because you can not sit the exam if you are not a registered student
and you need your student card as identification on the day of your exam.

STEP At the beginning of the exam


2 You want to make the start of the exam as stress free as possible so make sure that you have
the following available:
• Photo identification and your student number – your ACCA student card is ideal for this
• Paper supplied by your exam centre, pens and a calculator
• Details of which exam you are planning to sit

STEP Starting the exam


3 To start the exam you will need to do the following:
• Login – enter your ACCA student number and your date of birth – make sure that you
have both to hand
• Select the exam – the system will ask you which exam you want to do (ensure you pick
the International exam and not the UK stream) and then you <Confirm> that selection
• Instructions – the next three screens are instructions – read these carefully so that you
know what you have to do to complete the exam
• Launch the exam – please don’t click this screen until you have been advised to by your
invigilator AND you are ready because this starts the exam and starts the timer.

21
SKILLS BANK

STEP Answering the questions


4 The exam will start at question 1. You can progress through the questions by clicking <Next>
but you can also go back by clicking <Previous>. When you answer the questions you must
follow the following procedure :
• Enter your answer
• Click on <Submit> - you must do this otherwise as soon as you click <Next> to move to
the next question your entry will be lost and you will have to re-enter
• If you click <Next> and have not submitted your answer to a question then you will get a
reminder – clicking <OK> on the reminder does not submit your answer – you must go
back and re-enter your answer and then <Submit>
• If you are unsure of an answer then just put an answer in for now, make a note of it, and
revisit it later

You can see that it is very important to submit your answers as you go and to keep track of what you have done.
It is probably a good idea therefore to use your pad of paper to do two things:
1. Make any notes you want to help you answer the questions; and
2. Keep a record of the status of each question

For example:

Question Status
1 9
2 ?
3 9
4 X
Where

9 means that the question has been answered and I am reasonably confident of the answer.
? means that the question has been answered but I want to check the answer
X means that the question has not been answered and I need to go back to it
It would be most efficient to set this up at the beginning of the exam. In this way you will have a tally of which
questions really need to be checked over at the end of the exam and it reduces the chances that you will leave a
question unanswered.

22
SKILLS BANK

STEP At the end of the exam time


5 At the end of the exam you should check your answers and ensure that you have submitted an
answer for every question as well as double checking any answers you were not sure of.
You have two ways of navigating the questions:
1. Clicking <Previous> to work back through the questions one by one; or
2. Using the drop down menu which shows all the questions 1-50 indicating whether an
answer has been submitted or not; clicking on the question number will take you directly to
that question
Remember that if you choose to change a previously submitted answer you must
<Submit> the new one otherwise your original answer will be retained instead!

STEP Closing your exam session


6 Once you are satisfied that you are happy with your answers, if you have time left in the exam
then you have a couple of options to finish your exam session (subject to any advice you get
from the invigilators in your exam centre).
1. Let the time on the on screen clock run down to zero and the exam session will end
automatically; or
2. Click <Exit> - you will be asked to <Confirm> this so you can not accidentally end your
session early
In either case the next thing you will see will be the Results Screen that shows your mark and
whether you have passed or not.
It is important that you don’t <Exit> at this stage since you have no proof of your result!
Your invigilator will ask you to <Print> two copies and will instruct you to <Exit> once these have
been printed off.
You have now finished your exam.

3 Some final tips on timings


• Make use of the paper to make notes or to work out the answers to questions;
• If you find a particularly difficult question or a long calculation, move on and come back to it later in the
remaining time – it is important that you do not run out of time leaving easier questions later in the
paper unanswered
• Keep an eye on the clock so that you can pace yourself
• Be well prepared for the exam day so that you can concentrate on doing the exam rather than the
administration around it
• Get to your exam venue in plenty of time so that you are relaxed when you get into the exam room.

23
SKILLS BANK

Approaching Questions Logically – Multiple Choice Questions

1 What to do if you know the answer to the question


If you know the answer to a narrative question you should:
1. Locate the correct answer
2. Check the other answers
3. Read the question again to ensure you’re answering the correct question
4. Confirm that you have the correct answer
This systematic check will ensure that you do not throw away marks when you really do know the answer.

2 What to do if more than one answer appears plausible


Sometimes more than one option can seem to answer the question. In this case you have to firstly ensure
you’ve read the requirement carefully, as questions may be phrased in ways that are not what you’re expecting.
If you still identify more than one likely option, select the “most correct” answer. The approach adopted above
is useful here too but this time you have to think through the alternatives a bit more.
For example, Q45 on the Pilot Paper states:

Which of the following statements are correct?


(1) A cash flow statement prepared using the direct method produces a
different figure for operating cash flow from that produced if the
indirect method is used.
(2) Rights issues of shares do not feature in cash flow statements.
(3) A surplus on revaluation of a non-current asset will not appear as
an item in the cash flow statement.
(4) A profit on sale of the sale of a non-current asset will appear as
an item under Cash flows from Investing Activities in a cash flow
statement.
A 1 and 4
B 2 and 3
C 3 only
D 2 and 4

This is testing your knowledge of cash flow statements. At first sight, it may be tricky to identify the correct
answer as there are so many variations in the options.
Here are some steps to follow:

STEP Never
1 Firstly, identify any answers that are immediately wrong. In this question, the key thing to think
about is what cash effect there may be to any transactions. Remembering that revaluations are
purely an accounting adjustment means that statement (3) must be correct. So we can discard
options A and D from the answers.

24
SKILLS BANK

STEP Assess the remaining answers


2 We now need to consider statement (2) as this will differentiate answers B and C. Remembering
that it is only a bonus issue of shares that has no cash effect (rights issues give shareholders the
right to buy shares at a certain price) means that statement (2) is incorrect (we do get some cash
in), so in this case we can now discard answer B and have arrived at answer C.

STEP Read the question again…


3 Finally, we should re-read the requirement before submitting the question to ensure we are
answering the correct question. The question could easily have asked: “Which of the following
statements is incorrect” which would have led us to a different answer!

This systematic approach helps you to break a question down and work through to find the correct answer
logically.

3 How to approach numerical questions


In the exam, you will be asked to calculate numbers based on some information provided. If it is a multiple
choice question rather than a data entry question, the temptation may be to look at the options first, and then “fit”
your calculations to the one you think is most likely. This could lead you to answering the question incorrectly,
especially if you have not read the requirement carefully.
For example, the ACCA F3 examiner identified the following question which students struggled with in the June
2008 paper based exam:

Johnsons use the imprest method for accounting for petty cash. The
petty cash was counted and there was $57.22 in hand. The following
petty cash slips were found for the following:
$
Stamps 16.35
Sale of goods to staff 12.00
Coffee and tea purchase 18.23
Birthday cards for stuff 20.20

What is Johnsons’ imprest amount?


A $124
B $100
C $112
D $80

Working through this question, we find that, since the $12.00 was from sale of goods to the staff (i.e. a receipt in
the petty cash tin), the imprest amount is $100. It is important to understand what the question is asking and
how to approach answering these types of questions.

25
SKILLS BANK

STEP Read the requirement carefully


1 With numerical questions it is really important to first understand what the question is asking,
since you can easily either do too much work and waste time working out calculations that aren’t
required, or answer the question you want to answer, and not the question that is actually being
asked!
In the question above, many students picked option A as they assumed all slips were for
amounts paid and none for receipts.

STEP Ignore the options


2 Once you’re certain what the question is asking, you should ignore the options given, as you
may arrive at an answer that matches an option but is not what is required by the question.
Again, this is where many students tripped up above, as they saw option A was available and
immediately picked that rather than remembering what the requirement was for that question.

STEP Match your answer


3 Once you’ve calculated your answer, match it to the options. If you’ve worked carefully and
answered the question, the matching option will be available. If no option matches your
calculation, re-read the requirement to ensure you’ve understood what you have to do. If you
still can’t find the answer, you may want to guess the answer, make a note of the question
number and return to it at the end when you can judge how much of the time remaining you can
spend on it.

4 What to do if you still don’t know the answer…


If you have been through the above steps and can’t identify a preferred answer then you have to guess!

: What you SHOULD NOT do


Two main things to avoid:
1. Waste excessive time – time spent dithering over a single question could leave you with insufficient time
for the rest of the paper.
2. Not answering – this is a common yet serious error – even if you make a wild guess you start with a 25%
chance of success. Your chance of getting the 2 marks if you don’t offer an answer is zero!

; What you SHOULD do


Having used the three step approaches above to narrow down your possible answers, go with the one that
feels right. And move on.

If you have a flash of inspiration later in the exam go back and revisit it – but only if you are sure.

Data entry questions


The above systematic approach to multiple choice questions can be applied to any questions that you come
across in the exam. You may be asked to calculate a numerical figure which you then have to enter into a box in
the exam. For these questions you can use steps 1 and 2 above in How to Approach Numerical Questions.

26
SKILLS BANK

Skills practice
1. Practice keeping track of the questions you have answered when doing questions from the
Practice and Revision Kit
2. Always check your answers through (if you would have time in the exam) before looking at
the solutions in the back of the book
3. Practice as many multiple choice questions as possible.
4. If you don’t know the answer to a question – don’t just go to the answer at the back or just
guess – use the three step approach described above.

27
SKILLS BANK

Skill 3 – Answering questions efficiently


Once you have grasped an area fully, the third key skill in being successful at F3 is to answer questions
efficiently. If you are confident at how to arrive at an answer, you can often avoid doing extra calculations that do
not impact on the final answer by knowing when to stop.

1 Knowing the basics


Before you attempt any shortcuts, it is important to make sure you’re happy with the topic itself. You should work
through questions as you have been taught in lecture examples, and make sure you can arrive at the right
answer each time.

2 Identifying when to “Stop”


Whilst you will have enough time to answer every question fully in the exam, it makes sense to develop your
question technique as you revise to try to recognise at what point you’ve arrived at the answer and can stop.
This means you avoid wasting time and can use those extra seconds to think about other questions.
As an example, the ACCA F3 examiner identified the following question which students struggled with in the
June 2008 paper based exam:

Charles entered into the following transactions:

1. He sold goods on credit to Cody with a list price of $3,200. He


allows a 10% trade discount and a further 2% discount for payment
within seven days. Cody paid within two days.

2. He made a credit sale to Mary allowing a 5% trade discount on the


list price of $640.

3. He purchased goods for $600 and paid $590, receiving a discount for
immediate cash payment.

How much discount should be recorded in the Discount Allowed account


as a result of the above transactions?

A $57.60

B $10.00

C $352.00

D $409.60

28
SKILLS BANK

It is tempting, if you know the topic, to calculate absolutely everything here. But if you think about the
requirement, all you need to work out is what the discount allowed balance is.
Remember, this is only the discount we allow our customers for prompt payment. It is not any discount we
receive (so transaction 3 is irrelevant to this question!). It is also not any trade discount we allow our customers
who are bulk buyers or regular customers (they will always take this discount as there are no conditions
attached; we therefore record the sale initially net of this discount). Therefore transaction 2, which only deals
with a trade discount, is also irrelevant.
Now, we have eliminated a lot of the work, and are left with transaction 1:
The initial price was $3,200, on which there was a trade discount of 10%. So:

List price $3,200


Trade discount (10%) $(320)
Amount recorded as a sale initially $2,880
We know that there’s a further settlement discount of 2% if payment is received within seven days, which it is.
Therefore we can calculate the discount we allowed this customer on early payment:

Amount recorded as a sale initially $2,880.00

Further settlement discount (2%) $(57.60)


No more work is needed on this question – you now have the answer!
Spending time now working out the other trade discount or the discount received is unnecessary.

3 Building up your technique


There are several places on the syllabus where you may be able to apply this technique. As you practise
questions, compare your workings to the answers and highlight at what point on your workings you arrived at the
answer they wanted.
In this way you can start to pick up where you’re doing too much work to answer the question. Your goal should
be to finish your calculations as soon as you’ve got the correct answer!
You may like to make a list of key common areas where you can answer questions efficiently, and look over
them as part of your revision.

Skills practice
1. Practice questions building up to the exam with efficiency in mind. The more you do, the
more efficient you’ll become.
2. Identify from your question practice which areas this is likely to apply to and make a list of
key areas and the relevant shortcuts.

29
SKILLS BANK

30
Introduction
to accounting 1
Syllabus Guide Detailed Outcomes
Having studied this chapter you will be able to:
• Define and understand the principles of financial reporting.
• Identify and define the different business entities of: sole trader, partnership and limited liability company and
recognise the legal differences between them.
• Identify the advantages and disadvantages of operating as each of the three types of business entity.
• Identify the users of financial statements and state and differentiate between their information needs.
• Understand and identify the purpose of each of the main financial statements.
• Define and identify assets, liabilities, equity, revenue and expenses.

Exam Context
This chapter introduces the subject of accounting. Questions on this area will most likely focus on the different
characteristics of the three types of business entity: sole trader, partnership and limited liability company.

Qualification Context
Sole trader and partnership accounts are only examined in Financial Accounting. The Fundamentals and Professional
level papers of Financial Reporting (F7) and Corporate Reporting (P2) are set in the context of a limited liability
company. These papers will test your understanding of the content of financial statements and the detailed accounting
rules which companies must apply.

31
1: INTRODUCTION TO ACCOUNTING

Overview

Income statement Statement of financial


position

Financial statements Users of financial


information

Introduction
to accounting

Types of business entities

Sole trader Partnership Limited liability


company

Concept of separate entity

32
1: INTRODUCTION TO ACCOUNTING

1 Accounting
Definition
1.1 Accounting is a way of recording, analysing and summarising transactions of a business.

2 Proforma financial statements


Income statement
2.1 Income statement for the year ended 31 December 20X7:
$ $
Sales 200,000
Less: Cost of sales
Opening inventories 40,000
Purchases 110,000
Carriage inwards 20,000
170,000
Closing inventories (50,000)
(120,000)
Gross profit 80,000
Sundry income 5,000
Discounts receivable 3,000
88,000
Less: Expenses
Rent 11,000
Carriage outwards 4,000
Telephone 1,000
Electricity 2,000
Wages and salaries 9,000
Depreciation 7,000
Bad and doubtful debts 3,000
Motor expenses 5,000
Discounts allowable 1,000
(43,000)
Profit for the year 45,000

33
1: INTRODUCTION TO ACCOUNTING

Statement of financial position


2.2 Statement of financial position as at 31 December 20X7:
$ $
ASSETS
Non-current assets
Land and buildings 100,000
Office equipment 50,000
Motor vehicles 30,000
Furniture and fixtures 20,000
200,000
Current assets
Inventories 50,000
Trade receivables 30,000
Less: allowance for receivables (2,000)
28,000
Prepayments 5,000
Cash in hand and at bank 7,000
90,000
Total assets 290,000

CAPITAL AND LIABILITIES


Capital
Capital 170,000
Profit 45,000
Less: drawings (25,000)
190,000
Non-current liabilities
Bank loans 40,000

Current liabilities
Bank overdraft 16,000
Trade payables 40,000
Accruals 4,000
60,000
Total capital and liabilities 290,000

34
1: INTRODUCTION TO ACCOUNTING

3 Users of financial information


Lecture example 1 Idea generation

Required
What information would these users of financial information be interested in?

Solution
(a) Investors

(b) Employees

(c) Lenders

(d) Suppliers

(e) Customers

(f) Governments and their agencies

(g) Public

35
1: INTRODUCTION TO ACCOUNTING

4 Accounting records
4.1 In order to be able to produce an income statement and a statement of financial position a
business needs to keep a record of all its transactions.

4.2 This process is called bookkeeping.

4.3 Accounting records should be complete, accurate and valid if the information produced is
to be useful for the users of financial information.

4.4 The mechanics of bookkeeping and the accounting records a business should keep will be
covered in Chapters 4, 5 and 6.

5 Types of business entities


Quick Quiz Q2
5.1 Businesses fall into three main types:
(a) Sole trader

(b) Partnership

(c) Limited liability company

Sections 2.3, 2.4

The sole trader is the simplest of these forms.

6 The concept of business entity (separate entity)


6.1 A business is considered to be a separate entity from its owner and so the personal
transactions of the owner should never be mixed with the business transactions.

6.2 When considering a limited liability company this distinction is laid down in law – the
company has a separate legal identity.

6.3 In preparing accounts, any type of business is treated as being a separate entity from its
owner(s).

36
1: INTRODUCTION TO ACCOUNTING

7 Chapter summary
Section Topic Summary
1 Accounting Accounting is a way of recording, analysing and
summarising a business’ transactions.
2 Proforma financial Companies must follow a prescribed format when
statements producing their financial statements, there is however
no set format for a sole trader’s income statement and
statement of financial position.
3 Users of financial Financial statements are used by a wide variety of
information users, each with different information needs. Satisfying
the investors’ needs will mean that the majority of
other users’ needs are also met.
4 Accounting records All businesses must keep sufficient accounting records
in order to be able to produce accurate information
about the entity’s activities.
5 Types of business There are three main types of businesses. For sole
entities traders and partnerships the owners have unlimited
liability and bear all the risks and reap all the rewards
of being in business. For a limited liability company
the shareholders' liability is limited to the extent of
their investment.
6 The concept of The business entity concept states that a business is
business entity a separate entity from its owners.

37
1: INTRODUCTION TO ACCOUNTING

Chapter Summary

Income statement Statement of financial


position

• Shows the income and expenses for • Shows the assets and liabilities of
a business over a period of time, the business at a point in time
usually a year

Financial statements Users of financial


information

• Investors
• Employees
Introduction • Lenders
to accounting • Suppliers
• Customers
• Governments and
their agencies
• Public

Types of business entities

Sole trader Partnership Limited liability


company
'An individual sets up business on 'More than one individual enter into 'A separate legal entity from the owners'
their own' business together' • The company bears the risks and
• All the risks and rewards are • Risks and rewards are shared rewards
borne by the sole trader between the partners • The owners have limited liability

Concept of separate entity

• A business is a separate entity from its owner


• Personal transactions must be recorded separately (drawings)

38
Chapter 1: Questions

39
1: QUESTIONS

1.1 In a sole trader and a partnership the owners are personally liable if the business cannot meet its debts.
Is this statement true or false?
A True
B False (1 mark)
1.2 If a limited liability company goes into liquidation will the shareholders have to make a financial
contribution to help the company pay its creditors?
A Yes
B No (1 mark)
1.3 Which of the following statements most accurately defines the business entity concept?
A The business must be treated as being separate from its owners.
B A business must be set up as a separate legal entity. (1 mark)

40
Chapter 1: Answers

41
1: ANSWERS

1.1 A

1.2 B

1.3 A

END OF CHAPTER

42
Home study chapter -
The regulatory framework 2
Syllabus Guide Detailed Outcomes
Having studied this chapter you will be able to:
• Understand the role of the regulatory system including the roles of the
– International Accounting Standards Committee Foundation (IASCF)
– International Accounting Standards Board (IASB)
– Standards Advisory Council (SAC)
– International Financial Reporting Interpretations Committee (IFRIC)
• Understand the role of International Financial Reporting Standards (IFRS)

Exam Context
Questions on this chapter will be knowledge based and so it is important that you are familiar with the role of each body.
The role of IFRIC was tested in the Pilot Paper.

Qualification Context
Financial Accounting introduces the International Accounting Standards Board's role in issuing IFRSs and paper F3
examines some key standards. All of these standards are built upon in the Fundamentals level paper Financial
Reporting (F7) and the Professional level paper Corporate Reporting (P2).

43
2: HOME STUDY CHAPTER - THE REGULATORY FRAMEWORK

Overview

Regulatory
framework

IASCF

SAC IASB IFRIC

Issue IFRS

44
2: HOME STUDY CHAPTER - THE REGULATORY FRAMEWORK

1 Introduction
1.1 Financial statements are produced by an entity's managers in order to show its owners how
the entity has performed over a period of time.

1.2 Company financial statements particularly need to show a true and fair view.
This means a system of regulation is necessary to ensure that financial statements are
produced to a high standard and are comparable across different companies.

2 Regulatory system
2.1

International Accounting
Standards Committee Foundation
(IASCF)
(22 Trustees)

Standards Advisory Council


(SAC) International Accounting
Standards Board
(IASB) International Financial Reporting
(14 Board members) Interpretations Committee
(IFRIC)
Key:
Appoints
Reports to
Advises

International Accounting Standards Committee Foundation (IASCF)


2.2 The IASCF is a not-for-profit organisation based in the United States which heads up the
regulatory system.
Its Trustees appoint members to the IASB, IFRIC and SAC. They also oversee the
regulatory system and raise the finance necessary to support it.
It has no involvement in the standard setting process.

International Accounting Standards Board (IASB)


2.3 The IASB's principal aim is to develop a single set of high quality accounting standards:
International Financial Reporting Standards (IFRS).
Section 1.4.1 It also liaises with national accounting standard setters (for example the UK's ASB) to
achieve convergence in accounting standards around the world.

45
2: HOME STUDY CHAPTER - THE REGULATORY FRAMEWORK

International Financial Reporting Interpretations Committee (IFRIC)


2.4 The IFRIC issues guidance on both how to apply existing IFRSs in company financial
statements and how to account for new financial reporting issues where no IFRS exists. It
reports to the IASB.

Standards Advisory Council (SAC)


2.5 The SAC's principal role is to advise the IASB on a range of issues which include:
• The IASB's agenda and timetable for developing IFRSs
• Advising the IASB of areas that may need to be considered by IFRIC.

3 The role of International Financial Reporting


Standards (IFRS)
3.1 IFRSs provide guidance as to how items should be shown in a set of financial statements
both in terms of their monetary amount and any other disclosure.
For example: IAS 2: Inventory states at what amount a company should value its inventory
and also requires that the financial statements breakdown the inventory figure between its
components such as raw materials, work in progress and finished goods.

3.2 If a company follows the relevant accounting standards its financial statements should show
a true and fair view.

Lecture example 1 Exam standard question for 1 mark

What is the role of the International Accounting Standards Committee Foundation?


A To appoint members of the IASB
B To advise the IASB on new accounting standards they should consider issuing.

Solution

46
2: HOME STUDY CHAPTER - THE REGULATORY FRAMEWORK

Lecture example 2 Exam standard question for 1 mark

Which of the following bodies is involved is trying to achieve convergence of global accounting
standards?
A IASB
B IFRIC

Solution

4 Chapter summary
Section Topic Summary
1 Introduction Financial statements are relied on by many different
user groups to make economic decisions. A system of
regulation is therefore necessary to ensure that the
information produced is of a high standard.
2 Regulatory system The IASCF appoints members to the IASB, IFRIC and
SAC.

The IASB issues International Financial Reporting


Standards.

The IFRIC issues guidance on how to apply accounting


standards.
The SAC advises the IASB on its agenda.
3 The role of International financial reporting standards give
international financial guidance as to how transactions should be recorded in
reporting standards the accounts.

47
2: HOME STUDY CHAPTER - THE REGULATORY FRAMEWORK

Chapter Summary

Regulatory
framework

IASCF

• 22 Trustees who:
- appoint members to the IASB, IFRIC and SAC
- oversee the regulatory system
- raise finance to support the system
• Not involved in standard setting process

SAC IASB IFRIC

• Aim: to advise the IASB on • Aim to develop a single set of high quality • Issues guidance on how to
- their agenda and timetable for accounting standards (IFRS) apply existing IFRS
developing IFRS • Liaises with national accounting standard
- areas that may need to be considered setters (for example the UK's ASB)
by IFRIC

Issue IFRS

48
Chapter 2: Questions

49
2: QUESTIONS

2.1 Accounting standards are prepared by


A the IASB
B the IASC Foundation
C the IAASB (1 mark)

2.2 Which of the following best describes the role of The International Financial Reporting Interpretations
Committee?
A Issues International Financial Reporting Standards.
B Provides advice on the development of standards.
C Interprets International Financial Reporting Standards. (1 mark)

50
Chapter 2: Answers

51
2: ANSWERS

2.1 A

2.2 C

END OF CHAPTER

52
Accounting conventions 3
Syllabus Guide Detailed Outcomes
Having studied this chapter you will be able to:
• Define, understand and apply accounting concepts and qualitative characteristics.
• Understand the balance between qualitative characteristics.
• Identify and explain the main characteristics of alternative valuation bases (for example net realisable value).
• Understand the advantages and disadvantages of historical cost accounting.
• Understand the provision of International Financial Reporting Standards governing financial statements regarding
changes in accounting policies.
• Identify the appropriate accounting treatment if a company changes a material accounting policy.

Exam Context
Questions on this chapter are likely to test your understanding of the qualitative characteristics of information. For
example, the Pilot Paper required you to identify the factors that make information reliable. Questions may also ask you
to define accounting conventions.

Qualification Context
Your understanding of the remaining chapters of IASB Framework will be developed in the Fundamentals level paper
Financial Reporting (F7). You should also expect to see more detailed calculations on IAS 8 tested in Paper F7.

53
3: ACCOUNTING CONVENTIONS

Overview

The objective of financial Underlying assumptions


statements

IASB Framework

Qualitative characteristics of Elements of financial statements


financial information

Accounting conventions

Other issues

Concepts and conventions Alternative valuation bases

IAS 8: Accounting policies,


changes in accounting
estimates and errors

54
3: ACCOUNTING CONVENTIONS

1 Introduction
1.1 As noted in Chapter 2 financial statements should show a true and fair view of, or present
fairly, the entity's activities. They are produced to provide information to the entity's owners.

1.2 In order for this information to be useful it must possess certain characteristics.

2 The IASB's Framework for the Preparation and


Presentation of Financial Statements
Conceptual framework
2.1 The IASB's Framework is not an accounting standard.

2.2 It is a set of principles which underpin the foundations of financial accounting.

2.3 Whenever a new accounting standard is issued it will be based on the principles of the IASB
Framework. Furthermore its principles should be applied to account for any item where no
accounting standard exists.

2.4 The Framework is divided into seven sections.

1) The objective 2) Underlying 3) Qualitative


of financial assumptions characteristics
statements of financial
information

7) Concepts of
4) The elements
of financial Framework capital and capital
statements maintenance

5) Recognition in 6) Measurement of the


financial statements elements of financial
statements

Only sections 1 – 4 are examinable at Paper F3.

The objective of financial statements


2.5 To provide information about the financial position, financial performance and changes
in financial position of an entity that is useful to a wide range of users in making economic
decisions.

55
3: ACCOUNTING CONVENTIONS

Underlying assumptions
2.6 Accruals basis
The effects of transactions and other events are recognised when they occur (and not as
cash or its equivalent is received or paid) and they are recorded in the accounting records
and reported in the financial statements of the period to which they relate.
Going concern
The financial statements are normally prepared on the assumption that an entity is a going
Quick Quiz Q3 concern and will continue in operation for the foreseeable future.
If this is not appropriate, then additional disclosure about the basis of preparation must be
made in the financial statements.

Qualitative characteristics of financial information


2.7

Quick Quiz Q4, Comparability


Q10
Understandability

• Information should be readily • For same entity over different


understandable by users periods: consistency
who are assumed to have
• Between different entities:
reasonable knowledge
disclosure of accounting
policies

Relevance Reliability

• Assist users in evaluating • Faithful representation


past and predicting future • Substance over form
events • Neutrality
• Materiality • Prudence
• Completeness

56
3: ACCOUNTING CONVENTIONS

2.8 The elements of financial statements


The five elements of financial statements and their definitions are listed below.

Asset
A resource controlled by an entity as a result of past events and from which future
economic benefits are expected to flow to the entity.

Liability
A present obligation of the entity arising from past events, the settlement of which is
expected to result in an outflow of economic benefits.

Equity
The residual interest in the assets of an entity after deducting all its liabilities, so
EQUITY = NET ASSETS = SHARE CAPITAL + RESERVES

Income
Increases in economic benefits during the accounting period in the form of inflows or
enhancements of assets or decreases of liabilities that result in increases in equity, other
than those relating to contributions from equity participants.

Expenses
Decreases in economic benefits during the accounting period in the form of outflows or
depletions of assets or increases of liabilities that result in decreases in equity, other
than those relating to distributions to equity participants.

3 Alternative valuation bases


Historic cost
3.1 Financial statements are generally produced using the historical cost convention where
items are recorded at their historic cost.
For example, if an entity purchased a building in 20X6 for $1 million then the building would
be recorded as an asset at $1 million. The $1 million asset would then be depreciated to
reflect the wearing out of the building. However, in reality, the value of the building may
appreciate over time depending on market values.

57
3: ACCOUNTING CONVENTIONS

Lecture example 1 Idea generation

What are the advantages and disadvantages of recording the building at its historic cost of
$1 million (consider the Framework's qualitative characteristics)?

Solution
Advantages of historic cost
(1)
(2)
(3)
Disadvantages of historic cost
(1)
(2)

3.2 Note that in times of rising prices using the historical cost convention will lead to asset
values being too low and profits too high in a set of financial statements.

3.3 Due to the limitations of historic cost, alternative valuation bases exist. They are:
• replacement cost
• net realisable value
• economic value

Replacement cost
3.4 Assets are carried at the amount it would cost to acquire an equivalent asset today.
Liabilities are shown at the amount that would be required to settle the obligation today.
Replacement cost is also known as 'current cost'.

Net realisable value


3.5 This values items at their expected selling price less any costs that need to be incurred
before the item can be sold.
Inventory should always be shown in the financial statements at the lower of cost (historic
cost) and net realisable value.

58
3: ACCOUNTING CONVENTIONS

Lecture example 2 Preparation question

A Ltd has 100 items in inventory at the year end. The following information is available:
$
Total cost of items to date 1,000
Expected selling price per item 11
Costs which still need to be incurred per item before item can be sold 2

Required
(a) What is the historic cost of the inventory? $
(b) What is the net realisable value of the inventory? $
(c) What value for inventory should be shown in the financial statements? $

Workings

Economic value
3.6 This is the value of an item derived from its ability to generate net cash flows. It can also be
known as 'present value'.
Quick Quiz Q6 For example, the economic value of a machine would be calculated by determining the
value in today's prices, of the future cash inflows from selling items produced by the
machine less the related cash outflows.

59
3: ACCOUNTING CONVENTIONS

60
3: ACCOUNTING CONVENTIONS

Additional Notes

61
3: ACCOUNTING CONVENTIONS

4 Other examinable concepts and conventions


4.1 In addition to the concepts and conventions set out in the Framework, the following are also
relevant in the preparation of financial statements.

The business entity concept


4.2 This recognises the distinction between the business and its activities and the owners or
managers themselves. This is particularly important when considering sole trader or
partnership accounts as these businesses are not separately identified by law.

The money measurement concept


4.3 Only items which are capable of being measured in monetary terms should be recognised
in the financial statements. For example, even though a loyal workforce may be of benefit to
a business this value cannot be measured in monetary terms and is therefore not included
on the balance sheet.

The duality concept


4.4 Every transaction has two effects. This is the underlying principle of the double entry
accounting system.

The historical cost convention


4.5 Assets and liabilities are recorded at their historical cost. This is the amount recorded when
the transaction took place.

5 IAS 8: Accounting policies, changes in accounting


estimates and errors
Accounting policies – definition
5.1 Accounting policies are the significant principles, bases, conventions, rules and practices
applied by an entity in preparing and presenting the financial statements.
It is the way the entity has decided to treat an item in its financial statements, for example
whether non-current assets are carried at historic cost or a revalued amount.

Changes in accounting policy


5.2 Changes in accounting policy will only arise if:
(a) There is a new accounting standard or statutory requirement.
(b) Using the new policy makes the financial statements more relevant and reliable.

62
3: ACCOUNTING CONVENTIONS

Accounting treatment
5.3 Financial statements contain two years worth of figures. For example a company whose
year end is 31 December 20X7 will show information for 20X7 and 20X6.
The current year figures (20X7) will be produced using the new accounting policy.
In order for the financial statements to be comparable over time the comparative figures
(20X6) will be restated. This means they will be reproduced and drawn up using the 20X7
accounting policies.

5.4 Disclosure
The following disclosure should be made:
(a) The nature of the change in accounting policy
(b) The reasons for the change
(c) The amount of the adjustment in the current period and the comparative period.

Illustration:
Airport Parking purchased a plot of land in 20X1 for $250,000.
Airport Parking uses this plot of land as a car park and 'rents out' spaces to passengers using the
local airport.
The company deems the land to have an unlimited useful life and so does not depreciate it.
Airport Parking has always held the land at historic cost but now, in 20X7, wants to change its
accounting policy and revalue the land. Its current valuation is $800,000. The current value at the
end of 20X6 was $700,000.
The 20X7 financial statements must be produced using the new accounting policy (revaluation).
The 20X6 financial statements must be restated to show the land using the revaluation policy so
that the financial statements are comparable.

63
3: ACCOUNTING CONVENTIONS

The original 20X6 financial statements showed:


20X6
$'000
Land 250
Other assets 600
850

Share capital 100


Retained earnings 250
Liabilities 500
850
The 20X7 financial statements will show:
20X7 20X6
Restated
$'000 $'000
Land 800 700
Other assets 600 600
1,400 1,300

Share capital 100 100


Revaluation reserve (450 + (800 – 700)) 550 450 (700 – 250)
Retained earnings 250 250
Liabilities 500 500
1,400 1,300
The 20X7 financial statements will disclose:
(1) The nature of the change in accounting policy
(2) The reasons for the change
(3) The amount of the adjustment in the current period and the comparative period.
A suitable disclosure might say:
'During 20X7 Airport Parking changed its accounting policy for land. Land was previously held at
historic cost but the directors have now decided to revalue the land and show it at its current value.
This is in order to show more relevant information in the financial statements.
The 20X6 financial statements have been restated and are produced using the policy of
revaluation. The land has been increased to its value in 20X6 of $700,000 and a revaluation
reserve of $450,000 has been established. This increases restated net assets by $450,000.
In 20X7 the land was revalued by a further $100,000 in order to include the 20X7 valuation in the
financial statements'.

64
3: ACCOUNTING CONVENTIONS

Errors
5.5 These are material omissions from, or misstatements in, the financial statements that ought
to have been identified before the financial statements were finalised.
An error is accounted for in exactly the same way as a change in accounting policy.
For example, an entity may discover a material error in the 20X6 figures whilst producing the
20X7 financial statements. When the 20X7 financial statements are produced the 20X6
comparatives should be restated and the error corrected.

Illustration:
Capital Co depreciates its non-current assets on the reducing balance basis using a rate of 25%.
The 20X6 financial statements showed the following information:

20X6
$'000
Non-current assets – net book value 500
Other assets 400
900

Share capital 50
Retained earnings 600
Liabilities 250
900
During the preparation of the 20X7 financial statements it was discovered that the accountant in
charge of non-current assets had forgotten to charge depreciation in 20X6.
The 20X6 financial statements should be restated to correct this error.
The 20X7 financial statements will show:
20X7 20X6
Restated
$'000 $'000
Non-current assets – net book value (375 – 93.75) 281.25 375 (500 – 125)
Other assets 400 400
681.25 775

Share capital 50 50
Retained earnings (475 – 93.75) 381.25 475 (600 – 125)
Liabilities 250 250
681.25 775

5.6 Note that changes in accounting estimates are not examinable.

65
3: ACCOUNTING CONVENTIONS

6 Chapter summary
Section Topic Summary

2 The IASB’s Framework The IASB Framework provides a set of principles on


for the Preparation and which financial accounting is based.
Presentation of
Financial Statements The objective of financial statements is to provide
information on an entity’s financial position, financial
performance and financial adaptability.
The accruals basis requires that transactions are
recognised when they occur rather than when any
cash is received or paid.
The going concern basis assumes that the entity will
continue in operation for the foreseeable future.
In order for the information in the financial statements to
be useful it should posses the qualitative
characteristics of understandability, relevance,
reliability and comparability.
3 Alternative valuation Most transactions are recorded at their historic cost -
bases whilst this is a reliable figure it can soon become out of
date, especially when prices are rising.
Alternative valuation bases exist and these include:
replacement cost, net realisable value and economic
value (present value).
4 Other examinable The business entity concept states that a business is
concepts and a separate entity from its owners.
conventions The money measurement concept states that only
items which can be measured in monetary terms can
be shown in the financial statements.
The duality concept forms the basis of double entry
bookkeeping and states that every transaction has two
effects.
The historic cost convention records items at their
value when the transaction took place.
5 Accounting policies, An entity should apply its accounting policies
changes in accounting consistently from year to year.
estimates and errors If an entity changes its accounting policy this is
(IAS 8 applied retrospectively and the comparative figures are
restated and drawn up using the current year policies.
The correction of an error also requires the
comparative figures to be restated to correct the error.

66
3: ACCOUNTING CONVENTIONS

Chapter Summary

The objective of financial Underlying assumptions


statements

'To provide information about • Accruals


the financial position, financial • Going concern
performance and changes in
financial position of an entity' IASB Framework

Qualitative characteristics of Elements of financial statements


financial information

• Understandability • Asset
• Relevance • Liability
• Reliability • Equity
• Comparability • Income
Accounting conventions • Expenses

Other issues

Concepts and conventions Alternative valuation bases

• Business entity • Historic cost


• Money measurement • Replacement cost
• Duality • Net realisable value
IAS 8: Accounting policies, • Economic value
changes in accounting
estimates and errors

• Changes in accounting policies


• Errors

67
3: ACCOUNTING CONVENTIONS

68
Chapter 3: Questions

69
3: QUESTIONS

3.1 The Framework for the Preparation and Presentation of Financial Statements identifies two assumptions
which are the bedrock of accounting. What are they?
A Consistency and prudence
B Accruals and going concern
C Materiality and separate entity (1 mark)
3.2 In which of the following circumstances can a change of accounting policy be made?
(i) If the directors want to improve the value of the statement of financial position
(ii) If required by an accounting standard
(iii) If it results in reliable and more relevant information
A (ii) only
B (i) and (ii)
C (ii) and (iii)
D (i), (ii) and (iii) (2 marks)

70
Chapter 3: Answers

71
3: ANSWERS

3.1 B

3.2 C

END OF CHAPTER

72
Sources, records and
books of prime entry 4
Syllabus Guide Detailed Outcomes
Having studied Chapters 4 and 5 you will be able to:
• Identify and explain the function of the main data sources in an accounting system and how the accounting
system provides useful information.
• Outline the contents and purpose of different types of business documentation such as an invoice.
• Identify the main types of business transactions, for example, sales, purchases, payments and receipts.
• Understand and apply the concept of double entry accounting, the duality concept and the accounting equation.
• Identify the main types of ledger account and illustrate how to balance and close a ledger account.
• Understand and illustrate the uses of journals and the posting of journal entries into ledger accounts.
• Identify correct journals from given narrative.
• Record credit sale, credit purchase and cash transactions in ledger accounts and day books.
• Understand and record sales and purchase returns.
• Understand the need for a record of petty cash transactions and security over the petty cash system.
• Describe the features and operation of a petty cash imprest system.
• Account for petty cash using imprest and non-imprest methods.

Exam Context
Questions are unlikely to feature solely on this chapter; however, you should have a good understanding of what
constitutes an asset, liability, capital, income and expense. You should also be aware of the principal contents of each
book of prime entry and the purpose of the memorandum ledgers.

Qualification Context
These topics are only examined in Financial Accounting.

73
4: SOURCES, RECORDS AND BOOKS OF PRIME ENTRY

Overview

Statement of financial Income statement


position

Sources, records and


books of prime entry

Books of prime entry Memorandum ledgers

Cash book Sales day Purchase day Petty cash Journal book
book book book

74
4: SOURCES, RECORDS AND BOOKS OF PRIME ENTRY

1 Statement of financial position


1.1 An individual could prepare a list of everything they own and everything they owe.

Lecture example 1 Idea generation

Required
List out everything you own and owe.

Solution
(a) Own

(b) Owe

1.2 For a business, this list is formalised as a statement of financial position and show the
entity's assets and liabilities.
(a) Asset: is a resource controlled by the entity as a result of past events and from
which future economic benefits are expected to flow to the entity.
(b) Liability: is a present obligation of the entity arising from past events, the
settlement of which is expected to result in an outflow of economic benefits.

75
4: SOURCES, RECORDS AND BOOKS OF PRIME ENTRY

Proforma statement of financial position – sole trader


1.3 Statement of financial position as at 31 December 20X7:
$ $
ASSETS
Non-current assets
Land and buildings 100,000
Office equipment 50,000
Motor vehicles 30,000
Furniture and fixtures 20,000
200,000
Current assets
Inventories 50,000
Trade receivables 30,000
Less: allowance for receivables (2,000)
28,000
Prepayments 5,000
Cash in hand and at bank 7,000
90,000
Total assets 290,000

CAPITAL AND LIABILITIES


Capital
Capital 170,000
Profit 45,000
Less: drawings (25,000)
190,000
Non-current liabilities
Bank loans 40,000

Current liabilities
Bank overdraft 16,000
Trade payables 40,000
Accruals 4,000
60,000
Total capital and liabilities 290,000

Key features
1.4 (a) Always headed as at, for the date of the statement of financial position.
(b) Non-current assets - assets held and used in the business over the long-term (i.e.
more than one year).
(c) Current assets - not non-current assets! Conventionally listed in increasing order of
liquidity (i.e. closeness of assets to cash).
(d) Capital - what the business owes the proprietor/owner. In this case the sole trader
owns all of the business, i.e. its total net worth.
∴ CAPITAL = ASSETS - LIABILITIES
= NET ASSETS

76
4: SOURCES, RECORDS AND BOOKS OF PRIME ENTRY

(e) Don't include a caption (item heading) if there isn’t a value for it.
The statement of financial position is a snapshot of the business at one point in
time.

2 The income statement


Profit – example
2.1 Suppose a business buys three books for $10 each. Then it sells them for $15 each:
$
Sales 45 Income
Cost of sales (30) Expenditure
Gross profit 15
Profit is the excess of total income over total expenditure.
NOTE: The business may have other expenses such as rent, telephone bills, etc. to take off
before the ‘true’ profit is shown.

Proforma income statement - sole trader


2.2 Income statement for the year ended 31 December 20X7:
$ $
Sales 200,000
Less: Cost of sales
Opening inventories 40,000
Purchases 110,000
Carriage inwards 20,000
170,000
Closing inventories (50,000)
(120,000)
Gross profit 80,000
Sundry income 5,000
Discounts receivable 3,000
88,000
Less: Expenses
Rent 11,000
Carriage outwards 4,000
Telephone 1,000
Electricity 2,000
Wages and salaries 9,000
Depreciation 7,000
Bad and doubtful debts 3,000
Motor expenses 5,000
Discounts allowable 1,000
(43,000)
Profit for the year 45,000

77
4: SOURCES, RECORDS AND BOOKS OF PRIME ENTRY

Key features
2.3 (a) Headed up with the period for which the income and expenses are being included.
(b) The top part Sales X
Cost of sales (X)
Gross profit X
is called the trading account as it records just the trading activities (buying and
selling) of the business.
(c) Sundry income includes items like bank account interest.
(d) Do not include nil value captions.
The income statement is a summary of the business' performance over a period of
time – think of it as a DVD!

3 Relationship between the statement of financial


position and the income statement
3.1 Statement of financial position – shows the worth of business at a point in time.
Income statement – shows the trading activities over a period of time (financial
performance).

3.2 The accounting period is the period for which the income statement was prepared. This is
usually a year.

3.3 Therefore, there will be a statement of financial position at the beginning of the year (prior
year end) and at the end of the accounting period.
The income statement is for the intervening period.

Income statement for the year ended 31.12.X7

Statement Statement
of financial of financial
position position
as at as at
31.12.X6 31.12.X7

78
4: SOURCES, RECORDS AND BOOKS OF PRIME ENTRY

4 From business transactions to financial statements


4.1 A business will enter into a number and variety of transactions during an accounting period:

CASH TRANSACTIONS

Sales Purchases Wages Stationery Acquisition


of non-current
assets

CREDIT TRANSACTIONS

Sales Purchases

Ultimately all of these transactions must be summarised in the business' financial


statements (ie the statement of financial position and income statement).

4.2 This is achieved by having accounting records to record each stage of the process:

Assorted transactions
(eg invoices)

Categorised
(in Books of Prime Entry)

Summarised
(eg nominal ledger, trial balance)

FINANCIAL STATEMENTS
(eg Statement of Financial Position and Income Statement)

79
4: SOURCES, RECORDS AND BOOKS OF PRIME ENTRY

5 Books of prime entry


5.1 The business' transactions are categorised with other similar transactions in the books of
prime entry.

5.2
Books of prime entry

Cash book Sales day Purchase day Petty cash Journal book
book book book

Receipts Payments

Cash transactions Credit sales Credit purchases Small cash Adjustments


transactions and errors

Cash book
5.3 (a) Records receipts and payments into and out of the bank.
(b) For exam purposes often assumed to be two books, one for receipts, one for
payments.

Cash book (receipts)


5.4 Example:

Date Narrative Total Capital Sales Receivables


$ $ $ $
2.1.X7 F. Bloggs 4,000 4,000
5.1.X7 J. Spalding 200 200
6.1.X7 J. Smith 500 500
4,700 4,000 500 200

total cash reason why cash was


received received

80
4: SOURCES, RECORDS AND BOOKS OF PRIME ENTRY

Cash book (payments)


5.5 Example:
Date Narrative Total Purchases Van Rent Payables Petty cash Drawings
$ $ $ $ $ $ $

6.1.X7 Manley & 350 350


Co.
6.1.X7 Petty Cash 50 50
8.1.X7 Digby Co 1,000 1,000
1,400 1,000 350 50

total cash reason why payment


payment was made

Sales day book


5.6 Lists all sales made on credit, i.e. each individual invoice raised.

5.7 Example:

Date Customer $
3.1.X7 J. Spalding 200
5.1.X7 G. McGregor 400
8.1.X7 J. Spalding 400
14.1.X7 G. McGregor 300
TOTAL 1,300

Purchase day book


5.8 Lists all purchases made on credit, i.e. each individual invoice received.

5.9 Example:

Date Supplier $
1.1.X7 Tewson Co. 400
4.1.X7 Manley & Co. 350
16.1.X7 Manley & Co. 200
TOTAL 950

81
4: SOURCES, RECORDS AND BOOKS OF PRIME ENTRY

Petty cash book


5.10 (a) Records the movement of physical cash (kept on the premises) in and out of the
petty cash tin.
(b) Used for small incidental expenses.

5.11 Example:

Receipts Payments

Date Narrative Total Date Narrative Total Stationery Travel


$ $ $ $
6.1.X7 Cheque 50 7.1.X7 City 10 10
cashed Stationers
8.1.X7 F. Bloggs 2 2
Metro fare
12 10 2

Controlling petty cash – the imprest system


An imprest system acts as an accounting control by having a set amount of petty cash.
5.12 (a) Pre-set limit, say $50.
(b) Voucher filled in when money is taken out to pay expenses.
(c) At any time, vouchers + cash = pre-set limit.
Quick Quiz Q5, Q6
(d) At the end of the week/month, the petty cash book is filled in from the vouchers.
(e) The amount needed to bring the balance back up to the pre-set limit = money spent.

Journal book
5.13 Certain transactions do not ‘fit’ in the main books, for example:
(a) period end adjustments
(b) correction of errors
The journal book lists these sundry transactions.

6 Memorandum ledgers
Purpose
6.1 To know how much is owed by a particular customer or to a certain supplier at a point in
time.
For example, the sales day book shows the sales made on credit to all customers and the
cash book receipts shows the cash received from all sources.
J. Spalding owes the business $400 but this cannot be seen from the books of prime entry
without trawling back through the detailed information.
A separate memorandum ledger is kept to show this information.

82
4: SOURCES, RECORDS AND BOOKS OF PRIME ENTRY

6.2 There are two types of memorandum ledgers kept by the business:
(a) Receivables ledger – showing how much is owed by each individual customer.
(b) Payables ledger – showing how much is owed to each individual supplier.

6.3 The entries in these ledgers are made by rearranging the information in the day books into
individual customer and supplier accounts.

Receivables ledger
6.4 Example:
J. Spalding (Customer)

Date Narrative Sales Cash Total


$ $ $
3.1.X7 Invoice 1032 200 200
5.1.X7 Cash received 200 –
8.1.X7 Invoice 1101 400 400

G. McGregor (Customer)

Date Narrative Sales Cash Total


$ $ $
5.1.X7 Invoice 1033 400 400
14.1.X7 Invoice 1129 300 700

Payables ledger
6.5 Example:
Tewson Co. (Supplier)

Date Cash Purchases Total


$ $ $
1.1.X7 Invoice A112 400 400

Manley & Co. (Supplier)

Date Cash Purchases Total


$ $ $
4.1.X7 Invoice 063 350 350
6.1.X7 Cash book 350 –
16.1.X7 Invoice 097 200 200

83
4: SOURCES, RECORDS AND BOOKS OF PRIME ENTRY

7 Chapter summary
Section Topic Summary
1 Statement of financial The statement of financial position shows the assets
position and liabilities of a business at a particular point in
time.
2 The income statement The income statement shows its performance over a
period.
3 The relationship The income statement largely explains the movement
between the statement between the business’ assets and liabilities at the
of financial position beginning of the year and at the end of the year.
and the income
statement
4 From business A business will enter many transactions during the year.
transactions to All of these need to be recorded and summarised to
financial statements produce the entity’s financial statements.
5 Books of prime entry The business’ transactions must first be categorised
into the books of prime entry. The cash book
records money paid in to and out of the bank account;
the sales day book records credit sales; the purchase
day book records credit purchases; the petty cash
book records transactions made in petty cash and the
journal book is used to correct errors and make other
adjustments such as accruals and prepayments. The
totals on these books are then summarised in the
nominal ledger.
6 Memorandum ledgers There are two memorandum ledgers: the receivables
ledger and the payables ledger. The receivables
ledger shows how much the business is owed by
each individual customer at a point in time and the
payables ledger show how much it owes to each
individual supplier at any point in time.

84
4: SOURCES, RECORDS AND BOOKS OF PRIME ENTRY

Chapter Summary
• Shows the assets and liabilities of the • Shows the income and expenses for a business
business at a point in time. over a period of time, usually a year

Statement of financial Income statement


position

Sources, records and


books of prime entry
• Receivables ledger:
- amount owed by a particular
customer
• Payables ledger:
- amount owed to a particular
supplier

Books of prime entry Memorandum ledgers

'Categorise similar transactions together'

Cash book Sales day book Purchase day Petty cash book Journal book
book
• Cash receipts • Credit sales • Credit purchases • Small cash transactions • Correction of errors
into the bank made via the petty cash and period end
• Cash tin adjustments
payments from
the bank

85
4: SOURCES, RECORDS AND BOOKS OF PRIME ENTRY

86
Chapter 4: Question

87
4: QUESTION

4.1 Which of the following is not a book of prime entry?


A Wages day book
B Cash book
C Sales ledger (1 mark)

88
Chapter 4: Answer

89
4: ANSWER

4.1 C

END OF CHAPTER

90
Ledger accounts
and double entry 5
Syllabus Guide Detailed Outcomes
Having studied Chapters 4 and 5 you will be able to:
• Identify and explain the function of the main data sources in an accounting system and how the accounting
system provides useful information.
• Outline the contents and purpose of different types of business documentation such as an invoice.
• Identify the main types of business transactions, for example, sales, purchases, payments and receipts.
• Understand and apply the concept of double entry accounting, the duality concept and the accounting equation.
• Identify the main types of ledger account and illustrate how to balance and close a ledger account.
• Understand and illustrate the uses of journals and the posting of journal entries into ledger accounts.
• Identify correct journals from given narrative.
• Record credit sale; credit purchase and cash transactions in ledger accounts and day books.
• Understand and record sales and purchase returns.
• Understand the need for a record of petty cash transactions and security over the petty cash system.
• Describe the features and operation of a petty cash imprest system.
• Account for petty cash using imprest and non-imprest methods.

Exam Context
Your understanding of double entry will be crucial to passing Financial Accounting. Whilst an individual question may
not ask you to produce a double entry it will be instrumental in answering the question. For example, a question may ask
you to derive the income statement expense for electricity where amounts need to be accrued at the year end. You will
only get this right if you understand the double entry for recording expenses and accruals. A question could also
describe a transaction and ask you to identify the correct double entry to record this.

Qualification Context
Being confident at double entry will help you account for many of the more complex accounting standards you will meet
in the Fundamentals level paper, Financial Reporting (F7) and the Professional level paper, Corporate Reporting (P2).

91
5: LEDGER ACCOUNTS AND DOUBLE ENTRY

Overview

Ledger accounts
and double entry

Ledger accounts Double entry

Debit Credit

Balancing off

92
5: LEDGER ACCOUNTS AND DOUBLE ENTRY

1 Introduction
1.1 This chapter is designed to enable you to explain the principles of double entry and apply
these principles to the preparation of accounting records within the nominal/general ledger.

1.2 In Chapter 4 we saw how transactions were categorised in books of prime entry, the next
step is to summarise the information in a format nearer to that of the final financial
statements.

The nominal ledger


1.3 (a) Each item in the statement of financial position or income statement will have an
"account" (which might be a page in a book or a record on a computer).
(b) All the accounts are collected together in the nominal ledger.
(c) The books of prime entry are totalled up and two entries will be made in these
accounts with each of these totals – this is called double entry.

The dual effect


1.4 The method used stems from the fact that every transaction affects two things, for example:
(a) A sole trader pays $6,000 in the business bank account:
Cash increases by $6,000
Capital increases by $6,000
(b) A sole trader purchases goods on credit for $400:
Purchases increase by $400
Trade payables increase by $400

2 Ledger accounts (T-accounts)


2.1 Debit CAPITAL Credit
$ $

Decrease Capital Increase Capital

We make two entries from each total extracted from the books of prime entry, and call one a
Debit (Dr), and the other one a Credit (Cr).

TOTAL DEBITS = TOTAL CREDITS

93
5: LEDGER ACCOUNTS AND DOUBLE ENTRY

Principles of double entry bookkeeping


2.2 The cash account is a good starting point:
Dr CASH Cr
$ $

CASH IN = DEBIT CASH OUT = CREDIT

General rules
2.3 (a) DEBIT entry represents:
(i) an increase in an asset;
(ii) a decrease in a liability;
(iii) an item of expense.
(b) CREDIT entry represents:
(i) an increase in a liability;
(ii) a decrease in an asset;
(iii) an item of income.
This can be remembered as follows

Debits Credits
(increase) (increase)
Quick Quiz
Q1 - 4 Expenses Liabilities

Assets Income

Drawings Capital

94
5: LEDGER ACCOUNTS AND DOUBLE ENTRY

Lecture example 1 Preparation question

Required
What is the double entry for each of the following?
Explain each entry in terms of the general rules above.

Solution

Transaction Debit Credit

(a) Sales for cash.

(b) Sales on credit.

(c) Purchase for cash.

(d) Purchase on credit.

(e) Pay electricity bill.

(f) Receive cash from a credit customer.

(g) Pay cash to a credit supplier.

95
5: LEDGER ACCOUNTS AND DOUBLE ENTRY

Transaction Debit Credit

(h) Borrow money from the bank.

Lecture example 2 Technique demonstration

Douglas
Douglas had the following transactions during January:
(1) Introduced $5,000 cash as capital;
(2) Purchased goods on credit from Richard, worth $2,000;
(3) Paid rent for one month, $500;
(4) Paid electricity for one month, $200;
(5) Purchased car for cash, $1,000;
(6) Sold half of the goods on credit to Tish for $1,750;
(7) Drew $300 for his own expenses;
(8) Sold goods for cash, $2,100.
Required
Post transactions (1) to (8) to the relevant ledger accounts.

Solution
Cash
$ $

Capital
$ $

96
5: LEDGER ACCOUNTS AND DOUBLE ENTRY

Trade payables
$ $

Purchases
$ $

Rent
$ $

Electricity
$ $

Car
$ $

Drawings
$ $

97
5: LEDGER ACCOUNTS AND DOUBLE ENTRY

Trade receivables
$ $

Sales
$ $

3 Flow of information
3.1 In Lecture example 2 the original transactions were posted to the ledger accounts. A
business would firstly categorise this information in the books of prime entry. The totals
from the books of prime entry are then posted to the nominal ledger using double entry.
3.2

98
5: LEDGER ACCOUNTS AND DOUBLE ENTRY

4 Balancing off the ledger accounts


4.1 The totals from the books of prime entry may be posted to the nominal ledger each month. A
business will want to know the balance on each account. This is done by 'balancing off' each
account.

Lecture example 3 Technique demonstration

The following information has been posted to the cash account below.
Required
Balance off the cash account to determine the amount of cash held at the end of January.

Solution
Dr Cash Cr
$ $
2/1 Sales 500 1/1 Purchases 300
10/1 Sales 500 25/1 Telephone 50

Steps
4.2 (1) Add the debit and credit sides separately.
(2) Fill in the higher of the two totals on both sides.
(3) Literally 'balance' the account (what number do we need and on which side to make
the two sides equal?) – balance c/d
(4) Complete the 'double entry' – balance b/d on opposite side.

Lecture example 4 Technique demonstration

Douglas
Refer to Lecture example 2 on page 96.
Required
Balance off the ledger accounts for Douglas

Solution
Complete in the solution space for Lecture example 2.

99
5: LEDGER ACCOUNTS AND DOUBLE ENTRY

5 Chapter summary
Section Topic Summary

1 Introduction In chapter 4 the totals on the books of prime entry were


summarised in the nominal ledger. These amounts are
posted to the nominal ledger using double entry.
The principles of double entry work on the basis that for
each debit entry there must be a credit entry. This is
also known as the dual effect.
2 Ledger accounts A debit entry increases assets, expenses and drawings
and a credit entry increases liabilities, income and
capital – this can be remembered as DEAD CLIC.
3 Flow of information A business’ transactions are categorised in the books of
prime entry and the totals are then posted to the
nominal ledger. A trial balance (chapter 6) can then be
extracted from the balances on the nominal ledger
accounts and the statement of financial position and
income statement produced.
4 Balancing off the At the end of each period the nominal ledger accounts
ledger accounts (T accounts) are 'balanced off' to determine the closing
balance on each account.

100
5: LEDGER ACCOUNTS AND DOUBLE ENTRY

Chapter Summary

Ledger accounts
and double entry

Ledger accounts Double entry

• The totals from the books of prime entry


are totalled and then are posted to the
nominal ledger.

Debit Credit

• increases: • increases:
- expenditure - liability
Balancing off - asset - income
- drawings - capital

• Steps:
(1) Add the debit and credit sides
separately
(2) Fill in the higher of the two totals
on both sides
(3) Balance the account by inserting
the 'balance c/d' on the relevant
side
(4) Complete the double entry and put
the 'balance b/d' on the opposite
side

101
5: LEDGER ACCOUNTS AND DOUBLE ENTRY

102
Chapter 5: Questions

103
5: QUESTIONS

5.1 A credit balance of $3,000 brought down on X Co’s account in Y Co’s books means that
A X Co is owed $3,000 by Y Co
B Y Co is owed $3,000 by X Co
C Y Co has sold $3,000 of goods to X Co (1 mark)
5.2 Which of the statements below best describes the nominal ledger?
A A list of all assets and liabilities at a point in time
B A collection of accounts to record the transactions of the business
C A record of amounts owed to/from individual suppliers and customers
D An initial record of internally generated transactions (2 marks)

104
Chapter 5: Answers

105
5: ANSWERS

5.1 A The balance represents the outstanding amount i.e. purchases less cash paid.

5.2 B

END OF CHAPTER

106
From trial balance to
financial statements 6
Syllabus Guide Detailed Outcomes
Having studied this chapter you will be able to:
• Identify the purpose of a trial balance.
• Extract ledger balances into a trial balance.
• Prepare extracts of an opening trial balance.
• Identify and understand the limitations of a trial balance.

Exam Context
Questions on this chapter may require you to derive missing figures (for example, profit for the period) using the
accounting equation and identify the correct double entry to record transactions such as closing inventory or drawings.

Qualification Context
Financial Accounting is the only paper where you are required to produce financial statements for a sole trader.
Financial statements for limited liability companies are tested in detail in the Fundamentals level paper, Financial
Reporting (F7) and the Professional level paper, Corporate Reporting (P2).

107
6: FROM TRIAL BALANCE TO FINANCIAL STATEMENTS

Overview

Trial balance Closing inventory adjustment

From trial balance


to financial statements

Income statement Statement of financial


position

Accounting equation

108
6: FROM TRIAL BALANCE TO FINANCIAL STATEMENTS

1 Introduction
1.1 We saw in Chapters 4 and 5 that:
• transactions are categorised in the books of prime entry;
• the totals are then posted to the ledger accounts in the nominal ledger using double
entry;
• the ledger accounts are then balanced off and the balances brought down.

2 The trial balance


2.1 The trial balance consists of a list of the balances brought down on each ledger account,
separated in to debits and credits as below.

Example
2.2 Miss Smith – Trial Balance at as 31 December 20X7:

Account Debit Credit


$ $

Cash 720
Capital 500
Sales 2,200
Purchases 1,100
Furniture 500
Electricity 120
Telephone 60
Drawings 200
Total 2,700 2,700

2.3 The trial balance should balance, i.e.

Total debits = Total credits

If the trial balance doesn't balance then an error must have occurred.
The correction of errors is covered in Chapter 16.

109
6: FROM TRIAL BALANCE TO FINANCIAL STATEMENTS

Lecture example 1 Technique demonstration

Douglas
Refer to Lecture example 2 in Chapter 5 on pages 96 to 98 where the ledger accounts were
balanced off.
Using the ledger accounts for Douglas, prepare the trial balance as at the end of January.

Solution

110
6: FROM TRIAL BALANCE TO FINANCIAL STATEMENTS

3 The closing inventory adjustment


Objective
3.1 Whilst a business will purchase items to sell during the year it is unlikely that all of them will
have been sold by the year end.
The items still held at the year end are known as inventories.
These are an asset of the business and so should be included in inventories in the
statement of financial position.
Also when a business determines its profit for the year it should match the sales revenue
earned to the cost of goods it sold, i.e. cost of sales.

Lecture example 2 Preparation question

Colin opens a business selling cordless telephones. In the first month he buys 50 phones for $20
each, and sells 20 for $30 each. Complete the trading account below.

Solution
$ $
Sales

Cost of sales
Purchases
Less: closing inventories

Gross profit

Accounting treatment
3.2 The closing inventory adjustment is accounted for via a journal entry. The double entry is:
Dr Inventories (SOFP)
Cr Closing inventories (COS – I/S)

3.3 This adjustment is usually made after the preliminary trial balance has been prepared.
3.4 Last period's closing inventories will become this period's opening inventories. These items
will be sold in the year and so will form part of cost of sales. As the items are sold they will
no longer be an asset of the business and should be removed from the statement of
financial position. The double entry is:
Dr Opening inventories (COS – I/S)
Cr Inventories (SOFP)
This can be done as soon as the new period begins.

111
6: FROM TRIAL BALANCE TO FINANCIAL STATEMENTS

4 The income statement


4.1 The income statement is part of the double entry system and can be shown as a T-account.

Completing the income statement


4.2 The balances on all the income and expenditure T-accounts are transferred to the income
statement and the closing inventory adjustment is made.

4.3 The income and expenditure accounts have now been closed out and a new account will be
created for each income and expenditure item next year.

Lecture example 3 Technique demonstration

Douglas
Refer to Lecture example 1 on page 110.
The cost of goods remaining unsold at year end was $250.
Required
Prepare an income statement in ledger account form.

Solution

Income Statement a/c

112
6: FROM TRIAL BALANCE TO FINANCIAL STATEMENTS

5 The statement of financial position


Completing the statement of financial position
5.1 Statement of financial position:
(a) lists all ledger accounts with balances remaining;
(b) i.e. all assets and liabilities;
(c) is not part of double entry system so these balances are not transferred out.

5.2 At end of period, clear balances on income statement and drawings to capital account.

Lecture example 4 Technique demonstration

Douglas
Refer to Lecture example 1 on page 110 and lecture example 3 on page 112.
Required
Draw up an income statement for the period and a statement of financial position at the end of
January.

Solution
DOUGLAS
INCOME STATEMENT FOR THE MONTH OF JANUARY

$ $

Sales

Less cost of sales:

Purchases

Less: closing inventories

Gross profit

Less expenses:

Rent

Electricity

Net profit

113
6: FROM TRIAL BALANCE TO FINANCIAL STATEMENTS

DOUGLAS
STATEMENT OF FINANCIAL POSITION AS AT 31 JANUARY

NON-CURRENT ASSET $ $

Motor vehicle

CURRENT ASSETS

Inventories

Trade receivables

Cash

PROPRIETOR’S INTEREST $ $

Capital introduced on 1 January

Profit for the year

Less: drawings

Balance 31 January

CURRENT LIABILITIES

Trade payables

114
6: FROM TRIAL BALANCE TO FINANCIAL STATEMENTS

Lecture example 5 Technique demonstration

Douglas
Refer to Lecture example 4 on page 113.
Required
Transfer the profit and drawings to the capital account.

Solution

Drawings
5.3 Drawings are amounts being taken out of a business by its owner. Drawings are generally in
the form of cash, but an owner may also take inventory out of the business. Drawings of
inventories are recorded at the cost of the inventories not the sales price.

115
6: FROM TRIAL BALANCE TO FINANCIAL STATEMENTS

6 The accounting equation


6.1 The accounting equation expresses the statement of financial position as an equation.

6.2 At its most simple:

ASSETS = LIABILITIES
(debits) (credits)

Different types of liabilities (credits)

CAPITAL PROFIT PAYABLES


(less drawings)

Proprietor’s interest

Lecture example 6 Technique demonstration

Douglas
Refer to Lecture example 4.
Required
Prepare the accounting equation for Douglas.

Solution

116
6: FROM TRIAL BALANCE TO FINANCIAL STATEMENTS

7 Chapter summary
Section Topic Summary

2 The trial balance The trial balance consists of a list of the balances
brought down on each ledger account.
3 The closing inventory At the end of the year an adjustment must be made for
adjustment closing inventory to match sales revenue to the cost
of making those sales and also to reflect the fact that
the inventories are an asset of the business.
The opening inventory balance should also be
transferred to cost of sales.
4 The income statement The balances on all of the income and expenditure
ledger accounts are transferred to the income
statement along with any adjustments that will affect
profit (such as closing inventory).
5 The statement of The statement of financial position lists out the
financial position balances on all of the asset and liability ledger
accounts.
6 The accounting The accounting equation expresses the statement of
equation financial position as an equation:
Assets = capital + profit – drawings + payables

8 Double Entry Summary for Chapter 6


8.1 Closing inventory adjustment:

Dr Inventories (SOFP)
Cr Closing inventories (I/S)

8.2 Opening inventory adjustment:

Dr Opening inventories (I/S)


Cr Inventories (SOFP)

8.3 The accounting equation:

Assets = Liabilities
Assets = Capital + Profit - Drawings + Payables

117
6: FROM TRIAL BALANCE TO FINANCIAL STATEMENTS

Chapter Summary

Trial balance Closing inventory adjustment

'A list of the balances brought down on • Items held in inventories at the year end
each ledger account' should be recorded as an asset in the
statement of financial position and will
reduce cost of sales
• Record via a journal entry:
Dr Inventories (SOFP)
Cr Closing inventories (COS – I/S)

From trial balance


to financial statements

Income statement Statement of financial


position

Accounting equation

• Expresses the statement of financial position as an equation


• Assets = liabilities

118
Chapter 6: Questions

119
6: QUESTIONS

6.1 At the end of the accounting period and after the statement of financial position and income statement
have been prepared for a sole trader:
A All journals are reversed
B The balances on asset and liability accounts are transferred to the capital account
C The balances on the income statement and drawings account are transferred to the capital account
D Balances are carried forward on all the accounts in the nominal ledger (2 marks)

6.2 A business has cash of $1,100, trade payables of $2,500, a mortgage liability of $8,000 and land of
$16,000.
What is the proprietor's interest? $ (2 marks)

6.3 Joe, a sole trader, set up business on 1 October 20X6 with $40,000 of his own money. During the year to
30 September 20X7 he won $50,000 on the lottery and paid $30,000 of this into his business. He took
cash drawings of $5,000 during the year and at 30 September 20X7 the net assets of the business
totalled $59,000.
What was the profit or loss of the business for the year ended 30 September 20X7?
A $4,000 profit
B $6,000 profit
C $16,000 loss
D $6,000 loss (2 marks)
6.4 Joan
Joan, a second hand bookseller, has been in business for two months. In this time she:
(1) paid in cash $5,000 as capital;
(2) took the lease of a stall and paid two months’ rent. The annual rental was $1,200;
(3) purchased, on credit from J Fox, books at cost of $825;
(4) spent $420 cash on the purchase of other books from W Smith;
(5) paid an odd-job man $75 to paint the exterior of the stall and repair a broken lock;
(6) put an advertisement in the local paper at a cost of $10;
(7) sold three volumes containing "The Complete Works of Shakespeare" to an American for $60
cash;
(8) sold six similar sets on credit to a local school for $300;
(9) paid J Fox $525 on account for the amount due to him;
(10) received $200 from the school;
(11) purchased cleaning materials at a cost of $10 and paid a char lady $30;
(12) took $100 from the business to pay for her own personal expenses;
(13) made other cash sales during the two months of $1,500;
(14) all books had been sold by the end of two months.
Required
(a) Write up the relevant ledger accounts for these transactions.
(b) Balance off all of the ledger accounts.
(c) Prepare a trial balance, an income statement and a statement of financial position.

120
6: QUESTIONS

6.5 Brian
Brian set himself up in business on 1 January selling ice creams. During his first two months in business
he:
(1) Introduced $20,000 of cash as capital into the business;
(2) Purchased a second hand ice cream van from John. He paid John $10,500 cash;
(3) Paid Terry $200 to repair the ice cream machine in the van;
(4) Purchased on credit, inventories totalling $750;
(5) Spent $400 on petrol;
(6) Sold goods for $750 in cash;
(7) Paid $600 in tax and insurance;
(8) Made additional cash purchases of $80 for strawberry sauce and chocolate flakes;
(9) Withdrew $300 for his own expenses;
(10) The cost of goods remaining unsold was $500.
Required
(a) Post transactions (1) – (9) to the relevant ledger accounts.
(b) Balance off the ledger accounts.
(c) Prepare a trial balance.
(d) Prepare an income statement in ledger account form (remembering to deal with item 10).
(e) Draw up an income statement for the period and a statement of financial position at the end of
the period.
(f) Transfer the loss and drawings to the capital account.
6.6 Dealers
On 1 January the proprietor’s interest in a business, Dealers, was $18,500. At 31 January the assets and
liabilities of the business were as follows.
$
Plant and equipment 10,000
Motor vehicles 5,000
Trade payables 3,000
Trade receivables 2,000
Inventories 4,500
Accrued expenses 250
Balance in the bank 3,500
Cash in the till 250
On 7 January the proprietor had paid in additional capital of $2,000. On 14 January he had taken goods
at a cost of $350 for his own consumption and on 30 January had drawn cash of $1,250 from the
business, for his own personal expenditure.
Required
(a) Calculate the net asset value at 1 January.
(b) Calculate the net asset value of the business at 31 January.
(c) Calculate the profit of the business for the month of January.
(d) Show the accounting equation at 31 January.

121
6: QUESTIONS

122
Chapter 6: Answers

123
6: ANSWERS

6.1 C

6.2 $6,600
Dr Cr
$ $
Cash 1,100
Trade payables 2,500
Mortgage liability 8,000
Land 16,000
Proprietor's interest (balancing figure) 6,600
17,100 17,100

6.3 D
$
Net assets at 1.10.X6 40,000
Capital introduced 30,000
Drawings (5,000)
∴ loss for year (balancing figure) (6,000)
Net assets at 30.9.X7 59,000
6.4 Joan
Bank (SOFP)
$ $
(1) Capital 5,000 (2) Rent 200
(7) Sales 60 (4) Purchases 420
(10) Trade receivables 200 (5) Repairs 75
(13) Sales 1,500 (6) Advertising 10
(9) Trade payables 525
(11) Cleaning materials 10
(11) Cleaning 30
(12) Drawings 100
Balance c/d 5,390
6,760 6,760
Balance b/d 5,390

Capital (SOFP)
$ $
Balance c/d 5,000 (1) Bank 5,000
5,000 5,000
Balance b/d 5,000

Rent (I/S)
$ $
(2) Bank 200 Balance c/d 200
200 200
Balance b/d 200

124
6: ANSWERS

Trade payables (SOFP)


$ $
(9) Bank 525 (3) Purchases 825
Balance c/d 300
825 825
Balance b/d 300

Purchases (I/S)
$ $
(3) Trade payables 825 Balance c/d 1,245
(4) Bank 420
1,245 1,245
Balance b/d 1,245

Repairs (I/S)
$ $
(5) Bank 75 Balance c/d 75
75 75
Balance b/d 75

Advertising (I/S)
$ $
(6) Bank 10 Balance c/d 10
10 10
Balance b/d 10

Sales (I/S)
$ $
Balance c/d 1,860 (7) Bank 60
(8) Trade receivables 300
(13) Bank 1,500
1,860 1,860
Balance b/d 1,860

Trade receivables (SOFP)


$ $
(8) Sales 300 (10) Bank 200
Balance c/d 100
300 300
Balance b/d 100

Cleaning materials (I/S)


$ $
(11) Bank 10 Balance c/d 10
10 10
Balance b/d 10

125
6: ANSWERS

Cleaning (I/S)
$ $
(11) Bank 30 Balance c/d 30
30 30
Balance b/d 30

Drawings (SOFP)
$ $
(12) Bank 100 Balance c/d 100
100 100
Balance b/d 100

Trial Balance
Debit Credit
$ $
Bank 5,390
Capital 5,000
Rent 200
Trade payables 300
Purchases 1,245
Repairs 75
Advertising 10
Sales 1,860
Trade receivables 100
Cleaning materials 10
Cleaning 30
Drawings 100
7,160 7,160
Joan
Income statement for the two months ended……
$ $
Sales 1,860
Purchases (1,245)
Gross profit 615
Rent 200
Repairs 75
Advertising 10
Cleaning (10 + 30) 40
(325)
Profit for the year 290

126
6: ANSWERS

Joan
Statement of financial position as at….
$
Current Assets
Trade receivables 100
Bank 5,390
5,490

Proprietor's Interest $
Capital 5,000
Profit 290
Less: drawings (100)
5,190
Current Liabilities
Trade payables 300
5,490
6.5 Brian
(a) Bank (SOFP)
$ $
(1) Capital 20,000 (2) Van 10,500
(6) Sales 750 (3) Repairs & Maintenance 200
(5) Petrol 400
(7) Tax & Insurance 600
(8) Purchases 80
(9) Drawings 300

Capital (SOFP)
$ $
(1) Bank 20,000

Van (SOFP)
$ $
(2) Bank 10,500

Repairs & Maintenance (I/S)


$ $
(3) Bank 200

Purchases (I/S)
$ $
(4) Trade payables 750
(8) Bank 80

Trade payables (SOFP)


$ $
(4) Purchases 750

127
6: ANSWERS

Petrol (I/S)
$ $
(5) Bank 400

Sales (I/S)
$ $
(6) Bank 750

Tax & Insurance (I/S)


$ $
(7) Bank 600

Drawings (SOFP)
$ $
(9) Bank 300

(b) Bank (SOFP)


$ $
(1) Capital 20,000 (2) Van 10,500
(6) Sales 750 (3) Repairs & Maintenance 200
(5) Petrol 400
(7) Tax & Insurance 600
(8) Purchases 80
(9) Drawings 300

Bal c/d 8,670


20,750 20,750

Bal b/d 8,670

Capital (SOFP)
$ $
Bal c/d 20,000 (1) Bank 20,000
20,000 20,000
Bal b/d 20,000

Van (SOFP)
$ $
(2) Bank 10,500 Bal c/d 10,500
10,500 10,500

Bal b/d 10,500

Repairs & Maintenance (I/S)


$ $
(3) Bank 200 Bal c/d 200
200 200

Bal b/d 200

128
6: ANSWERS

Purchases (I/S)
$ $
(4) Trade payables 750
(8) Bank 80 Bal c/d 830
830 830

Bal b/d 830

Trade payables (SOFP)


$ $
Bal c/d 750 (4) Purchases 750
750 750

Bal b/d 750

Petrol (I/S)
$ $
(5) Bank 400 Bal c/d 400
400 400

Bal b/d 400

Sales (I/S)
$ $
Bal c/d 750 (6) Bank 750
750 750
Bal b/d 750

Tax & Insurance (I/S)


$ $
(7) Bank 600 Bal c/d 600
600 600

Bal b/d 600

Drawings (SOFP)
$ $
(9) Bank 300
Bal c/d 300
300 300
Bal b/d 300
(c) Trial balance
Debit Credit
$ $
Bank 8,670
Capital 20,000
Van 10,500
Repairs and Maintenance 200
Purchases 830
Trade payables 750
Petrol 400
Sales 750
Tax & Insurance 600
Drawings 300
21,500 21,500

129
6: ANSWERS

(d) Income Statement


$ $
Purchases 830 Sales 750
Gross profit c/d 420 Closing inventories 500
1,250 1,250
Petrol 400 Gross profit b/d 420
Repairs & Maintenance 200
Tax & Insurance 600
Net loss c/d 780
1,200 1,200

Net loss b/d 780

Purchases (I/S)
$ $
(4) Trade payables 750
(8) Bank 80 Balance c/d 830
830 830
Balance b/d 830 Income statement 830

Petrol (I/S)
$ $
(5) Bank 400 Balance c/d 400
400 400

Balance b/d 400 Income statement 400

Repairs & Maintenance (I/S)


$ $
(3) Bank 200 Balance c/d 200
200 200

Balance b/d 200 Income statement 200

Tax & Insurance (I/S)


$ $
(7) Bank 600 Balance c/d 600
600 600

Balance b/d 600 Income statement 600

Sales (I/S)
$ $
Balance c/d 750 (6) Bank 750
750 750

Income statement 750 Balance b/d 750

130
6: ANSWERS

Inventories (SOFP)
$ $
Closing inventories (I/S) 500

(e) Brian
Income statement for the two months ended 28 February
$ $
Sales 750
Less cost of sales:
Purchases 830
Less: closing inventories (500)
330
Gross profit 420
Less expenses:
Petrol 400
Repairs & Maintenance 200
Tax & Insurance 600
((1,200)
Net loss for the period (780)

Brian
Statement of financial position as at 28 February

Non current assets $


Motor vehicles 10,500

Current assets 500


Inventories 500
Bank 8,670
Total assets 19,670

Proprietor’s interest $ $
Capital introduced on 1 January 20,000
Loss for the period (780)
Less: drawings (300)
(1,080)
Balance at 28 February 18,920

Current liabilities
Trade payables 750
Total capital and liabilities 19,670
(f)
Drawings (SOFP)
$ $
(9) Bank 300
Bal c/d 300
300 300

Bal b/d 300 Capital a/c 300

131
6: ANSWERS

Income statement
$ $
Purchases 830 Sales 750
Gross profit c/d 420 Closing inventories 500
1,250 1,250
Petrol 400 Gross profit b/d 420
Repairs & Maintenance 200
Tax & Insurance 600
Net loss c/d 780
1,200 1,200
Net loss b/d 780 Capital a/c 780
Capital account (SOFP)
$ $
Bal c/d 20,000 (1) Bank 20,000
20,000 20,000

Drawings 300 Bal b/d 20,000


Net loss 780
Bal c/d 18,920
20,000 20,000
Bal b/d 18,920
6.6 Dealers
(a) Net assets = proprietor’s interest
∴ Net assets at 1 January are $18,500
(b) Net assets = assets – liabilities
At 31 January the assets total:
$ $
Plant and equipment 10,000
Motor vehicles 5,000
Trade receivables 2,000
Inventories 4,500
Balance in the bank 3,500
Cash in the till 250
25,250
At 31 January the liabilities total:
Trade payables 3,000
Accrued expenses 250 3,250
∴ Net assets at 31 January 22,000

(c) Profit = Increase in net Drawings between Additional capital


assets between + the same two – paid in between
two points in points in time the same two points
time in time
∴ Profit for the month of January =
(22,000 – 18,500) + (350 + 1,250) – 2,000 = $3,100

132
6: ANSWERS

(d) Accounting equation at 31 January


ASSETS = CAPITAL + PROFIT – DRAWINGS + PAYABLES
25,250 = 20,500 + 3,100 – 1,600 + 3,250
$ $
Plant & equipment 10,000 Capital at 1 January 18,500
Motor vehicles 5,000 Additional capital 2,000
Inventories 4,500 Profit 3,100
Trade receivables 2,000 23,600
Balance in bank 3,500 Less: drawings 1,600
Cash in the till 250 22,000
Trade payables 3,000
Accrued expenses 250
25,250 25,250

133
6: ANSWERS

END OF CHAPTER

134
Sales tax 7
Syllabus Guide Detailed Outcomes
Having studied this chapter you will be able to:
• Understand the general principles of the operation of a sales tax.
• Calculate sales tax on transactions and record the consequent accounting entries.

Exam Context
This topic is likely to be tested in two main ways. You may be asked to identify the correct journal entry to post sales and
purchases transactions including sales tax. You may also be required to consider how sales tax affects the calculation of
amounts to be capitalised for non-current assets and the amount for trade receivables where discounts are offered.

Qualification Context
Financial Accounting introduces accounting for sales tax. More detailed rules and calculations relating to this area are
covered in the Fundamentals level paper, Taxation (F6).

135
7: SALES TAX

Overview

Output tax Input tax

Accounting treatment

Sales tax

Irrecoverable sales tax Discounts

136
7: SALES TAX

1 Introduction
1.1 This chapter is designed to enable you to prepare basic accounting entries for sales tax,
known in many countries as Value Added Tax (VAT).

Sales tax
1.2 A business' sales and purchases are often subject to sales tax. This is an indirect tax, as it
is not levied directly on the individual like personal income tax. Sales tax is collected by
traders who charge it on the goods they sell to the customer.
A business charges sales tax on its sales (output tax) and suffers sales tax on its
purchases (input tax). Typically, a business which is registered for sales tax only needs to
make a payment to the tax authorities of the net amount of sales tax (i.e. sales tax owed on
outputs less sales tax suffered on inputs).

Purchases Sales
Goods into factory Goods out of factory
(input tax) (output tax)

1.3 A registered business shows:


(a) items of income and expenditure net of sales tax;
(b) trade receivables and trade payables gross of sales tax.

1.4 Illustration (all figures include sales tax at 15%).


Purchase raw materials $115.00
Sell finished product $287.50
Required
Calculate the amounts due to or from the sales tax authority.
$
Input tax

Output tax

The rate of sales tax will always be provided in an exam question.

137
7: SALES TAX

2 Accounting treatment

Lecture example 1
A business buys goods for $1,000 plus 15% sales tax. They then sell those goods for $1,500 +
15% sales tax.
The purchases will cost ($1,000 × 1.15) = $1,150
The sales will raise ($1,500 × 1.15) = $1,725
The sales tax payable to tax authorities will be:
$
Payable on outputs (sales) (15% × $1,500) 225.00
Reclaimable on inputs (purchases) (15% × $1,000) (150.00)
Net sales tax to tax authorities 75.00
As the business is purely collecting the sales tax for the tax authorities, and is able to set off its
sales tax suffered it does not include sales tax as either an expense or income in the income
statement. The sales tax is accounted for when the transaction occurs.
Required
(a) Post the double entry to the ledger account below.
$ $
Dr Purchases 1,000
Dr Sales tax control account 150
Cr Trade payables 1,150

Solution
(a)
Purchases (I/S) Trade payables (SOFP)

Sales tax control account (SOFP)

138
7: SALES TAX

Points to note
Purchases – NET
Trade payables – GROSS
(b) Post the double entry to the ledger account below.
$ $
Dr Trade receivables 1,725
Cr Sales 1,500
Cr Sales tax control account 225

Solution
Sales (I/S) Trade receivables (SOFP)

Sales tax control account (SOFP)


$ $
Balance b/d 150

Points to note
Sales – NET
Trade receivables – GROSS

3 Irrecoverable sales tax


3.1 In some tax regimes, sales tax on certain inputs is never recoverable. For example, sales
tax on business entertaining or on cars may not be recoverable. In this case the tax is a
genuine expense of the business and is charged to the income statement or included in the
cost of an asset to be depreciated. For example, the double entry for buying a car where
the sales tax is irrecoverable would be:

Dr Motor vehicles account Cost + sales tax


Cr Cash account Cost + sales tax

139
7: SALES TAX

4 Sales tax and discounts


4.1 Many businesses offer discounts to their customers. There are two types:
• trade discounts
• settlement discounts

4.2 Sales tax is calculated on the amount after all discounts.

4.3 The calculation and accounting treatment of discounts is covered in Chapter 14.

140
7: SALES TAX

Additional Notes

141
7: SALES TAX

5 Rates of sales tax


5.1 Most goods and services are subject to sales tax at the standard rate. This may be 17½%
as it is in the UK or any other rate provided by the question.

5.2 Some goods are known as zero rated or exempt goods.

5.3 If goods are zero rated, such as books and newspapers, then sales tax is still charged on
them but it is charged at a rate of 0%. A business making zero rated supplies can still claim
back any input sales tax suffered on its purchases (at the relevant rate).

5.4 Exempt goods, such as banking, however are not subject to sales tax and a business
making only exempt supplies cannot be registered for sales tax and cannot therefore
reclaim input sales tax suffered on its purchases.

6 Chapter summary
Quick Quiz 6.1
Section Topic Summary

1 Introduction A business acts as a collecting agent for the tax


authorities and charges sales tax (output tax) on its
sales and reclaims sales tax (input tax) on its
purchases.
2 Accounting treatment Sales and purchases are recorded at the net amount.
Sales tax may be charged at various rates, however
the rate of sales tax will always be provided in an
exam question.
3 Irrecoverable sales tax Sales tax may not be recoverable on certain purchases.
Where this is the case the question will state that the
sales tax is not recoverable and the cost recorded will
be the gross amount.
4 Sales tax and The effect of discounts on sales tax is covered in
discounts Chapter 14.
5 Rates of sales tax Zero rated supplies have sales tax charged on them at
0% whereas exempt supplies are not subject to sales
tax.

142
7: SALES TAX

7 Double Entry Summary for Chapter 7


7.1 Recording a credit purchase with sales tax:

Dr Purchases net
Dr Sales tax control account tax
Cr Trade payables gross

7.2 Recording a credit sale with sales tax:

Dr Trade receivables gross


Cr Sales net
Cr Sales tax control account tax

143
7: SALES TAX

Chapter Summary

Output tax Input tax

• A business charges sales tax on its sales • A business will recover sales tax on its
• This is paid over to the tax authority once any purchases
recoverable input tax is deducted

Accounting treatment

• Sales and purchases are recorded net of sales tax


• Trade receivables and trade payables are recorded gross of sales tax

Sales tax

Irrecoverable sales tax Discounts

• Sales tax cannot be recovered on certain items • Covered in Chapter 14


such as some non-current assets
• The question will always state if this is the case

144
Chapter 7: Questions

145
7: QUESTIONS

7.1 Elmo is a trader registered for sales tax. All his sales and purchases carry sales tax at a rate of 15%. A
customer has just returned goods sold for $230 plus sales tax, the double entry for this transaction is
A Debit payables $264.50, Credit sales tax $34.50, Credit sales $230
B Debit sales $264.50, Credit trade receivables $264.50
C Debit sales $230, Debit sales tax $34.50, Credit trade receivables $264.50
D Debit sales $230, Debit irrecoverable sales tax $34.50, Credit trade receivables $264.50
(2 marks)

7.2 During 20X1 Fergus buys two vans and a car each costing $10,000 plus sales tax at 15%. The car will be
used 70% for business use and 30% personal use. He depreciates vehicles on a straight line basis, vans
over five years and cars over six years. What is his depreciation expense to the nearest $ for the year?
In the tax regime in which Fergus operates sales tax is only recoverable on items used wholly for
business purposes.
A $5,917
B $6,517
C $6,100
D $5,666 (2 marks)

146
Chapter 7: Answers

147
7: ANSWERS

7.1 C

7.2 A Sales tax on the car is not recoverable as it is not wholly used for business purposes. Sales tax is
however recoverable on the vans.
$
Vans (2 × $10,000) ÷ 5 = 4,000
Car ($10,000 × 115%) ÷ 6 = 1,917
5,917

END OF CHAPTER

148
Inventory 8
Syllabus Guide Detailed Outcomes
Having studied this chapter you will be able to:
• Recognise the need for adjustments for inventory in preparing financial statements.
• Record opening and closing inventory.
• Identify the alternative methods of valuing inventory.
• Understand and apply the IASB requirements for valuing inventories.
• Recognise which costs should be included in valuing inventories.
• Calculate the value of closing inventory using 'first in, first out' and 'average cost'.
• Understand the use of continuous and period end inventory records.
• Understand the impact of accounting concepts on the valuation of inventory.
• Identify the impact of inventory valuation methods on profit and on assets.

Exam Context
Accounting for inventories and inventory valuation is a basic principle that affects any business. Examination questions
are likely to test your understanding of the terms cost and net realisable value. You should also expect calculations on
this area and be able to make adjustments for both opening and closing inventory.

Qualification Context
The Fundamentals level paper Management Accounting (F2) explores inventories in more detail. There you will look at
the classification of costs (for example, production versus non production and fixed versus variable) and you will also
cover detailed calculations on overhead absorption.

149
8: INVENTORY

Overview

Accounting adjustments

Inventory

Valuation Effects on profit

Cost Net realisable value

Methods of estimating cost

FIFO AVCO

150
8: INVENTORY

1 Introduction
1.1 For some businesses, for example manufacturing entities, inventory can be a significant
figure.

1.2 It impacts the financial statement in two ways:


(a) Statement of financial position: a potentially large balance within Current Assets
(b) Income statement: opening and closing inventory have a direct
impact on cost of sales and therefore profits

1.3 Businesses must therefore ensure that their financial statements account for inventory
accurately in terms of:
(a) the accounting adjustment
(b) its valuation

2 Accounting adjustment
2.1 Inventory is generally accounted for as a year end adjustment via a journal entry.

2.2 Opening inventory


The trial balance produced by the entity at the end of the year will show an inventory figure.
This amount generally relates to the opening inventory – i.e. the goods held by the business
at the beginning of the year.
Such goods will have been sold during the year. They are no longer an asset of the entity
but will form part of the costs that should be matched against sales revenue when
determining profit.
The accounting entry is:
Dr Cost of sales (I/S)
Cr Inventories (SOFP)

2.3 Closing inventory


The goods held by the business at the end of the year must be included as an asset in the
statement of financial position and within cost of sales in the income statement.
The accounting entry is:
Dr Inventories (SOFP)
Cr Cost of sales (I/S)

151
8: INVENTORY

2.4 The inventories figure comprises two elements:

QUANTITY × VALUATION

Quantity: normally ascertained by inventory count at end of accounting period or


by continuous inventory records.
Valuation: much more subjective, so guidance is provided in IAS 2.

2.5 Inventory overview


Inventory = Quantity x Valuation

Continuous Inventory Lower of


Inventory records count Cost and NRV

All costs to get $


item to current Selling price X
location in Less: completion costs (X)
current condition Less: selling costs (X)
X

Actual cost Deemed cost

FIFO Average
Cost

3 Valuation
3.1 The basic rule per IAS 2: Inventories is:
'Inventories should be measured at the lower of cost and net realisable value.'

3.2 This is another example of prudence in presenting financial information.


(a) If inventory is expected to be sold at a profit:
(i) value at cost
(ii) do not anticipate profit.
(b) If inventory is expected to be sold at a loss:
(i) value at net realisable value
(ii) do provide for the future loss.

152
8: INVENTORY

4 Cost
4.1 The cost of an item of inventory includes:
For example:
• purchase price
• import duties
Cost of purchase
But not:
• sales tax
• trade discounts

Relating to productions:
Costs of conversion • direct labour
• direct/variable overheads
Section 5.4
• an allocation of fixed
overheads (based on
normal level of activity)
Other costs incurred in bringing
the inventories to their present For example:
location and condition • carriage inwards

Lecture example 1 Exam standard question worth 2 marks

According to IAS 2: Inventories, which of the following should not be included in determining the
cost of the inventories of an entity?
(1) Labour costs
(2) Transport costs to deliver goods to customers
(3) Administrative overheads
(4) Depreciation on factory machine

A All four items


B 1 only
C 2 and 3 only
D 2, 3, and 4 only

Solution

153
8: INVENTORY

5 Net realisable value (NRV)


5.1 The net realisable value of an item is essentially its net selling proceeds after all costs have
been deducted.

5.2 It is calculated as:


$
Estimated selling price X
Less: estimated costs of completion (X)
Less: estimated selling and distribution costs (X)
X

Lecture example 2 Preparation question

Jessie is trying to value her inventory. She has the following information available:
$
Selling price 35
Costs incurred to date 20
Cost of work to complete item 12
Selling costs per item 1
Required
What is the net realisable value of Jessie's inventory? $

Workings

No netting off
5.3 The IAS 2 rule 'lower of cost and net realisable value' should be applied as far as
possible on an item by item (or line by line) basis.

154
8: INVENTORY

Illustration
5.4 Suppose an entity has four items of inventories on hand at the year end. Their costs and
NRVs are as follows:
Inventory item Lower of cost
Cost NRV and NRV
$ $ $
1 27 32 27
2 14 8 8
3 43 55 43
4 29 40 29
113 135 107
It would be incorrect to compare total cost of $113 with total NRV of $135 and state
inventories as $113.
A loss on item 2 of $6 can be foreseen and should therefore be recognised.
The comparison should be made for each item of inventory and thus a value of $107 would
be attributed to inventories.
This would be accounted for by the journal entry:
$ $
Dr Inventories (SOFP) 107
Cr Cost of sales (I/S) 107

6 Theoretical methods of estimating cost


Issue
6.1 If various batches of inventories have been purchased at different times during the year and
at different prices, it may be impossible to determine precisely which items are still held at
the year end and therefore what the actual purchase cost of the goods was. IAS 2 therefore
Section 4.3
allows an entity to approximate the cost of its inventories. There are two methods
examinable at Paper F3:
• First in, first out (FIFO)
• Average cost
6.2 (a) FIFO
Under FIFO it is assumed that:
(i) first goods purchased/produced will be the first to be sold
(ii) remaining inventories are the most recent purchases/production.
(b) Average Cost (AVCO)
There are two average costs available:
(i) Simple average cost
The cost of all purchases/production during the year is divided by the total
number of units purchased

155
8: INVENTORY

(ii) Weighted average cost


The weighted average of the cost of similar items is recalculated each time a
new item is purchased/produced during the period (IAS 2 requires the weighted
average to be used)

Lecture example 3 Preparation question

On 1 January 20X7 a company held 200 units of finished goods valued at $10 each. During
January the following transactions took place.

Date Units purchased Cost per unit


10 January 300 $10.85
20 January 350 $11.50
25 January 250 $13.00
Sales during January were as follows:

Date Units sold Sales price per unit


14 January 280 $18.00
21 January 400 $18.00
28 January 80 $18.00
Required
Determine the valuation of closing inventories and cost of sales using:
(a) FIFO
(b) Weighted average cost

Solution
(a) Closing inventories (FIFO)
Purchases
1.1.X7 10.1.X7 20.1.X7 25.1.X7

Sales

Cost of sales (FIFO)

156
8: INVENTORY

(b) Closing inventories and cost of sales (AVCO)


Average Total Cost of
Units Cost Unit Cost Cost Sales
$ $ $ $
1.1.X7 b/f

10.1.X7 Purchase

14.1.X7 Sale

20.1.X7 Purchase

21.1.X7 Sale

25.1.X7 Purchase

28.1.X7 Sale
Workings

Advantages and disadvantages


6.3 FIFO: more “realistic” value on statement of financial position.
Average cost: can be complex as weighted average is required by IAS 2.

157
8: INVENTORY

7 Valuation effects on profit


7.1 All of the inventory valuation methods affect profits. Using the FIFO, and average cost
examples above, this can be illustrated in an income statement.
FIFO Weighted average
$ $ $ $
Sales (760 × $18) 13,680 13,680
Cost of sales
Opening inventories 2,000 2,000
Purchases 10,530 10,530
Closing inventories (4,285) (4,160)
8,245 8,370
Gross profit 5,435 5,310

7.2 The only figure that varies is the closing inventories, the result being quite different profit
figures.
This re-emphasises the significance of inventory valuation in the preparation of financial
statements.

Effects in times of changing prices


7.3 In the above example, the purchase price of inventories was rising during the period. Notice
that when prices are rising:
FIFO will tend to give higher inventory values and higher profits.

158
8: INVENTORY

8 Chapter summary
Section Topic Summary

1 Introduction Inventories can be a significant figure in an entity’s


accounts and will impact both the profit figure and the
net asset position. It is important therefore that it is
recorded correctly.
2 Accounting adjustment As seen in chapter 6 the income statement matches the
sales revenue earned in a period with the cost of sales
incurred to generate that revenue. There are therefore
two inventory adjustments: the opening inventory
adjustment and the closing inventory adjustment.
3 Valuation Inventories should be valued at the lower of cost and
net realisable value.
4 Cost The cost of inventory includes the cost of purchase,
costs of conversion and any other costs necessary
to bring the inventory to its present location and
condition.
5 Net realisable value Net realisable value is the estimated selling price less
(NRV) the costs to completion and any selling and
distribution costs.
6 Theoretical methods of Methods available to estimate the cost of inventories
estimating cost are first in, first out (FIFO) and average cost. Under
FIFO the inventories held at the year end are the most
recent purchases but under average cost the cost of all
inventories purchased during the year is weighted to
produce an average figure.
7 Valuation effects on In times of rising prices, using FIFO will mean the
profit financial statements show higher inventory values and
higher profits.

159
8: INVENTORY
• Opening inventory:
Dr Cost of sales (I/S)
Chapter Summary Cr Inventories (SOFP)
• Closing inventory:
Dr Inventories (SOFP)
Cr Cost of sales (I/S)

Accounting adjustments

Inventory

Valuation Effects on profit

'Inventories should be measured at the lower of cost • Closing inventory


and net realisable value' valuation will differ
• This is on a line by line basis depending on cost
method used
• This has a direct
impact on cost of
sales and
Net realisable value therefore on gross
Cost
profit
• When prices are
• Calculation: rising FIFO will
• Cost includes: give higher
Selling price X
- costs of purchase inventory values
Less: completion costs (X)
- costs of conversion and therefore
Less: selling costs (X)
- other costs higher profits
X

Methods of estimating cost

FIFO AVCO

'First in, first out' 'Average cost'


• The first goods • Simple average calculation:
purchased will be Total purchases cost ÷ total number of units purchased
the first sold • Weighted average (required by IAS 2):
• Year-end A new average is recalculated each time inventories are
inventories relate purchased
to the most recent
purchases

160
Chapter 8: Questions

161
8: QUESTIONS

8.1 An item of inventory could be sold for $100 after it has been modified at a cost of $21. The company
incurs selling and distribution costs of 5% of selling price on each article sold. The cost is $45 per unit
excluding carriage inwards of $2 and production overheads of $17 per unit.
Following the rules in IAS 2 at what valuation should this item be included in the inventories of the
company? $ (2 marks)

8.2 Harrow Co sells one line of inventory. At the year end it has 200 units in inventory which originally cost
$10 per unit and had incurred delivery costs of $120 in total. They expect these goods to sell for $13 per
unit. Harrow Co incurs selling costs amounting to 10% of the selling price on all its sales.
In the statement of financial position these items should be valued at:
A $2,000
B $2,080
C $2,120
D $2,600 (2 marks)

8.3 Lamp makes the following purchases in the year.


Units $/unit Total ($)
(i) 21.01.X9 100 12.00 1,200
(ii) 30.04.X9 300 12.50 3,750
(iii) 31.07.X9 40 12.80 512
(iv) 01.09.X9 60 13.00 780
(v) 11.11.X9 80 13.50 1,080
At the year end 200 units are in inventory but eight are damaged and are only worth $10 per unit. These
are identified as having been part of the 11.11.X9 delivery. Lamp operates a FIFO system for valuing
inventories.
The figure for inventories at 31 December 20X9 is:
A $2,524
B $2,594
C $2,622
D $2,700 (2 marks)
8.4 Inventories
At the year end, Biggs Co holds the following inventories:
(1) 10 units of L in a completed state; each unit cost $160 to make and has a selling price of $200.
(2) 45 units of M in a partly completed state. Costs to date have amounted to $240 per unit and
completion costs will amount to $90 per unit. Selling price per unit is $360.
(3) 60 units of N purchased for $40 each. These sell at $56 each and would now cost $48 each if
additional units were bought.
(4) 50 units of O costing $10 each. These cannot be sold unless they are modified at a cost of $2 per
unit. After that, the selling price will be $8.
The company’s selling costs are 25% of the selling price.
Required
Calculate the value of inventories that would be shown on the statement of financial position at the end of
the year.

162
8: QUESTIONS

8.5 T Bag
T Bag commenced business as a tea importer on 1 January 20X5. His purchases and sales during his
first six months of trading are set out below:
Purchases Sales
Tonnes Price per tonne Total price Tonnes Proceeds
$ $ $
1 January 30 700 21,000
15 February 20 750 15,000
27 February 40 36,000
31 March 40 820 32,800
16 April 25 880 22,000
30 April 35 35,000
30 May 35 900 31,500
8 June 10 1,050 10,500
28 June 70 77,000

132,800 148,000
Required
Calculate the value of closing inventories and produce a trading account for the 6 months ended 30 June
20X5 assuming:
(a) Inventories are valued on a FIFO basis
(b) Inventories are valued on a weighted average basis

163
8: QUESTIONS

164
Chapter 8: Answers

165
8: ANSWERS

8.1 $64 NRV = 100 – 21 – (5% × 100) = $74


Cost = 45 + 2 + 17 = $64
Lower of cost and NRV = $64

8.2 C $
200 @ $10 = 2,000
Delivery costs 120
2,120
Cost/ unit = $10.60
Net realisable value per unit = $13 × 90% = $11.70
⇒ valued at cost

8.3 B $
Inventories = 8 @ $10 = 80
72 @ $13.50 = 972
60 @ $13 = 780
40 @ $12.80 = 512
20 @ $12.50 = 250
200 2,594
8.4 Inventories
The inventories total on the statement of financial positionwould be:
$12,200 ($1,500 + $8,100 + $2,400 + $200).
Workings
(1) 1 unit of L would be valued at:
$
Selling price 200
Less selling costs (25%) 50
∴ NRV 150
Cost 160
NRV is lower and so 10 units of L are valued at $1,500
(2) 1 unit of M would be valued at:
$ $
Selling price 360
Less: Selling costs (25%) 90
Costs to completion 90 180
∴ NRV 180
Cost 240
NRV is lower and so 45 units of M are valued at $8,100
(3) 1 unit of N would be valued at:
$
Selling price 56
Less selling costs (25%) 14
∴ NRV 42
Cost 40
Cost is lower and 60 units of N are valued at $2,400
Replacement cost is irrelevant.

166
8: ANSWERS

(4) 1 unit of O would be valued at:


$ $
Selling price 8
Less: Selling costs (25%) 2
Costs of modification 2 4
∴ NRV 4
Cost 10
NRV is lower and so 50 units of O are valued at $200
8.5 T Bag
Trading account for the 6 months ended 30 June 20X5
FIFO Weighted
average
$ $
Sales 148,000 148,000

Cost of sales
Purchases 132,800 132,800
Closing inventories (W) (15,000) (13,245)
(117,800) (119,555)
Gross profit 30,200 28,445
Workings
(W1) FIFO method
1 Jan 15 Feb 31 Mar 16 Apr 30 May 8 June
Purchases in tonnes 30 20 40 25 35 10
Sales in tonnes:
27 Feb (30) (10)
30 Apr (10) (25)
28 June (15) (25) (30)
Inventories at 30 June 20X5 – – – – 5 10
Cost per tonne $900 $1,050

∴ Valuation $4,500 $10,500

Total valuation $4,500 + $10,500 = $15,000

(W2) Weighted average method


Average Total Cost of Sales
Tonnes Cost Unit Cost Cost
$ $ $ $
1 Jan 30 700 21,000
15 Feb 20 750 15,000
50 720 36,000
27 Feb (40) (28,800) 28,800
10 7,200
31 March 40 820 32,800
16 April 25 880 22,000
75 827 62,000
30 April (35) (28,945) 28,945
40 33,055
30 May 35 900 31,500
8 June 10 1,050 10,500
85 883 75,055
28 June (70) (61,810) 61,810
15 13,245 119,555

167
8: ANSWERS

END OF CHAPTER

168
Tangible non-current
assets 9
Syllabus Guide Detailed Outcomes
Having studied this chapter you will be able to:
• Define non-current assets and recognise the difference between current and non-current assets.
• Explain the difference between capital and revenue items and classify expenditure accordingly.
• Prepare ledger entries to record the acquisition, disposal, depreciation and accumulated depreciation of non-
current assets.
• Calculate and record profits or losses on disposal of non-current assets in the income statement.
• Record the revaluation of a non-current asset and calculate its subsequent depreciation and profit or loss on
disposal.
• Illustrate how non-current asset balances and movements are disclosed in company financial statements.
• Explain the purpose and function of an asset register.
• Understand and explain the purpose of depreciation.
• Calculate the charge for depreciation using the straight line and reducing methods, identifying when each is
appropriate.
• Calculate the adjustments to depreciation necessary if changes are made in the estimated useful life and/or
residual value of a non-current asset.
• Record depreciation in the income statement and statement of financial position.

Exam Context
Tangible non-current assets and depreciation are an important part of the F3 syllabus and you should expect several
questions on this area. Questions are likely to focus on areas such as calculating depreciation and asset values (both on
assets held at historic cost and revalued amounts), profits or losses on disposal of assets and the components that can
be included in the cost of a non-current asset.

Qualification Context
The knowledge covered in this chapter is developed in the Fundamentals level paper Financial Reporting (F7) where
you will deal with more complex issues such as impairments of non-current assets and leasing.

169
9: TANGIBLE NON-CURRENT ASSETS

Overview

Capital versus revenue


Cost
expenditure

Tangible non-current
assets

Revaluations Depreciation Disposals

Straight line Reducing balance


method depreciation

170
9: TANGIBLE NON-CURRENT ASSETS

1 Introduction
1.1 The purchase of a non-current asset is often a significant cost to a business which will have
a large impact on its financial statements.

1.2 It is important therefore that this expenditure is accounted for appropriately.

2 Non-current assets
Definition
2.1 Non-current assets are assets which are intended to be used by the business on a
continuing basis and include both tangible and intangible assets.
Intangible non-current assets are covered in Chapter 10.

2.2 The accounting treatment of tangible non-current assets is covered by IAS 16: Property,
Plant and equipment.
Tangible non-current assets are defined as those which:
(a) are held for use in the production or supply of goods or services or for administrative
purposes; and
(b) are expected to be used during more than one period.

Lecture example 1 Idea generation

Required
What examples of tangible non-current assets can you identify?

Solution
(a)

(b)

(c)

(d)

171
9: TANGIBLE NON-CURRENT ASSETS

Capital versus revenue expenditure


2.3 (a) Capital expenditure: results in the acquisition, replacement or improvement
of non-current assets.
Section 1.3 (b) Revenue expenditure: – for the trade of the business, or
– to repair, maintain and service non-current assets.

2.4 Capital expenditure results in the appearance of a non-current asset in the statement of
financial position of the business.
Revenue expenditure results in an expense in the income statement.

Cost
2.5 Tangible non-current assets should initially be recorded at cost.
Cost includes:
• Purchase price: excluding sales tax and trade discounts but including import
duties
• Directly attributable costs to bring the asset to its intended location and ready to
use. These include:
(a) Initial delivery and handling costs
(b) Installation and assembly costs
(c) Costs of testing whether the asset is working properly
(d) Professional fees
The following costs may not be included:
(a) The cost of maintenance contracts
(b) Administration and general overhead costs
(c) Staff training costs

2.6 The asset can then be kept at cost and depreciated or the entity may choose to revalue its
tangible non-current assets.

Lecture example 2 Exam standard worth 2 marks

On 10 December 20X7 an entity bought a machine.


The breakdown on the invoice showed:
$
Cost of machine 20,000
Delivery costs 200
One-year maintenance contract 900
21,100
Further installation costs of $500 were also incurred.

172
9: TANGIBLE NON-CURRENT ASSETS

Required
At what amount should the machine be capitalised in the entity's records?
A $20,000
B $20,700
C $20,200
D $21,600

Solution

3 Depreciation
3.1 Tangible non-current assets are used in the business to generate the income shown in the
income statement.
Assets will eventually be worn out (used up) and so there is a cost of generating income.
This cost should be shown in the income statement to 'match' against the income.
This is called depreciation.

3.2 Depreciation results in the non-current asset being systematically charged to the income
statement over several accounting periods in recognition of the fact that the asset will
contribute to the income-generating activities of each of these periods.
A formal definition is given by the accounting standard, IAS 16:

"…the systematic allocation of the depreciable amount of an asset over its useful life."

'Depreciable amount' = cost/revalued amount – residual value


'Residual value' = the amount the asset is expected to be sold for at the end of its
useful life (scrap value).

3.3 Land normally has an unlimited useful life and is therefore not depreciated. Buildings have
a limited life and, therefore, are depreciable assets.

173
9: TANGIBLE NON-CURRENT ASSETS

4 Methods of depreciation
4.1 There are two main methods for calculating depreciation:
(a) Straight line method
(b) Reducing balance method

5 Straight line method


5.1 The depreciation charge is the same every year.

Formula
cost − residual value
5.2 Depreciation = or (Cost – Residual value) × %
useful life (years)
where:
Residual value = expected proceeds/scrap value at the end of the asset's useful life.
Useful life = the number of years the business expects to make use of the asset.

5.3 This method is suitable for assets which are used up evenly over their useful life.

Lecture example 3 Preparation question

A business buys a machine for $2,500. It is expected to have a useful life of three years after
which time it will have a scrap value of $250.
Required
(a) Calculate the annual depreciation charge.
(b) Calculate the cost, accumulated depreciation and net book value (NBV) for each year of the
asset's life. Note: NBV = cost – accumulated depreciation to date.

Solution
(a)

(b)
Year Cost Accumulated NBV
depreciation
$ $ $
1

174
9: TANGIBLE NON-CURRENT ASSETS

6 Reducing balance depreciation


6.1 This method is suitable for those assets which generate more revenue in earlier years than
in later years; for example a machine which may become progressively less efficient as it
gets older.
Under this method the depreciation charge will be higher in the earlier years and reduce
over time.

Formula
6.2 Depreciation = Depreciation rate (%) × Net Book Value (NBV)
where: net book value (NBV) = cost – accumulated depreciation to date
Note: This method does not take account of any residual value, since the NBV under this
method will never reach zero. The depreciation rate percentage will be provided in
the question.

Lecture example 4 Preparation question

A business buys a machine costing $6,000. The depreciation rate is 40% on a reducing balance
basis.
Required
Calculate depreciation expense, accumulated depreciation and net book value of the asset for the
first three years.

Solution
Year NBV b/d Depreciation Depreciation Accumulated NBV c/d
rate expense depreciation
$ $ $ $
1

175
9: TANGIBLE NON-CURRENT ASSETS

7 Accounting for depreciation


Dual effect
7.1 Depreciation has a dual effect which needs to be accounted for:
(a) It reduces the value of the asset in the statement of financial position.
(b) It is an expense in the income statement.

7.2 The asset remains at its original cost in the asset account.
Two accounts are set up to record depreciation:
Dr Depreciation expense
Cr Accumulated depreciation

Accumulated depreciation account


7.3 (a) Used to provide for the reduction in value of the asset.
(b) Reduces original cost of the asset on the statement of financial position. (The balance
on the account is offset against the cost account for the corresponding asset.)
(c) Separate account kept for each class of asset (eg motor vehicles, buildings, plant and
machinery).

Lecture example 5 Preparation question

Required
Using the information in Lecture example 3, show:
(a) The journal entry which would have been written at the end of the first year.
(b) The treatment of depreciation for all years in the relevant ledger accounts.
(c) The relevant income statement and statement of financial position extracts for each year.

Solution
(a) Journal entry
Debit Credit
$ $

176
9: TANGIBLE NON-CURRENT ASSETS

(b) Machine (SOFP)

Depreciation expense (I/S)

Accumulated depreciation (SOFP)

(c) Income statement (extracts)


Year 1 Year 2 Year 3
$ $ $

Expenses

177
9: TANGIBLE NON-CURRENT ASSETS

Statement of financial position (extracts)

Cost Accumulated Net book


depreciation value
$ $ $

(Year 1)

(Year 2)

(Year 3)

8 Disposal of non-current assets


Profit or loss on disposal
8.1 When a non-current asset is disposed, of its net book value needs to be removed from the
statement of financial position.
The sales proceeds received are unlikely to be exactly the same as the asset's net book
value and so a profit or loss on disposal will arise.
If:
Sales proceeds > NBV ⇒ profit on disposal
Sales proceeds < NBV ⇒ loss on disposal
This is not a 'true' profit or loss, but rather a book adjustment to reflect the fact that the
depreciation charged over the asset's life wasn't completely accurate.

Accounting treatment
8.2 Everything to do with the disposal is transferred to a Disposal Account.
Steps:
(1) Remove the cost of the asset:
Dr Disposal account
Cr Non-current asset
(2) Remove the accumulated depreciation charged to date:
Dr Accumulated depreciation
Cr Disposal account
Note: Steps (1) and (2) have effectively transferred the NBV of the asset to the disposal
account.

178
9: TANGIBLE NON-CURRENT ASSETS

(3) Account for the sales proceeds:


Dr Cash
Cr Disposal account
(4) Balance off disposal account to find the profit or loss on disposal.
A gain on disposal is shown in the income statement as sundry income, a loss as an
expense.

Lecture example 6 Preparation question

The machine costing $6,000 in Lecture example 4 is sold in year 3 for $3,000. No depreciation is
charged in the year of disposal.
Required
(a) Calculate the profit or loss on disposal of the machine.
(b) Complete the ledger accounts to show how the disposal would be accounted for.

Solution
(a)

(b)
Machine (SOFP)
$ $
Bal b/d 6,000

179
9: TANGIBLE NON-CURRENT ASSETS

Accumulated depreciation (SOFP)


$ $
Bal b/d 3,840

Disposal account (I/S)


$ $

Part exchange allowance


8.3 Instead of receiving sales proceeds as cash, a part exchange allowance could be offered
against the cost of a replacement asset:
Dr New asset cost
Cr Disposal account
The part exchange allowance takes the place of proceeds in the disposals account.

Lecture example 7 Preparation question

Assume in Lecture example 6 that instead of cash proceeds of $3,000, there is a part exchange
allowance of $3,000 on a replacement machine costing $10,000.
Required
(a) Calculate the profit or loss on disposal of the machine.
(b) Calculate the amount of cash paid for the new machine.
(c) Complete the ledger accounts to show both the disposal and the acquisition.

180
9: TANGIBLE NON-CURRENT ASSETS

Solution
(a)

(b)

(c)
Old machine (SOFP)
$ $
Bal b/d 6,000

Accumulated depreciation (SOFP)


$ $
Bal b/d 3,840

New machine (SOFP)


$ $

Disposal account (I/S)


$ $

181
9: TANGIBLE NON-CURRENT ASSETS

9 Revaluations
9.1 If an entity owns a property it may notice that its value increases over time.

9.2 IAS 16 requires tangible non-current assets to initially be recorded at cost. The entity can
then either keep the asset at cost (and depreciate it) or choose to revalue it (depreciation is
still required).
This is a choice of accounting policy.

9.3 If an entity chooses a policy of revaluation then all items in the same class of assets must be
revalued.
Examples of classes of assets are:
• land and buildings
• plant and machinery
• motor vehicles

9.4 Revaluations must be carried out sufficiently often so that the assets carrying value is not
materially different from its market value.

Steps and accounting treatment


9.5 (1) Adjust cost account to revalued amount.
(2) Remove accumulated depreciation charged on the asset to date.
(3) Put the balance to the revaluation reserve.
Note: The balance posted to the revaluation reserve will equal the new revalued amount
less the previous net book value.

9.6 The required journal is:


Dr Non-current asset cost
Dr Accumulated depreciation
Cr Revaluation reserve

9.7 Depreciation should now be based on the revalued amount.

182
9: TANGIBLE NON-CURRENT ASSETS

Lecture example 8 Preparation question

A building costing $100,000 on which depreciation of $20,000 has been charged is to be revalued
to $150,000.
Required
(a) Show the double entry to record the revaluation and make the postings to the ledger
accounts.
(b) What would be the depreciation charge for the year if the building has a remaining useful life
of 40 years?

Solution
(a)

Building (SOFP)
$ $

Accumulated depreciation (SOFP)


$ $

183
9: TANGIBLE NON-CURRENT ASSETS

Revaluation reserve (SOFP)


$ $

(b)

184
9: TANGIBLE NON-CURRENT ASSETS

Additional Notes

185
9: TANGIBLE NON-CURRENT ASSETS

10 Depreciation revisited
10.1 Depreciation is charged to allocate the wearing out of an asset (depreciable amount) to the
income statement over its useful life.
There are two main depreciation methods available:
• straight line
• reducing balance
10.2 The useful life of an item of property, plant and equipment should be reviewed at least every
financial year-end and, if expectations are significantly different from previous estimates, the
depreciation charge for current and future periods should be revised.
Section 3.12
This is achieved by writing the net book value off over the asset's revised remaining useful
life.

Lecture example 9 Preparation question

1.1.X1 Asset cost $40,000


Estimated useful life five years
No residual value
1.1.X3 Total useful life revised to four years.
Required
Calculate the depreciation charge, accumulated depreciation and NBV for each year of the asset's
life (year end 31 December).

Solution

Depreciation Accumulated NBV


charge depreciation
$ $ $
20X1
20X2
20X3
20X4

186
9: TANGIBLE NON-CURRENT ASSETS

Review of depreciation method


10.3 The depreciation method should be reviewed at least every financial year-end and, if there
Section 3.7-3.9 has been a significant change in the expected pattern of the asset's use, the method should
be changed.
This is achieved by writing the net book amount off over the remaining useful life, using the
revised method.

Lecture example 10 Preparation question

1.1.X1 Asset cost $40,000


Residual value $1,500
Useful life five years
Depreciation: 25% reducing balance
1.1.X3 Change depreciation method to straight line
Required
Calculate the depreciation charge, accumulated depreciation and NBV for each year of the asset’s
life (year ended 31 December).

Solution

Depreciation Accumulated NBV


charge depreciation
$ $ $
20X1
20X2
20X3
20X4
20X5

187
9: TANGIBLE NON-CURRENT ASSETS

11 Chapter summary
Section Topic Summary

2 Non-current assets Capital expenditure results in a non-current asset


being shown on the statement of financial position.
Revenue expenditure, such as repairs and
maintenance, is shown as an expense in the income
statement.
Tangible non-current assets should initially be recorded
at cost. This includes the purchase price of the item
plus any directly attributable costs to bring the item to
its intended location and ready to use.
3 Depreciation Depreciation is an expense charged in relation to the
asset each year to reflect the using up of the asset.
Land usually has an unlimited useful life and so is not
depreciated.
4 Methods of Depreciation is usually calculated on a straight line or
depreciation reducing balance basis.
5 Straight line method This method is suitable for assets which are used up
evenly during their life time. The depreciation expense
is the same each year.
6 Reducing balance This method is suitable for assets which generate more
depreciation revenue in the earlier years of their life. The
depreciation expense is higher in the initial years.
7 Accounting for Depreciation is recorded by way of a journal entry. The
depreciation expense is recorded as a debit entry and reduces profit.
The credit is made to the accumulated depreciation
account and reduces the carrying value of the asset in
the statement of financial position.
8 Disposal of non-current On disposal of a non-current asset the sales proceeds
assets are compared to the net book value of the asset in
order to calculate the profit or loss on disposal.
Where an asset is given in part exchange for another
asset, the part exchange allowance takes the place of
the sales proceeds.
9 Revaluations An entity may choose to revalue its assets rather than
hold them at cost – this is a choice of accounting
policy. Where an entity revalues, it must revalue all
assets in the same class and the depreciation charge
is based on the revalued amount.

188
9: TANGIBLE NON-CURRENT ASSETS

Section Topic Summary

10 Depreciation revisited If an entity changes the method of depreciation used


from straight line to reducing balance (or vice versa) or
revises the useful life of an asset it should write off the
asset’s net book value using the revised method or
useful life.

12 Double Entry Summary for Chapter 9


12.1 Depreciation adjustment:

Dr Depreciation expense (I/S)


Cr Accumulated depreciation (SOFP)

12.2 Disposal of a non-current asset (four steps):


(1) Remove the cost of the asset:
Dr Disposal account (I/S)
Cr Non-current assets (SOFP)

(2) Remove the accumulated depreciation charged to date:

Dr Accumulated depreciation (SOFP)


Cr Disposal account (I/S)

(3) Account for the sales proceeds:


Dr Cash (SOFP)
Cr Disposal account (I/S)

(4) Balance off the disposal account to determine the profit or loss on disposal.

12.3 Revaluation of a non-current asset:

Dr Non-current asset cost (SOFP)


Dr Accumulated depreciation (SOFP)
Cr Revaluation reserve (SOFP)

189
9: TANGIBLE NON-CURRENT ASSETS
• Cost includes the purchase price plus directly
attributable costs
Chapter Summary • Directly attributable costs include:
- delivery
- installation/ testing
• Capital expenditure: acquisition, replacement or improvement of - professional fees
non-current assets • Directly attributable costs exclude:
• Revenue expenditure: trading expenses or the repair, - maintenance contracts
maintenance and service of non-current assets - administration and general overheads
- staff training

Capital versus revenue


Cost
expenditure

Tangible non-current
assets

Revaluations Depreciation Disposals

'The wearing out of an asset as it • Steps:


• Steps:
generates revenue' (1) Remove the cost of the asset
(1) Adjust cost to
• Accounting adjustment: (2) Remove the accumulated
revalued amount
Dr Depreciation expense depreciation charged to date
(2) Remove the
Cr Accumulated depreciation (3) Account for sales proceeds
accumulated
(4) Balance off disposal account to
depreciation
find the profit or loss on disposal
charged to date
(3) Put the balance to • Profit/ loss on disposal calculation:
the revaluation
Straight line Reducing balance Proceeds X
reserve method depreciation Less: NBV (X)
Profit/(loss) X/(X)
• The balance transferred
to the revaluation reserve • Depreciation charge is the • Depreciation charge is
is: same each year higher in the earlier years
'revalued amount • Formula: of the asset's life
– net book value' cost - residual value • Formula:
• Revaluation is a choice useful life Depreciation rate
of accounting policy. All (%) × net book value
assets in the same class or
must be revalued (cost – residual
• Depreciation is now value) × %
based on the revalued
amount

190
Chapter 9: Questions

191
9: QUESTIONS

Data for Questions 9.1 and 9.2


Bungo Co purchases a car for its managing director, which would cost $17,000, by paying $1,000 cash, and
trading in an old vehicle. The old vehicle had a net book value of $15,500 immediately before the trade in took
place.

9.1 What is the effect of the above transaction on the profit for the year in respect of the disposal of the old
vehicle?
A Reduce profit by $1,500
B Increase profit by $1,500
C Reduce profit by $500
D Increase profit by $500 (2 marks)

9.2 Bungo Co charges depreciation at 10% per annum, with a full year’s charge in the year of acquisition.
What will the annual depreciation charge on the new vehicle be? $ (2 marks)

9.3 A company held property, plant and equipment at 31 December 20X5 with a net book value of $22,700.
During 20X6 items with a net book value of $2,100 were sold, realising a profit of $700. The depreciation
charge in the 20X6 income statement was $4,300. Items with a book value of $15,200 were revalued to
$21,250. At 31 December 20X6 the company’s statement of financial position showed the net book value
of property, plant and equipment as $44,100.
What was the cost of new property, plant and equipment acquired during 20X6?
A $13,150
B $17,550
C $22,050
D $21,750 (2 marks)
9.4 Nick
Nick started trading on 1 January 20X8 and bought equipment for his business as follows:
1 January 20X8 – Purchased a cutting machine for $4,960. The estimated useful life of the
machine is eight years, after which it will have no resale value.
2 January 20X8 – Purchased a car for $6,800.
1 March 20X8 – Purchased a van for $3,800. This has an estimated useful life of four years,
after which Nick believes he could sell it for $200.
1 May 20X8 – Purchased office furniture costing $5,400. This has an estimated useful life of
10 years with no resale value.
Depreciation for all assets, except the car, is to be calculated on the straight line basis, time apportioned
where the asset is owned for part of a year. The car is to be depreciated at 40% per annum on the
reducing balance basis.

Required
For the years ending 31 December 20X8 and 31 December 20X9, prepare relevant extracts from the
financial statements, together with the appropriate ledger accounts.

192
9: QUESTIONS

9.5 Eggo
On 1 January 20X4 Eggo Co, a manufacturer, acquired two identical grinding machines at a cost of
$10,000 each, and a duplicating machine at a cost of $3,000. The grinding machines are depreciated at
the rate of 30% per annum on a reducing balance basis, and the duplicating machine, which has an
estimated life of 10 years and a residual value of $500, is depreciated on a straight line basis. On
1 January 20X5 one of the grinding machines was sold for $5,000 and replaced by a new one costing
$12,000.
Required
Prepare the relevant ledger accounts dealing with the non-current assets, depreciation and the disposal
for the years to 31 December 20X4 and 31 December 20X5, respectively.
9.6 Hopkins
During 20X4 Hopkins gave his old van in part-exchange for a new van. The old van had cost $4,000 and
had accumulated depreciation of $2,400 at the date of exchange. Hopkins received a part-exchange
allowance of $1,800 and made a cash payment of $6,200 for the new van. Depreciation is over four years
on a straight line basis.
Required
(a) Calculate the profit or loss on disposal of the old van.
(b) Calculate the depreciation expense for the year ended 20X4.

193
9: QUESTIONS

194
Chapter 9: Answers

195
9: ANSWERS

9.1 D Profit on disposal = (17,000 – 1,000) – 15,500 = $500

9.2 $1,700
10% × $17,000 = $1,700

9.3 D

Property, plant and equipment (NBV)


$ $
B/d 22,700 Disposals 2,100
Revaluation 6,050 Depreciation 4,300
(21,250-15,200)
Additions ? C/d 44,100
50,500 50,500

∴ additions = $21,750
9.4 Nick
Income statement for the year ended 31 December .... (extract)
20X8 20X9
Depreciation Expense $ $
Machine 620 620
Car 2,720 1,632
Van 750 900
Furniture 360 540
4,450 3,692
Statement of financial position as at 31 December 20X8 (extract)
Non-current assets Cost Accumulated Net Book
depreciation Value
$ $ $
Machine 4,960 620 4,340
Car 6,800 2,720 4,080
Van 3,800 750 3,050
Furniture 5,400 360 5,040
20,960 4,450 16,510
Statement of financial position as at 31 December 20X9 (extract)
Non-current assets Cost Accumulated Net Book
depreciation Value
$ $ $
Machine 4,960 1,240 3,720
Car 6,800 4,352 2,448
Van 3,800 1,650 2,150
Furniture 5,400 900 4,500
20,960 8,142 12,818

Machine (SOFP)

$
1.1.X8 Bank 4,960

196
9: ANSWERS

Machine – Accumulated Depreciation (SOFP)

$ $
31.12.X8 bal c/d 620 31.12.X8 Dep’n expense:
machine 620
620 620
1.1.X9 bal b/d 620
31.12.X9 bal c/d 1,240 31.12.X9 Dep’n expense
: machine 620
1,240 1,240
1.1.Y0 bal b/d 1,240

Car (SOFP)

$
2.1.X8 Bank 6,800

Car – Accumulated Depreciation (SOFP)

$ $
31.12.X8 bal c/d 2,720 31.12.X8 Dep’n expense: car 2,720
2,720 2,720
1.1.X9 bal b/d 2,720
31.12.X9 bal c/d 4,352 31.12.X9 Dep'n expense: car 1,632
4,352 4,352
1.1.Y0 bal b/d 4,352

Van (SOFP)

$
1.3.X8 Bank 3,800

Van – Accumulated Depreciation (SOFP)

$ $
31.12.X8 bal c/d 750 31.12.X8 Dep’n expense: van 750
750 750
1.1.X9 bal b/d 750
31.12.X9 bal c/d 1,650 31.12.X9 Dep’n expense: van 900
1,650 1,650
1.1.Y0 bal b/d 1,650

Furniture (SOFP)

$
1.5.X8 Bank 5,400

197
9: ANSWERS

Furniture – Accumulated Depreciation (SOFP)

$ $
31.12.X8 bal c/d 360 31.12.X8 Dep’n expense: furniture
furniture 360
360 360
1.1.X9 bal b/d 360
31.12.X9 bal c/d 900 31.12.X9 Dep’n expense:
furniture 540
900 900
1.1.Y0 bal b/d 900

Depreciation Expense : Machine (I/S)

$ $
31.12.X8 Acc’d dep’n: machine 620 31.12.X8 I/S 620
31.12.X9 Acc’d dep’n: machine 620 31.12.X9 I/S 620

Depreciation Expense : Car (I/S)

$ $
31.12.X8 Acc’d dep’n: car 2,720 31.12.X8 I/S 2,720
31.12.X9 Acc’d dep’n: car 1,632 31.12.X9 I/S 1,632

Depreciation Expense : Van (I/S)

$ $
31.12.X8 Acc’d dep’n: van 750 31.12.X8 I/S 750
31.12.X9 Acc’d dep’n: van 900 31.12.X9 I/S 900

Depreciation Expense : Furniture (I/S)

$ $
31.12.X8 Acc’d dep’n: furniture 360 31.12.X8 I/S 360
31.12.X9 Acc’d dep’n: furniture 540 31.12.X9 I/S 540

Workings: Depreciation charge


20X8 20X9
$ $
Machine 4,960 ÷ 8 620 620
Car 6,800 × 40% 2,720 (6,800 – 2,720) × 40% 1,632
(note: reducing balance method)
Van (3,800 – 200) ÷ 4 = 900
10
900 × = 750
12
(10 months) 750 (full year) 900
8
Furniture (5,400) × (8 months) 360 (full year) 540
12

198
9: ANSWERS

9.5 Eggo
Grinding machines (SOFP)
$ $
1.1.X4 Bank 20,000 31.12.X4 Balance c/d 20,000

1.1.X5 Balance b/d 20,000 1.1.X5 Disposals 10,000


1.1.X5 Bank 12,000 31.12.X5 Balance c/d 22,000
32,000 32,000

1.1.X6 Balance c/d 22,000

Grinding machines – Accumulated Depreciation (SOFP)


$ $
31.12.X4 Balance c/d 6,000 31.12.X4 Dep'n expense (W) 6,000

1.1.X5 Disposals 3,000 1.1.X5 Balance b/d 6,000


31.12.X5 Balance c/d 8,700 31.12.X5 Dep'n expense (W) 5,700
11,700 11,700

1.1.X6 Balance b/d 8,700

Duplicating machine (SOFP)


$ $
1.1.X4 Bank 3,000 31.12.X4 Balance c/d 3,000

1.1.X5 Balance b/d 3,000 31.12.X5 Balance c/d 3,000


1.1.X5 Balance b/d 3,000

Duplicating machine – Accumulated Deprecation (SOFP)

$ $
31.12.X4 Balance c/d 250 31.12.X4 Depreciation expense 250
250 250
1.1.X5 Balance b/d 250
31.12.X5 Balance c/d 500 31.12.X5 Depreciation expense 250
500 500
1.1.X6 Balance b/d 500

Depreciation expense (I/S)


$ $
31.12.X4 Acc dep'n – grinding machines 6,000
31.12.X4 Acc dep'n – duplicating machine 250 31.12.X4 I/S 6,250
6,250 6,250

31.12.X5 Acc dep'n – grinding machines 5,700


31.12.X5 Acc dep'n – duplicating machine 250 31.12.X5 I/S 5,950
5,950 5,950

199
9: ANSWERS

Disposal account (I/S)


$ $
1.1.X5 Grinding machines 10,000 1.1.X5 Bank 5,000
1.1.X5 Acc Dep'n – grinding machine 3,000
31.12.X5 I/S – loss on disposal 2,000
10,000 10,000

Working
Depreciation charge

Year ended 31 December 20X4 20X5


$ $
Grinding machines ($20,000 × 30%) 6,000
Duplicating machine ($3,000 – $500) ÷ 10 250

Grinding machines
Machine 1 ($10,000 – $3,000) × 30% 2,100
Machine 2 ($12,000 × 30%) 3,600
Duplicating machine 250
6,250 5,950
9.6 Hopkins
(a) $200 profit
Working
Disposal account (I/S)
$ $
Cost 4,000 Accumulated depreciation 2,400
Profit on disposal 200 Part exchange allowance 1,800

4,200 4,200

Or alternatively:
$
"Proceeds" – part-exchange allowance 1,800
Net book value ($4,000 – $2,400) (1,600)
Profit on disposal 200
(b) $2,000
$
Cost of new van to be depreciated 8,000
($6,200 + $1,800)

Depreciate over four years 2,000

END OF CHAPTER

200
Intangible non-current
assets 10
Syllabus Guide Detailed Outcomes
Having studied this chapter you will be able to:
• Recognise the difference between tangible and intangible non-current assets.
• Identify types of intangible assets.
• Identify the definition and treatment of research and development costs in accordance with IFRS.
• Calculate amounts to be capitalised as development expenditure or to be expensed from given information.
• Calculate and account for the charge for amortisation and explain its purpose.

Exam Context
Intangible non-current assets are a smaller part of the syllabus than tangible non-current assets; however you should
still expect this area to be tested. Questions are likely to focus on the difference between tangible and intangible assets,
the accounting treatment for research and the capitalisation criteria for development expenditure. You should also be
confident in calculating amortisation.

Qualification Context
The knowledge covered in this chapter forms a platform which will be built on in the Fundamentals level paper Financial
Reporting (F7). There you will cover internally generated intangible assets and goodwill.

201
10: INTANGIBLE NON-CURRENT ASSETS

Overview

Intangible non-current
assets

Research Development expenditure

Accounting treatment Accounting treatment

Amortisation

202
10: INTANGIBLE NON-CURRENT ASSETS

1 Definition
1.1 An intangible non-current asset is an identifiable non-monetary asset without physical
substance.
1.2 The following are examples of intangible assets:
– Development expenditure
– Goodwill
– Concessions, patents, licences, trade marks.
The Paper F3 syllabus only requires knowledge of the accounting treatment of research and
development expenditure.

2 Research and development expenditure


2.1 Many companies, such as pharmaceutical companies, spend huge amounts on research
and development every year in order to maintain or enhance their competitive position.

2.2 Companies need to account for these costs and whilst the credit entry will be recorded as a
current liability, the question remains as to where the debit entry should be shown.
The choices are:
(a) to debit the income statement with an expense, or
(b) to debit the statement of financial position with an intangible non-current asset.
An intangible non-current asset should only be recorded when the entity is confident that the
expenditure will generate future profit.

3 IAS 38: Intangible assets


Definitions
3.1 (a) Research is original and planned investigation undertaken with the prospect of
gaining new scientific or technical knowledge and understanding.
(b) Development is the application of research findings or other knowledge to a plan or
design for the production of new or substantially improved materials, devices,
products, processes, systems or services before the start of commercial production or
use.

203
10: INTANGIBLE NON-CURRENT ASSETS

4 Accounting treatment
4.1
Research Development

• No certainty that the expenditure • Future profits are expected


will generate future profit

• Show as an expense in income • MUST capitalise as an intangible non-


statement current asset if all of the relevant criteria
are satisfied
• Dr Research expense (I/S) • Dr Intangible non-current assets (SOFP)
Cr Bank/payables Cr Bank/payables

P robable future economic benefits


I ntention to complete and use/sell asset
R esources adequate and available to complete
and use/sell asset
A bility to use/sell the asset
T echnical feasibility of completing asset for
use/sale
E xpenditure can be measured reliably

• Amortise asset over its useful life once


asset is ready for use

Lecture example 1 Preparation question

Z Co incurred the following costs during the year ended 31 August 20X8.
(1) $20,000 on salaries for market research staff sent out to canvass drivers' opinions on a
potential new car.
(2) $100,000 to purchase a machine to manufacture components for the new car. It has an
estimated useful life of 10 years.
(3) $25,000 on materials to manufacture a prototype and $50,000 on salaries relating to its
design and manufacture. The new car is expected to go on sale in 20X9.
Required
How should each of the above items be shown in the financial statements for the year ended
31 August 20X8?

204
10: INTANGIBLE NON-CURRENT ASSETS

Solution

5 Amortisation of capitalised development expenditure


5.1 A tangible non-current asset, such as a machine, is capitalised and then depreciated over its
useful life. This is to allocate its costs over the accounting periods which benefit from its use.

5.2 In the same way development expenditure which is incurred now will generate revenue and
profits in the future.
The cost of the development expenditure should be matched against the revenue it
produces. This is called amortisation.

5.3 The 'depreciable amount' (cost less residual value) should be amortised over the useful life
in the same way that revenues are expected to be generated.

5.4 Amortisation should begin when the asset is ready for use.

205
10: INTANGIBLE NON-CURRENT ASSETS

5.5 It is an expense in the income statement and is accounted for using the following entry:
Dr Amortisation expense (I/S)
Cr Accumulated amortisation (SOFP)

Lecture example 2 Technique demonstration

Development Co incurs the following expenditure in years 20X1 – 20X5.


Research Development
$ $
20X1 35,000 55,000
20X2 – 65,000
20X3 – –
20X4 – –
20X5 38,000 –
The development expenditure meets the IAS 38 criteria that require capitalisation ('PIRATE'). The
item developed in 20X1 and 20X2 goes on sale on 1.1.X3 and it will be three years from then until
any competitor is expected to have a similar product on the market.
Required
Show income statement and statement of financial position extracts for the years 20X1 – 20X5
inclusive.

Solution
Income statement extracts
X1 X2 X3 X4 X5
$ $ $ $ $
Expenses
Research expenditure
Amortisation of development expenditure

Statement of financial position extracts


X1 X2 X3 X4 X5
$ $ $ $ $
Non-current assets
Development expenditure
Amortisation

Net book value

206
10: INTANGIBLE NON-CURRENT ASSETS

6 Chapter summary
Quick Quiz Section Topic Summary
1 Definition An intangible non-current asset is an identifiable non-
monetary asset without physical substance.
2 Research and Some entities spend significant sums of money on
development research and development it is therefore essential that
expenditure these transactions are accounted for appropriately.
3 Intangible assets IAS 38 defines research and development.
(IAS 38) Research expenditure is incurred where the entity is
acquiring new scientific or technical knowledge.
Development expenditure relates to the application of
research findings.
4 Accounting treatment Research relates to costs incurred to obtain
knowledge or understanding. There is no certainty of
future profit from this expenditure and so it should be
shown as an expense in the income statement.
Development expenditure MUST be capitalised as
an intangible non-current asset provided all of the
PIRATE criteria are met. This asset will then be
amortised over the period during which it is expected
to generate income.
5 Amortisation of Amortisation is essentially the same as depreciation but
capitalised relates to intangibles. Where an entity has capitalised
development development expenditure it should amortise the
expenditure intangible once the asset is ready for use.

207
10: INTANGIBLE NON-CURRENT ASSETS

Chapter Summary

Intangible non-current
assets

Research Development expenditure

'Investigation to gain new scientific or technical knowledge 'Application of research findings or other knowledge
and understanding' to produce new/substantially improved materials,
processes etc'

Accounting treatment Accounting treatment

• Future profits are expected


• There is no certainty of future profits
• Capitalise as an intangible non-current asset if all PIRATE
• Write-off as an expense in the income
criteria are satisfied
statement
• PIRATE:
Probable future economic benefits
Intention to complete and use/sell asset
Resources adequate and available to use/sell asset
Ability to use/sell asset
Technical feasibility of completing asset for use/sale
Expenditure can be measured reliably

Amortisation

• Amortise asset over its useful life once asset is ready for use

208
Chapter 10: Question

209
10: QUESTION

10.1 Which of the following statements about research and development are true?
(1) Development expenditure shown on the statement of financial position should be amortised over
the periods expected to benefit from the product or service.
(2) Development expenditure must be capitalised if it meets various criteria.
(3) Research expenditure is always written off.
A All of the above
B (1) and (2)
C (2) and (3)
D (1) and (3) (2 marks)

210
Chapter 10: Answer

211
10: ANSWER

10.1 A

END OF CHAPTER

212
Accruals and
prepayments 11
Syllabus Guide Detailed Outcomes
Having studied this chapter you will be able to:
• Understand how the matching concept applies to accruals and prepayments.
• Identify and calculate the adjustments needed for accruals and prepayments in preparing financial statements.
• Prepare the journal entries and ledger entries for the creation of an accrual or prepayment.
• Understand and identify the impact on profit and net assets of accruals and prepayments.

Exam Context
Accruals and prepayments are key accounting adjustments and you should expect to see them tested in Paper F3. You
may be asked to calculate the statement of financial position amount for accruals and prepayments and/or the relevant
expense that would be shown in the income statement. Alternatively, you may be asked to determine the appropriate
journal entries to record accruals and prepayments. The Pilot Paper included questions on the calculation of a year-end
prepayment of an expense and the income to be shown in the income statement where rent is received both in advance
and in arrears.

Qualification Context
This area is a basic skill which is not tested in detail in any other paper. The matching concept however is fundamental
to the preparation of financial statements and this is relevant to Paper F7, Financial Reporting.

213
11: ACCRUALS AND PREPAYMENTS

Overview

Accruals and prepayments

Accounting treatment

Year end adjustments Reversing out accruals and Presentation in the statement
prepayments of financial position

Accrued income
and deferred income

Accounting treatment

214
11: ACCRUALS AND PREPAYMENTS

1 Introduction
1.1 This chapter is designed to enable you to apply accounting concepts and principles in
relation to the calculation of and adjustments for accruals and prepayments.

1.2 IAS 1 requires financial statements to be prepared on an accruals basis. This is so that
transactions and events are recognised when they occur (and not as cash or its equivalent
is received or paid) and they are recorded in the accounting records and reported in the
financial statements of the period to which they relate.
The accruals basis is also an underlying assumption in the IASB's Framework for the
Preparation and Presentation of Financial Statements.

Accruals
1.3 Accruals are expenses incurred by the business during the accounting period but not yet
paid for, i.e. expenses in arrears.

Example
1.4 Fred prepares accounts to 31 December each year. On 1 January 20X8, he pays a
telephone bill of $60 which relates to the period October-December 20X7.
Although the payment does not go through the cash book until 20X8, this expense must be
included in the accounts for the year ended 31 December 20X7, as it was incurred during
this period.

Prepayments
1.5 Prepayments arise when expenses are paid for before they have been used, i.e. expenses
in advance.

Example
1.6 On 20 December 20X7 Fred pays for insurance on his business premises for the 12 months
commencing 1 January 20X8.
Although the payment was made in 20X7, the expense should not appear in the accounts
for 20X7. The accounts for 20X7 will show a prepayment for the full amount of the insurance
cost and the expense will be recorded in 20X8.

215
11: ACCRUALS AND PREPAYMENTS

2 Accounting treatment
Year-end adjustments
2.1 Adjustments for accruals and prepayments tend to occur at the end of the year and are
made by way of a journal entry. The required entries are:

Accruals
Dr Expense (I/S)
Cr Accruals (SOFP)

Prepayments
Dr Prepayments (SOFP)
Cr Expense (I/S)

Presentation in the statement of financial position


2.2 Accruals:
Sub-heading under 'current liabilities'
Prepayments:
Sub-heading under 'current assets'.

Lecture example 1 Preparation question

Fiona set up a business on 1 January 20X7. Her cash payments for the year to 31 December 20X7
included:

Date paid Amount Period


$
Electricity
10.3.X7 96 2 months to 28 February 20X7
12.6.X7 120 quarter to 31 May 20X7
14.9.X7 104 quarter to 31 August 20X7
10.12.X7 145 quarter to 30 November 20X7

Rent
1.2.X7 375 3 months to 31 March 20X7
6.4.X7 1,584 12 months to 31 March 20X8
Note: On 6 March 20X8 Fiona received an electricity bill for $168 for the quarter to 28 February
20X8.

216
11: ACCRUALS AND PREPAYMENTS

Required
(a) Calculate the expense incurred by Fiona for electricity and rent for the year ended
31 December 20X7.
(b) Calculate the amount of any accruals/prepayments at the end of the year.
(c) State the journal entry required for the year-end adjustments.

Solution

217
11: ACCRUALS AND PREPAYMENTS

Lecture example 2 Preparation question

Required
Using the figures from Lecture example 1:
Complete the necessary entries in Fiona’s ledger accounts as at 31 December 20X7, then balance
off the accounts.

Solution
Electricity expense (I/S)
$ $
10.3.X7 Cash 96
12.6.X7 Cash 120
14.9.X7 Cash 104
10.12.X7 Cash 145

Rent expense (I/S)


$ $
1.2.X7 Cash 375
6.4.X7 Cash 1,584

Accruals (SOFP)
$ $

Prepayments (SOFP)
$ $

Section 1.9

218
11: ACCRUALS AND PREPAYMENTS

3 Reversing out accruals and prepayments


Problem
3.1 Using the figures from Lecture example 1, what is Fiona’s rent expense for the year to 31
December 20X8 assuming that on 10 April 20X8 she paid rent of $1,740 for the 12 months
commencing 1 April 20X8?

3.2 1.1.X8 1.4.X8 31.12.X8

Expense = ( 3 12 × $1,584) + ( 9 12 × $1,740) = $1,701

Double entry
3.3
Rent expense
$ $
10.4.X8 Cash 1,740 31.12.X8 Prepayments
( 3 12 × 1,740 ) 435

Prepayments
$ $
1.1.X8 Balance b/d 396
31.12.X8 Rent 435

This does not produce a sensible answer! The rent expense in the ledger account would
result in a charge to the income statement of $1,305 (not $1,701) and the balance on the
prepayment account would be overstated by $396.

219
11: ACCRUALS AND PREPAYMENTS

Solution
3.4 The opening prepayment must therefore be reversed, ie:
Debit Rent expense (I/S) $396
Credit Prepayments (SOFP) $396
Post this to the ledger accounts in 3.3 and balance off – the expense should now be correct!

Summary
3.5 Accruals and prepayments brought forward at the start of the year must be reversed.
Reversal of accrual
Dr Accruals (SOFP)
Cr Expense (I/S)
Prepayments

Dr Expense (I/S)
Cr Prepayments (SOFP)

Approach to questions
3.6 There are four steps to follow:
(1) Reverse opening accrual/prepayment.
(2) Post cash paid during the year.
(3) Post closing accrual/prepayment.
(4) Balance off the accounts.

Lecture example 3 Preparation question

In 20X8 Fiona paid the following electricity bills:

Date paid Amount Period


$
12.3.X8 168 quarter to 28 February 20X8
9.6.X8 134 quarter to 31 May 20X8
12.9.X8 118 quarter to 31 August 20X8
12.12.X8 158 quarter to 30 November 20X8
During March 20X9 Fiona received an electricity bill for $189 for the quarter to 28 February 20X9.
Required
Calculate the electricity expense and accrual for the year ended 31 December 20X8 and complete
the ledger accounts.

220
11: ACCRUALS AND PREPAYMENTS

Solution
Electricity expense (I/S)
$ $

Accruals (SOFP)
$ $

Lecture example 4 Exam standard question for 2 marks

Jimmy Co prepares its financial statements for the year to 30 June each year. The company pays
for its insurance quarterly in advance on 1 March, 1 June, 1 September and 1 December each
year. The annual insurance premium was $24,000 until 31 August 20X6, after that date it
increased to $30,000 per year.
Required
What insurance expense and end of year prepayment should be included in the financial
statements for the year ended 30 June 20X7?
Expense Prepayment
A $29,000 $2,500
B $29,000 $5,000
C $28,500 $2,500
D $28,500 $5,000

221
11: ACCRUALS AND PREPAYMENTS

Solution

222
11: ACCRUALS AND PREPAYMENTS

Additional Notes

223
11: ACCRUALS AND PREPAYMENTS

4 Accrued income and deferred income


4.1 Accruals and prepayments relate to when expenses are paid in arrears or advance. Income
may also be received in arrears or advance.

Accrued income
4.2 This relates to when income has been earned during the accounting period but not invoiced
or received.

Illustration
4.3 Jenny owns a property which she rents out for $3,000 per quarter. The property was
occupied all year; however Jenny only received $9,000 in rent because she forgot to send
out the final invoice of the year.
As the property was let for 12 months, Jenny's income statement should show income of
$12,000 (4 × $3,000) as this is what she has earned.
She will therefore need to accrue the 'missing' income of $3,000 as a year end journal and
also show a receivable for "rent in arrears".
The adjustment is:
$ $
Dr Rent in arrears (SOFP) 3,000
Cr Rental income (I/S) 3,000
The rent in arrears is shown in the statement of financial position within current assets.

Deferred income
4.4 This relates to when income is received in advance of it being earned.

Illustration
4.5 Ben has a year end of December and rents out his property for $1,000 per month. His
tenant pays on time each month and during December 20X7 paid Ben $2,000 as he would
be away when the January 20X8 payment was due.
Ben has received income of $13,000 but only $12,000 of this relates to the current year. He
must therefore remove $1,000 of income from this year's accounts because it relates to next
year. A liability will also be shown for "rent in advance".
The adjustment is:
$ $
Dr Rental income (I/S) 1,000
Cr Rent in advance (SOFP) 1,000
The rent in advance is shown in the statement of financial position within current liabilities.

224
11: ACCRUALS AND PREPAYMENTS

Approach to questions
4.6 The approach for accrued income and deferred income is exactly the same as for accruals
and prepayments.
There are four steps to follow:
(1) Reverse opening rent in arrears/advance.
(2) Post cash received during the year.
(3) Post closing rent in arrears/advance.
(4) Balance off the accounts.

5 Chapter summary
Quick Quiz Q2-5
Section Topic Summary

1 Introduction An entity should produce its financial statements using


the accruals basis. This is an underlying assumption in
the IASB Framework.
Accruals are made when expenses are paid in
arrears, whereas prepayments arise when expenses
are paid for in advance.
2 Accounting treatment Accruals increase expenses and are shown as a liability
on the statement of financial position at the year end.
Prepayments reduce expenses and are an asset on the
statement of financial position.
3 Reversing out accruals Accruals and prepayments from the previous year are
and prepayments reversed at the beginning of the next accounting period
so that the current year expense is correct.
4 Accrued income and These follow a similar theory to accruals and
deferred income prepayments but relate to income.
An entity will accrue income where it has earned the
income during the period but not yet invoiced for it.
This will increase income and be shown as a receivable
at the year end.
Where an entity has received income in advance of it
being earned it should be deferred to the following
period. This will reduce income and be shown as a
payable at the year end.

225
11: ACCRUALS AND PREPAYMENTS

6 Double Entry Summary for Chapter 11


6.1 Accruals adjustment:

Dr Expense (I/S)
Cr Accruals (SOFP)

6.2 Prepayments adjustment:

Dr Prepayments (SOFP)
Cr Expense (I/S)

6.3 Approach to questions (four steps):


(1) Reverse opening accrual/ prepayment:

Accruals:
Dr Accruals (SOFP)
Cr Expense (I/S)

Prepayments:
Dr Expense (I/S)
Cr Prepayments (SOFP)

(2) Post cash paid during the year.


(3) Post closing accrual/ prepayment.
(4) Balance off the ledger accounts.

226
11: ACCRUALS AND PREPAYMENTS

Chapter Summary
'Accruals: expenses incurred by the business during the period but not yet
paid for'
'Prepayments: expenses paid for before they have been used'

Accruals and prepayments

Accounting treatment

Year end adjustments Reversing out accruals and Presentation in the statement
prepayments of financial position
• Accruals increase expenses • Accruals and prepayments brought forward • Accruals: current liabilities
and represent a liability: at the start of the year must be reversed • Prepayments: current assets
Dr Expenses (I/S)
• Steps to answering questions:
Cr Accruals (SOFP)
(1) Reverse opening accrual/prepayment
(2) Post cash paid during the year
• Prepayments decrease (3) Post closing accrual/prepayment
expenses and are an asset at (4) Balance off the accounts
the year end:
Dr Prepayments (SOFP)
Cr Expenses (I/S)

Accrued income
and deferred income

'Accrued income: income earned but not yet invoiced'


'Deferred income: income invoiced but not yet earned'

Accounting treatment

Accrued income: Dr Receivable (SOFP)


Cr Income (I/S)
Deferred income: Dr Income (I/S)
Cr Payable (SOFP)
• Opening accrued and deferred income balances
must be reversed at the beginning of each year.

227
11: ACCRUALS AND PREPAYMENTS

228
Chapter 11: Questions

229
11: QUESTIONS

Data for Questions 11.1 – 11.3


A company made the following payments in 20X5 in respect of rent and telephone expenses:
Rent Date paid Amount
$
Quarter ended 31 January 20X5 02.01.20X5 300
Quarter ended 30 April 20X5 02.01.20X5 300
Quarter ended 31 July 20X5 30.04.20X5 450
Quarter ended 31 October 20X5 31.07.20X5 450
Quarter ended 31 January 20X6 01.11.20X5 450

Telephone
Quarter ended 31 January 20X5 02.03.20X5 270
Quarter ended 30 April 20X5 05.06.20X5 310
Quarter ended 31 July 20X5 02.09.20X5 320
Quarter ended 31 October 20X5 10.12.20X5 330
A telephone bill for $345 in respect of the quarter ended 31 January 20X6 was received by the company in
February 20X6. The company's year end is December.

11.1 What balance should have been brought forward on the accruals account in relation to rent payable at
1 January 20X5?
A $100 credit
B $200 credit
C $100 debit
D $200 debit (2 marks)
11.2 What will be the income statement charge for telephone expenses for the year ended 31 December
20X5?
A $1,165
B $1,180
C $1,255
D $1,280 (2 marks)

11.3 At 31 December 20X5 what balance will be included as a prepayment or accrual in respect of rent?
A $300 prepayment
B $200 accrual
C $150 prepayment
D $150 accrual (2 marks)

11.4 At 31 December 20X8 Blue Anchor Co has an insurance prepayment of $250. During the year they pay
$800 in respect of various insurance contracts. The closing accrual for insurance is $90.
What is the income statement charge for insurance for year ended 31 December 20X9? $
(2 marks)

230
11: QUESTIONS

11.5 Max has paid his rent for the period 1 April 20X0 to 30 June 20X1 of $4,800. His first set of accounts is
drawn up for the period from 1 April 20X0 to 28 February 20X1. His accounts should reflect
A Rent expense of $4,800 only
B Rent expense of $3,520, a prepayment of $1,280
C Rent expense of $3,600, a prepayment of $1,200
D Rent expense of $3,840, a prepayment of $960 (2 marks)

11.6 Constains Co has an insurance prepayment of $320 at 31 March 20X2. During the year ended 31 March
20X2 Constains paid two insurance bills, one for $1,300 and one for $520. The charge for the year in the
accounts for insurance was $1,760.
What was the prepayment at 31 March 20X1? $ (2 marks)

11.7 An electricity prepayment for $300 was treated as an accrual in a sole trader’s income statement. As a
result the profit was
A Overstated by $600
B Understated by $300
C Understated by $600 (1 mark)
11.8 A. Cruel
A. Cruel prepares his financial statements for the year to 31 December each year. He pays rent on his
premises quarterly in advance on 1 February, 1 May, 1 August and 1 November. The annual rent was
$12,000 until 30 September 20X7 and $15,000 per year thereafter.
(i) What rent expense and prepayment should be included in the financial statements for the year
ended 31 December 20X7?
Expense Prepayment
A $12,750 $1,250
B $12,750 $2,500
C $15,000 $2,250
D $15,000 $1,250
(ii) The following year the reversal of the prepayment will result in which of the following in the rent
expense account?
A Credit balance of $1,250
B Debit balance of $1,250
C Credit balance of $2,500
D Debit balance of $2,250
(iii) A. Cruel has just looked at the accounts you have prepared and is confused as he knows he has
paid more rent than is showing in the income statement.
Which accounting concept means that the income statement may not just show the cash paid?
A Going concern
B Accruals
C Business entity

231
11: QUESTIONS

11.9 Fairlop
The accounts of Fairlop are made up to 31 December every year. When preparing the accounts for 20X7
you extract the following information from the payments side of the cash book:
$
20X6
1 October Rent (to 31.3.X7) 500
20X7
10 January Electricity 300
1 April Rent 550
10 April Electricity 300
10 July Electricity 250
1 October Rent 550
10 October Electricity 250
20X8
10 January Electricity 350
You ascertain that rent is paid half-yearly in advance and that electricity bills relate to the quarter ended in
the month before payment.
Required
Calculate the following amounts:
(i) The rent expense for the year ended 31 December 20X7
(ii) The electricity expense for the year ended 31 December 20X7
(iii) The balance on the prepayment account at 31 December 20X7
(iv) The balance on the accruals account at 31 December 20X7

232
Chapter 11: Answers

233
11: ANSWERS

11.1 B 2
3 × 300 = 200

11.2 D
$
Reverse accrual at 1.1.X5 (180)
Paid (270 + 310 + 320 + 330) 1,230
Accrual at 31.12.X5 ( 2 3 x 345) 230
∴ I/S charge 1,280

11.3 C 1
3 × 450 = 150

11.4 $1,140
$250 + $800 + $90 = $1,140

11.5 B Rent for the 15-month period $4,800


Prepayment 4 15 × $4,800 $1,280

11.6 $260
Insurance Expense
$ $
∴ Prepayment reversal (β) 260
Cash 1,300 I/S 1,760
Cash 520 Prepayment 320

2,080 2,080

11.7 C The prepayment would have decreased the electricity expense by $300 and increased profits.
Treating the prepayment as an accrual would have increased the electricity expense and
decreased profit. Profit is therefore understated by 2 × $300 = $600.
11.8 A. Cruel
(i) A
Rent expense: $
January – September 20X7 ($12,000 × 9/12) 9,000
October – December 20X7 ($15,000 × 3/12) 3,750
12,750
Prepayment:
1 November payment of $3,750 ($15,000 × ¼) relates to November, December and January.
∴ prepay January 20X8 expense: $3,750 × 1/3 = $1,250.
(ii) B
(iii) B

234
11: ANSWERS

11.9 Fairlop
(i) Rent expense: $1,075
(ii) Electricity expense: $1,150
(iii) Prepayments: $275
(iv) Accruals: $350
Workings
Prepayments (SOFP)

$ $
1.1.X7 Balance b/d (500 × 3/6) 250 1.1.X7 Rent 250
31.12.X7 Rent (550 × 3/6) 275 31.12.X7 Balance c/d 275
525 525

1.1.X8 Balance b/d 275

Accruals (SOFP)

$ $
1.1.X7 Electricity 300 1.1.X7 Balance b/d 300
31.12.X7 Balance c/d 350 31.12.X7 Electricity 350
650 650

1.1.X8 Balance b/d 350

Rent (I/S)

$ $
1.1.X7 Prepayments 250
1.4.X7 Bank 550
1.10.X7 Bank 550 31.12.X7 Income statement 1,075
31.12.X7 Prepayments 275
1,350 1,350

Electricity (I/S)

$ $
1.1.X7 Accruals 300
10.1.X7 Bank 300
10.4.X7 Bank 300
10.7.X7 Bank 250
10.10.X7 Bank 250
31.12.X7 Accruals 350 31.12.X7 Income statement 1,150
1,450 1,450

235
11: ANSWERS

END OF CHAPTER

236
Irrecoverable debts
and allowances 12
Syllabus Guide Detailed Outcomes
Having studied this chapter you will be able to:
• Explain and identify examples of receivables and payables.
• Identify the benefits and costs of offering credit facilities to customers.
• Understand the purpose of credit limits and an aged receivables analysis.
• Prepare the bookkeeping entries to write off a bad debt, record a bad debt recovered and create and adjust an
allowance for receivables.
• Identify the impact of bad debts on the income statement and on the statement of financial position.
• Illustrate how to include movements in the allowance for receivables in the income statement and how the
closing balance of the allowance should appear in the statement of financial position.
• Account for contras between trade receivables and payables.
• Prepare, reconcile and understand the purpose of supplier statements.
• Classify items as current or non-current liabilities in the statement of financial position.

Exam Context
Questions on this topic are likely to require you to perform basic calculations dealing with writing off debts, adjusting for
cash subsequently received and adjusting the allowance for receivables. You will also need to be able to determine the
balances to be shown in the income statement and statement of financial position.

Qualification Context
This area is a basic skill and detailed calculations are not tested in any other paper.

237
12: IRRECOVERABLE DEBTS AND ALLOWANCES

Overview Amounts recovered

Bad debts

Irrecoverable debts
and allowances

Doubtful debts

Allowances

Specific General

238
12: IRRECOVERABLE DEBTS AND ALLOWANCES

1 Introduction
1.1 This chapter is designed to enable you to calculate and make adjustment for bad debts, and
allowances for receivables.

1.2 A trade receivable should only be classed as an asset if it is probable that it is recoverable
(ie that the customer will pay the amounts due).

2 Bad debts
2.1 If a debt is definitely irrecoverable it should be written off to the income statement as a
bad debt. This is an example of prudence.

Accounting treatment
2.2 Dr Bad debt expense (I/S)
Cr Trade receivables (B/S)

You may see the debit entry being made to an 'irrecoverable debts expense' account. This
is effectively the same thing.

Lecture example 1 Preparation question

Fight & Co has trade receivables at 31 December 20X7 of $65,000. A review of customer files
indicates that two customers, Ali and Tyson, which owe $7,000 and $8,000 respectively, have
gone bankrupt and their debts are considered irrecoverable.
Required
(a) Calculate the balance c/d on the trade receivables account at the end of the year.
(b) Calculate the bad debt expense shown in the income statement.

Solution

Trade receivables (SOFP)


$ $
31.12.X7 Bal b/d 65,000

Bad debt expense (I/S)


$ $

239
12: IRRECOVERABLE DEBTS AND ALLOWANCES

3 Doubtful debts
3.1 If a debt is possibly irrecoverable an allowance for the potential irrecoverability of that debt
should be made. A new account is created, Allowance for receivables, this account is offset
against the trade receivables’ balance on the statement of financial position and the
expense taken to the income statement.

Accounting treatment
3.2 Dr Doubtful debts expense (I/S)
Cr Allowance for receivables (SOFP)

Again, an 'irrecoverable debts expense' account can also be used.

Lecture example 2 Preparation question

A further review of Fight & Co's customer files indicates there is some uncertainty as to whether a
debt of $3,500 owed by Bugner is recoverable.
(a) Calculate the allowance for receivables shown on the statement of financial position.
(b) Calculate the doubtful debts expense shown in the income statement.
(c) Show how the information from Lecture examples 1 and 2 would be shown in extracts from
the income statement and statement of financial position.

Solution

Allowance for receivables (SOFP)

Doubtful debts expense (I/S)

240
12: IRRECOVERABLE DEBTS AND ALLOWANCES

Types of allowance
3.3 (a) Specific: provided against a particular/named individual customer.
(b) General: percentage applied to total trade receivables after:
(i) writing off bad debts;
(ii) deducting full balance of any customers for which specific allowance has been
created.

Order of calculation
3.4 (a) Write up trade receivables and account for credit sales and cash received in period.
(b) Write off bad debts
Dr Bad debt expense (I/S)
Cr Trade receivables (SOFP)
(c) Make any entries for specific allowances:
Dr Doubtful debts expense (I/S)
Cr Allowance for receivables (SOFP)
(d) In workings, calculate the general allowance on trade receivables (after bad debts
written off and excluding full amounts for which specific allowance has been made).
(e)
$
Total trade receivables 100
Less: specific allowances (20)
80

General allowance @ 5% = 4

∴total allowance:
Specific 20
General 4
24

241
12: IRRECOVERABLE DEBTS AND ALLOWANCES

Lecture example 3 Preparation question

A business’s trade receivables account showed a year end balance of $47,440. It was decided that
amounts totalling $340 should be written off as irrecoverable, a specific allowance was to be made
against an amount of $400 due from Dodgy Co, a customer, and a general allowance of 2% was to
be made against remaining debts.
Required
(a) Calculate the allowance for receivables shown in the statement of financial position.
(b) Calculate the bad and doubtful debts expense shown in the income statement.

Solution
Trade receivables (SOFP)
$ $
Balance b/d 47,440

Allowances for receivables (SOFP)


$ $

Bad and doubtful debts expense (I/S)


$ $

General allowance
$
Trade receivables (net of bad debts written off)
Less: specific allowance

General allowance @ 2%

242
12: IRRECOVERABLE DEBTS AND ALLOWANCES

4 Effect in subsequent periods


Bad debts written off last year, customer pays this year
4.1 If a bad debt is recovered having previously been written off, it is credited to the bad debt
expense account, i.e. the accounting treatment from the original write-off is reversed.
Accounting treatment
(1) Cash received

Dr Cash
Cr Trade receivables

Reverse original
write off

Dr Trade receivables
Cr Bad debt expense

OR

(2) Short method

Dr Cash
Cr Bad debt expense

Lecture example 4 Preparation question

Fight & Co (see Lecture example 1) subsequently receive a cheque of $7,000 from Ali.
Required
Show the treatment of this recovery in the relevant ‘T’ accounts.

Solution
Trade receivables (SOFP)
$ $
1.1.X8 Bal b/d 50,000

243
12: IRRECOVERABLE DEBTS AND ALLOWANCES

Bad debt expense (I/S)


$ $

Cash (SOFP)
$ $

Doubtful debts – specific allowance last year, customer pays outstanding


amounts this year
4.2 A credit entry for the cash is made to the trade receivables account because the debt is still
included in the total trade receivables figure.
The allowance is then reversed as it is no longer needed.
Accounting treatment
(a) Record the cash received
Dr Cash (SOFP)
Cr Trade receivables (SOFP)
then:
(b) Remove allowance
Dr Allowance for receivables (SOFP)
Cr Doubtful debts expense (I/S)

244
12: IRRECOVERABLE DEBTS AND ALLOWANCES

Lecture example 5 Preparation question

Required
Show the accounting treatment for Fight & Co if, having made a specific allowance (see Lecture
example 2), during the next year Bugner repays his debt of $3,500 to Fight & Co in cash?

Solution
Trade receivables (SOFP)
$ $
1.1.X8 Bal b/d 50,000

Allowance for receivables (SOFP)


$ $
1.1.X8 Bal b/d 3,500

Bad and doubtful debt expense (I/S)


$ $

Doubtful debts – specific allowance last year, goes bad this year
4.3 The debt is no longer doubtful, but definitely bad. It should therefore be removed from the
trade receivables and the allowance for receivables accounts.

Dr Allowance for receivables (SOFP)


Cr Trade receivables (SOFP)

245
12: IRRECOVERABLE DEBTS AND ALLOWANCES

Lecture example 6 Preparation question

Required
Following on from the information used in Lecture example 2, suppose that in the next accounting
period, the debt from Bugner is considered to have gone bad.
What double entry would be required to record this?

Solution

Doubtful debts - general allowance


4.4 Allowance is usually changed at the end of each period to reflect the change in value of total
trade receivables.

Accounting treatment
4.5
(1) Remove opening allowance
Dr Allowance for receivables
Cr Doubtful debts expense

Replace with closing allowance

Dr Doubtful debts expense


Cr Allowance for receivables

246
12: IRRECOVERABLE DEBTS AND ALLOWANCES

OR

(2) Short method:


Increase/decrease opening allowance to arrive at required closing allowance
Increase:
Dr Doubtful debts expense
Cr Allowance for receivables

Decrease:
Dr Allowance for receivables
Cr Doubtful debts expense

Lecture example 7 Preparation question

The following information is available for A Co.


Year ended 31 December 20X7: Trade receivables $20,000
Year ended 31 December 20X8: Trade receivables $30,000
A Co requires a general allowance of 5% of trade receivables in each year.
Required
Show the required adjustment to the allowance for receivables account in the year ended 31
December 20X8 using both methods described in section 4.5

Solution
Long method: 4.5 (1)

Allowance for receivables


$ $

Doubtful debts expense


$ $

247
12: IRRECOVERABLE DEBTS AND ALLOWANCES

Short method: 4.5 (2)


Allowance for receivables
$ $

Doubtful debts expense


$ $

Lecture example 8 Exam standard for 2 marks

At 30 September 20X7 G Co had an allowance for receivables of $24,000.


During the year ended 30 September 20X8 G Co recovered $2,000 from a customer whose
balance was written off in 20X7 and wrote off further debts totalling $18,000. The closing
allowance for receivables is required to be $21,000. No adjustments have been made for this
information.
Required
What amount should appear in the income statement for the year ended 30 September 20X8 for
the above items?
A $13,000
B $15,000
C $17,000
D $23,000

248
12: IRRECOVERABLE DEBTS AND ALLOWANCES

Solution

5 Chapter summary
Quick Quiz
Section Topic Summary

1 Introduction A trade receivable is an asset of the business which


should only be shown in the financial statements if it is
believed to be recoverable.
2 Bad debts Bad or irrecoverable debts must therefore be written
off as an expense in the income statement.
3 Doubtful debts An allowance should be made against trade
receivables where there is concern as to whether or not
a balance will be recoverable. There are two types of
allowance: specific and general.
Specific allowances relate to particular customer
balances whereas a general allowance is usually a
percentage of remaining debts.
4 Effect in subsequent The key to being able to account for the effect in
periods subsequent periods is to know what accounting entries
have previously been made and then make any
relevant adjustments.
For example, if cash is received from a receivable that
was previously written off then the receivable has
already been removed from the accounts.
Consequently the only adjustments needed are to
record the cash received and remove the bad debt
expense recorded last year which has proved to be
unnecessary.

249
12: IRRECOVERABLE DEBTS AND ALLOWANCES

6 Double Entry Summary for Chapter 12


6.1 Bad (irrecoverable) debt adjustment:

Dr Bad debt expense (I/S)


Cr Trade receivables (SOFP)

6.2 Doubtful debt adjustment:

Dr Doubtful debts expense (I/S)


Cr Allowance for receivables (SOFP)

6.3 Recording of cash received from a customer whose balance was previously written off:

Dr Cash (SOFP)
Cr Bad debt expense (I/S)

6.4 Recording of cash received from a customer against which a specific allowance was
previously made:

Record cash received:


Dr Cash (SOFP)
Cr Trade receivables (SOFP)

Remove the allowance:


Dr Allowance for receivables (SOFP)
Cr Doubtful debts expense (I/S)

6.5 Writing a balance off as irrecoverable where a specific allowance was previously made:

Dr Allowance for receivables (SOFP)


Cr Trade receivables (SOFP)

250
12: IRRECOVERABLE DEBTS AND ALLOWANCES

Chapter Summary Amounts recovered

• Record cash received and remove bad debt expense which


was previously recognised:
Dr Cash
Cr Bad debt expense

Bad debts

'A debt which is definitely irrecoverable'


• Write off to the income statement:
Dr Bad debt expense (I/S)
Cr Trade receivables (SOFP)

Irrecoverable debts
and allowances

Doubtful debts

'A debt which is possibly irrecoverable'


• Make an allowance against the debt:
Dr Doubtful debts expense (I/S)
Cr Allowance for receivables (SOFP)

Allowances

• Reduce the value of trade receivables shown in the


statement of financial position

Specific General

• Provided where there is doubt over the recoverability of a particular


customer's balance • A percentage applied to total trade
receivables after
• When cash is subsequently received
(1) writing off bad debts
(1) Record cash:
(2) deducting the total balance owed by
Dr Cash
customers where a specific allowance
Cr Trade receivables
has been made
(2) Remove allowance
Dr Allowance for • The general allowance is increased or
decreased as necessary at each year end
receivables
Cr Doubtful debts expense
• If debt subsequently goes bad remove from trade receivables and the
allowance for receivables:
Dr Allowance for receivables
Cr Trade receivables
• Note the 'loss' was originally recorded when the allowance was made.

251
12: IRRECOVERABLE DEBTS AND ALLOWANCES

252
Chapter 12: Questions

253
12: QUESTIONS

12.1 A company receives news that a major customer has been declared bankrupt. The entries now required
are:
A Debit bad debt expense, Credit trade receivables
B Debit sales, Credit trade receivables (1 mark)

12.2 At 1 January 20X9 Farriers has an allowance for receivables of $2,000 consisting of a specific allowance
for $700 in respect of Black Lion Co and a $1,300 general allowance. During the year Black Lion goes
into liquidation and the debt is written off. No other debts go bad and at 31 January 20X9 the balance on
the trade receivables is $50,950. Farriers wishes to provide for a debt of $950 from Verulam and to have
a general allowance of 2½% of good trade receivables. The bad and doubtful debts charged to the
income statement for 20X9 is:
A $900
B $924
C $1,600
D $2,200 (2 marks)

12.3 The preliminary trial balance of Jessie and Co as at 30 September 20X7 included:
Debit Credit
$ $
Trade receivables 90,350
Allowance for receivables (brought forward as at 1 October 20X6) 2,490
Bad and doubtful debt expense 1,985
Further adjustments are to be made as follows:
(i) No entries have been made in respect of cash of $1,320 received from Dome Co whose balance
had been written off last year, and
(ii) At 30 September 20X7 an allowance is required against a balance of $1,950 due from Jed Co as
well as a general allowance of 1.5% of remaining debts.
What is the bad and doubtful debt expense in the income statement? $ (2 marks)
12.4 Gillian
On 31 December 20X4, Gillian’s nominal ledger included a trade receivables balance of $47,900 along
with an allowance for receivables (brought forward as at 1 January 20X4) of $2,551. Of this $537 relates
to a specific customer, the remainder being a general allowance. After a review of trade receivables at the
year end, the following adjustments are to be made:
(1) Debts totalling $1,615 are to be written off as irrecoverable.
(2) No entry has yet been made in the books for $418 cash received on 31 December 20X4 from
David, a customer whose debt was written off during 20X3.
(3) Cash posted to the trade receivables account during the year include $537 from Jim. The amount
due from Jim had been specifically provided against at 31 December 20X3.
(4) Specific allowance is to be made against debts totalling $835 together with a general allowance of
2%.
Required
(a) Write up the relevant ledger accounts for the year ended 31 December 20X4.
(b) Show the relevant extracts from the financial statements.

254
12: QUESTIONS

12.5 Johnson & Co


(1) Johnson & Co had total receivables owing to them at 31 December 20X7 of $9,650. They
included $700 owed by T Black, who had fled the country six months earlier, and various debts
due from K White, totalling $335 and dating back to the years 20X1-20X5. It was decided that the
above debts should be written off.
(2) During 20X8 Johnson & Co made sales on credit of $40,385 and received cash from trade
receivables of $32,050. There were no irrecoverable debts. However, there was some doubt as to
whether a debt of $450 owed by J Green would be met and it was decided to make an allowance
against this specific debt and against 2% of the remaining debts.
(3) During 20X9 credit sales totalled $50,235 and cash of $37,140 was received from trade
receivables. A review of trade receivables at the year end revealed the following:
(i) The amount owed by J Green was now considered irrecoverable and should be written off;
(ii) Other irrecoverable debts totalling $545 were to be written off;
(iii) Allowance was to be made against an amount of $250 owed by P Brown;
(iv) The general allowance was to be maintained at 2% of good debts.
Required
Produce ledger accounts to record the above transactions for the years ended 31 December 20X7, 20X8
and 20X9.

255
12: QUESTIONS

256
Chapter 12: Answers

257
12: ANSWERS

12.1 A

12.2 A
$
Trade receivables balance 50,950
Less specific allowance (950)
50,000

General allowance = 2½% × $50,000 = $1,250


Movement in general allowance is a reduction of $50 ($1,300 – $1,250)
Charge to I/S $
Specific allowance Verulam 950
Less: decrease in general allowance (50)
900

12.3 $1,451

$
Bad and doubtful debts expense per trial balance 1,985
Less: bad debt recovered (1,320)
Add: increase in allowance (W) 786
1,451

$
Allowance c/d – specific 1,950
– general 1.5% × (90,350 – 1,950) 1,326
3,276
Less allowance b/d 2,490
∴ increase 786
12.4 Gillian
(a)
Trade receivables (SOFP)
$ $
31.12.X4 Balance b/d 47,900 31.12.X4 Bad debts 1,615
31.12.X4 Balance c/d 46,285

47,900 47,900

Allowance for receivables (SOFP)


$ $
Irrecoverable & doubtful debts (β) 807 1.1.X4 Balance b/d 2,551
31.12.X4 Balance c/d (W1) 1,744

2,551 2,551

Irrecoverable and doubtful debts expense (I/S)


$ $
Trade receivables 1,615 Bank (bad debt recovered) 418
Allowance for receivables 807
∴ Income statement 390
1,615 1,615

258
12: ANSWERS

Working
Receivables Allowance
$ $
Trade receivables 46,285
Less: specific allowance (835) 835
45,450
General allowance ($45,450 × 2%) 909
1,744
(b) Gillian
Statement of financial position as at 31 December 20X4 (extract)

CURRENT ASSETS $ $
Trade receivables 46,285
Less: allowance for receivables (1,744)
44,541

Income statement for the year ended 31 December 20X4 (extract)


$
Less expenses:
Irrecoverable and doubtful debts expense 390
12.5 Johnson & Co
Trade receivables (SOFP)
$ $
31.12.X7 Balance b/d 9,650 31.12.X7 Irrecoverable 1,035
debts expense
(700 + 335)
31.12.X7 Balance c/d 8,615
9,650 9,650

1.1.X8 Balance b/d 8,615 Bank 32,050


Sales 40,385 31.12.X8 Balance c/d 16,950
49,000 49,000

1.1.X9 Balance b/d 16,950 Bank 37,140


Sales 50,235 31.12.X9 Irrecoverable 995
debts expense
(450 + 545)
31.12.X9 Balance c/d 29,050
67,185 67,185

1.1.YO Balance b/d 29,050

Irrecoverable (bad) & doubtful debts expense (I/S)


$ $
31.12.X7 Trade receivables 1,035 31.12.X7 Income statement 1,035

31.12.X8 Allowance for 780 31.12.X8 Income statement 780


receivables (W1)

31.12.X9 Trade receivables 995


31.12.X9 Allowances for 31.12.X9 Income statement 1,041
receivables 46
1,041 1,041

259
12: ANSWERS

Allowance for receivables (SOFP)


$ $
31.12.X8 Balance b/d 780 31.12.X8 Irrecoverable and 780
doubtful debts
expense
1.1.X9 Balance b/d 780
31.12.X9 Irrecoverable and 46
doubtful debts
expense (β)
31.12.X9 Balance c/d (W2) 826
826 826
Workings
(W1) Allowance for receivables as at 31 December 20X8.
Receivables Allowance
$ $
Trade receivables 16,950
Less: specific allowance (J Green) (450) 450
16,500
General allowance ($16,500 × 2%) 330
780
(W2) Allowance for receivables as at 31 December 20X9.
Receivables Allowance
$ $
Trade receivables 29,050
Less: specific allowance (P Brown) (250) 250
28,800
General allowance required ($28,800 × 2%) 576
826

END OF CHAPTER

260
Provisions
and contingencies 13
Syllabus Guide Detailed Outcomes
Having studied this chapter you will be able to:
• Understand the definition of 'provision', 'contingent liability' and 'contingent asset', distinguish between them and
classify items accordingly.
• Identify and illustrate the different methods of accounting for provisions, contingent liabilities and contingent
assets.
• Calculate provisions and changes in provisions and account for the movement in provisions.
• Report provisions in the final accounts.

Exam Context
Questions on this area are likely to focus on identifying when a provision or contingent liability should be made or
disclosed in the financial statements. You may also be required to calculate a provision. The Pilot Paper included a
question on how a remote contingent liability should be accounted for.

Qualification Context
Your understanding of IAS 37 will be developed at the Fundamentals level paper Financial Reporting (F7) where you are
likely to have to consider whether the provision criteria are satisfied based on more subjective scenarios.

261
13: PROVISIONS AND CONTINGENCIES

Overview

Accounting treatment Recognition criteria

Provisions

Provisions and
contingencies

Contingent liabilities Contingent assets

262
13: PROVISIONS AND CONTINGENCIES

1 IAS 37: Provisions, contingent liabilities and


contingent assets
1.1 Introduction
Before the introduction of IAS 37, there was little guidance on when a provision must and
must not be made.
This caused problems as entities tended to choose to make and then release provisions in
order to smooth out profits, rather than making a provision where they had an obligation to
incur expenditure.
IAS 37 aims to prevent this happening in the future.

2 Provisions
2.1 Definition
A provision is a liability of uncertain timing or amount.

2.2 Recognition
A provision should only be recognised (ie. included in the financial statements) when:
(a) An entity has a present obligation (legal or constructive) as a result of a past event;
(b) It is probable that an outflow of economic resources will be required to settle the
obligation; and
(c) A reliable estimate can be made of the amount of the obligation.
Unless all three conditions are met, no provision can be recognised.

2.3 Legal obligation


A legal obligation usually arises out of a contract.
Illustration
Grass Co sells lawnmowers and offers a one-year warranty on all models.
Once Grass Co sells a lawnmower (the past event) it has a legal obligation to repair any
defects according to the warranty agreement.
It should therefore make an estimate of the probable costs of repair and make a provision
for this amount in its financial statements.

2.4 Constructive obligation


A constructive obligation arises through past behaviour and actions where the entity has
raised a valid expectation that it will carry out a particular action.
Illustration
Seed Co also sells lawnmowers. It does not offer a warranty on its products; however it has
a reputation for making free reasonable repairs to lawnmowers bought from the business.
Customers buying from Seed Co all expect to receive this benefit.

263
13: PROVISIONS AND CONTINGENCIES

Here no warranty is offered and so Seed Co does not have a legal obligation. Its past
actions however have created a constructive obligation. It should also therefore make a
provision for the probable costs of repairs.

2.5 Accounting treatment


The provision represents both a cost to the business and a potential liability:
Dr Expense (I/S)
Cr Provision (SOFP)
The required provision will be reviewed at each year end and increased or decreased as
necessary.
To increase a provision:
Dr Expense (I/S)
Cr Provision (SOFP)
To decrease a provision:
Dr Provision (SOFP)
Cr Expense (I/S)

Lecture example 1 Preparation question

Grass Co is reviewing its warranty obligations. Based on sales during 20X7 it has established that
if all lawnmowers sold required minor repairs this would cost $1m whereas if major repairs were
required this would cost $6m.
Grass Co expects that 75% of lawnmowers will have no faults, 20% will need minor repairs and
5% major repairs.
Required
(a) What provision should be made in 20X7 and what accounting entry is needed to record it?
(b) What entry should be made in 20X8 assuming the provision required then is $0.75m?
(c) What entry should be made in 20X9 assuming the provision required then is $0.3m?

Solution

264
13: PROVISIONS AND CONTINGENCIES

3 Contingent liabilities
3.1 A contingent liability is an uncertain liability that does not meet the three criteria for
recognising a provision.
IAS 37 defines a contingent liability as the following:
(a) A possible obligation that arises from past events and whose existence will be
confirmed only the occurrence or non-occurrence of one or more uncertain future
event not wholly within the control of the entity; or
(b) A present obligation that arises from past events but is not recognised because:
(i) it is not probable that an outflow of economic resources will be required to
settle the obligation; or
(ii) the amount of the obligation cannot be measured with sufficient reliability.
Contingent liabilities should be disclosed in the notes unless probability of an outflow of
resources embodying economic benefits is remote.

Illustrative example
3.2 Company A has entered into an agreement to act as guarantor on a bank loan taken out by
Mr Smith. Mr Smith is a financially secure individual, and the directors are of the opinion that
the chances of him defaulting on the loan are slim.
How should company A account for this guarantee?

Solution
3.3 Company A has a present obligation (it is legally obliged to honour the guarantee).
However, as the likelihood of Company A having to pay out under the guarantee is not
probable then no provision for the liability should be made. Instead, the guarantee should be
disclosed in the notes as a contingent liability (unless considered remote, in which case it
should be ignored altogether).

265
13: PROVISIONS AND CONTINGENCIES

3.4 Decision Tree

4 Contingent assets
4.1 A possible asset that arises from past events and whose existence will be confirmed only by
the occurrence or non-occurrence of one or more uncertain future events not wholly within
the control of the entity.
Contingent assets should be disclosed in the notes where an inflow of economic benefits is
probable, otherwise they should be ignored.
If the probability of an inflow of economic benefits is virtually certain then the asset is not a
contingent asset and should be recognised in the financial statements.

266
13: PROVISIONS AND CONTINGENCIES

5 Chapter summary
Section Topic Summary
Quick Quiz
2 Provisions A provision should only be made in the financial
statements when an entity has a present obligation to
incur expenditure. It must also be more likely than not
that the expenditure will be incurred and a reliable
estimate of the amount is known.
3 Contingent liabilities A contingent liability should be disclosed where the
criteria for making a provision are not met, but where
there is either a possible obligation or a present
obligation but it is only possible that the expenditure
will be incurred.
4 Contingent assets Contingent assets should only be included in the
financial statements if it is certain to be received and
should be disclosed if probable.

6 Double Entry Summary for Chapter 13


6.1 Adjustment to create or increase a provision:

Dr Expense (I/S)
Cr Provision (SOFP)

6.2 Adjustment to decrease a provision:

Dr Provision (SOFP)
Cr Expense (I/S)

267
13: PROVISIONS AND CONTINGENCIES

Chapter Summary • Entity has a present obligation as a result of a past


event
• It is probable that an outflow of economic resources
• Recognise in financial statements:
will be required to settle the obligation
Dr Expense (I/S)
Cr Provision (SOFP) • A reliable estimate can be made of the amount

Accounting treatment Recognition criteria

Provisions

'A liability of uncertain timing or amount'

Provisions and
contingencies

Contingent liabilities Contingent assets

'An uncertain liability that does not meet the 'A possible asset that arises from past events
three criteria for recognising a provision' and whose existence will be confirmed by one or
• Possible obligation more uncertain future events not wholly within
• Present obligation the control of the entity'.
- which is not probable • Disclose where probable
- where the amount cannot be • Recognise if virtually certain
measured reliably
• Disclose in a note to the financial
statements

268
Chapter 13: Questions

269
13: QUESTIONS

13.1 H Co is currently in the middle of a protracted lawsuit which it is vigorously defending. The directors are
reasonably confident that the action will not be successful but are aware that the opposite outcome is a
possibility. It is difficult to quantify any potential damages, but the directors feel they are unlikely to
exceed $50,000.
How should the above item be treated in the financial statements?
A Provision
B Contingent liability
C Contingent asset (1 mark)
13.2 How should a contingent liability and a probable contingent asset be accounted for?
A Probable contingent assets and contingent liabilities should be disclosed in the financial
statements.
B Probable contingent assets must always be accrued and contingent liabilities must always be
disclosed in the financial statements.
C Contingent liabilities must always be either accrued or disclosed and probable contingent assets
must always be disclosed in the financial statements.
D Contingent liabilities must always be provided for and probable contingent assets must be
disclosed in the financial statements.
(2 marks)

270
Chapter 13: Answers

271
13: ANSWERS

13.1 B
13.2 A

END OF CHAPTER

272
Control accounts 14
Syllabus Guide Detailed Outcomes
Having studied this chapter you will be able to:
• Understand the purpose of control accounts for accounts receivable and accounts payable.
• Understand how control accounts relate to the double entry system.
• Prepare ledger control accounts from given information.
• Perform basic control account reconciliations for accounts receivable and accounts payable and identify errors
which would be highlighted by performing them.
• Identify and correct errors in control accounts and ledger accounts.
• Account for discounts allowed and discounts received.
• Account for contras between trade receivables and trade payables.

Exam Context
Questions on this topic are likely to require you to correct the closing balance on a receivables or payables control
account including items such as contras and discounts. You may also be required to calculate receivables/payables
balances where goods are sold/bought with trade and/or settlement discounts.

Qualification Context
This chapter covers topics which are only examined in Financial Accounting.

273
14: CONTROL ACCOUNTS

Overview
Reconciliations

Receivables ledger control account Receivables ledger


Payables ledger control account Payables ledger

Control accounts

Contra entries Returns, credit notes, refunds Discounts


and over payments allowed and received

Trade discounts Settlement discounts

Sales tax considerations

274
14: CONTROL ACCOUNTS

1 Recap
1.1 In Chapters 4 and 5 we saw how a business' transactions were categorised in the books of
prime entry. The totals of these were then posted using double entry to the nominal ledger
to give a summary of the information.

1.2 For example, credit sales:

1.3 The nominal ledger contains three ledger accounts which are affected when a business sells
on credit:
(a) Sales
(b) Bank
(c) Trade receivables – this shows the total amount owed by all customers at a
particular point in time.
– it is also called the receivables ledger control account
(RLCA)

1.4 In order to chase overdue debts however a business must know how much each customer
owes at a particular time.
This balance could be determined by going back into the detail of the books of prime entry
and extracting the information for each customer.
This is a very time consuming process and so instead a memorandum ledger is
maintained for each individual customer showing invoices raised, cash received and
therefore the amount owed to the business.
This memorandum ledger is called a receivables ledger.

1.5 The reverse is true when a business buys on credit.

275
14: CONTROL ACCOUNTS

Terminology
1.6 In the nominal ledger:
• Receivables ledger control account (trade receivables/RLCA):
total owed by all credit customers.
• Payables ledger control account (trade payables/PLCA):
total owed to all credit suppliers.

Memorandum ledgers:
• Receivables ledger:
balance owed by each individual credit customer
• Payables ledger:
balance owed to each individual credit supplier

2 The flow of information


2.1
Sales Receipt Payment
to Purchase
Invoice from
suppliers Invoice
customers

Memo SDB Cash book PDB Memo


Receivables Payables
Ledger Ledger

Customer A Supplier X

Customer B PLCA Supplier Y


Nominal Ledger
Trade payables
Customer C Supplier Z

Bank

Sales Purchases

Trial Balance

Financial Statements

276
14: CONTROL ACCOUNTS

2.2 The information in the receivables ledger control account (RLCA) and receivables ledger
(RL) is posted from the same source documents.
Therefore

the balance on the RLCA should equal the sum of all balances from the RL

Similarly

the balance on the PLCA should equal the sum of all balances from the PL

2.3 If the balances do not agree then an error has been made. This will be identified through a
control account reconciliation (Section 5).

Lecture example 1 Preparation question

A Co has the following information:


10 January 20X6
Sells $150 of goods to customer A
Sells $200 of goods to customer B
15 January 20X6
A Co purchases $100 of goods from supplier Y
A Co purchases $1,300 of goods from supplier Z
21 January 20X6
A Co receives full payment from customer B and this money is used to pay supplier Y.
Required
(1) Record the above transactions in the books of prime entry and the memorandum ledgers.
(2) Post the totals from the BOPE to the nominal ledger.
(3) Balance off nominal ledger accounts.
(4) Reconcile the memorandum ledgers to the control accounts.

Solution
(1) Books of prime entry
Sales day book
Date Customer Amount

277
14: CONTROL ACCOUNTS

Purchase day book


Date Supplier Amount

Cash receipts book


Date Narrative Total Sales Receivables

Cash payments book


Date Narrative Total Purchases Payables

Memorandum ledgers
Receivables ledger
Customer A Customer B

Payables ledger
Supplier Y Supplier Z

278
14: CONTROL ACCOUNTS

(2) & (3) Nominal ledger


RLCA (SOFP) PLCA (SOFP)

Bank (SOFP) Sales (I/S)

Purchases (I/S)

(4) Reconciliation
Balance per list of balances
$
Receivables ledger
Customer A
Customer B

Balance per RLCA

Balance per list of balances


$
Payables ledger
Supplier Y
Supplier Z

Balance per PLCA

279
14: CONTROL ACCOUNTS

3 Other entries
A business must ensure that any transaction recorded in the receivables ledger control
account or the payables ledger control account is also reflected in the memorandum
ledgers.

Contra entries
3.1 Sometimes a business may have a customer which also supplies the business with goods.
Illustration:
P Co is a printing business which sells stationery to F Co, a florist. F Co supplies P Co with
flowers and plants for its offices.
During October, P Co sells stationery worth $200 to F Co and F Co delivers flowers and
plants to P Co worth $70.
P Co has the following amounts in its books:
Receivables: $200
Payables: $70
The two businesses agree to offset the balances receivable and payable via a contra.
The contra will be for the lower of the two amounts: $70. This will decrease both
receivables and payables by $70 and the remaining $130 can then be paid in cash.

3.2 A contra entry is always recorded as:


Dr PLCA
Cr RLCA
This will reduce both receivables and payables.

3.3 Note that the memorandum ledgers will also need to be updated for the contra entry.

Returns, credit notes and refunds


3.4 Sometimes when a business has made a sale, the customer will return the goods.

3.5 Steps:
(1) Goods are sold to the customer for $250:
Dr RLCA $250
Cr Sales $250
(2) Customer pays for goods:
Dr Bank $250
Cr RLCA $250
At this point the balance on the receivables ledger control account is nil.

280
14: CONTROL ACCOUNTS

(3) Customer returns the goods and is issued with a credit note:
Dr Sales (returns) $250
Cr RLCA $250
This entry reverses the original sale.
The receivables ledger control account will show a credit balance reflecting that the
business owes money to the customer. This could be offset against future purchases
or the customer may request a refund.
(4) The business refunds the customer:
Dr RLCA $250
Cr Bank $250
Once again the balance on the receivables ledger control account is nil.

3.6 Again, the memorandum ledgers must also be updated.

Over payment
3.7 If a customer pays too much to settle an invoice or pays an invoice twice the business will
owe the excess to the customer.
This may be held and treated like a credit note or the monies refunded to the customer.

Interest on overdue accounts


3.8 If a customer is late in settling their account then an entity may decide to charge them
interest.
This will increase the amount they owe and will be shown as interest receivable in the
income statement.
Interest on overdue accounts is recorded using the following journal:

Dr RLCA
Cr Interest receivable (I/S)

4 Discounts
4.1 There are two types of discounts:
(a) Trade discounts
(i) given at the time of the sale/purchase, they reduce the selling price as an
inducement to purchase;
(ii) usually for regular customers or bulk buyers.
(b) Settlement discounts
(i) offered, but not necessarily taken, as an inducement to settle a debt early;
(ii) eg. 5% discount if settled within 14 days.

281
14: CONTROL ACCOUNTS

Terminology
4.2 Discounts allowed: offered by the business to their customer.
Discounts received: received by a business from their supplier.

Discounts allowed
4.3 Accounting treatment
Sales are recorded net of (i.e. after) trade discounts but inclusive of (i.e. before) settlement
discounts.
Therefore trade discounts never appear in the financial statements.
Settlement discounts allowed are recorded as discounts allowed and are shown as an
expense in the income statement:
Dr Discounts allowed (I/S)
Cr RLCA (SOFP)

Lecture example 2 Preparation question

(a) On 1 January 20X7 a business made a sale on credit for $12,000. A trade discount of
$2,000 was available with a further 10% settlement discount if payment were made within
10 days.
Required
Record the initial sale.

Solution

The initial sale would be recorded as:


Sales (I/S) RLCA (SOFP)

(b) On 4.1.X7, the customer pays for the goods taking advantage of the settlement discount.
The discount will be 10% of sales value.
Required
Record the full settlement of the amount owed.

282
14: CONTROL ACCOUNTS

Solution

Bank (SOFP) Discounts allowed (I/S)

(c) Required
What would your answer be to part (b) if the settlement discount were not taken?

Solution
Bank (SOFP) RLCA (SOFP)

Discounts received
4.4 Accounting treatment
Purchases are recorded net of trade discounts but inclusive of settlement discounts.
Again trade discounts never appear in the financial statements.
Settlement discounts received are recorded as discounts received and are shown as
sundry income in the income statement.
Dr PLCA (SOFP)
Cr Discounts received (I/S)

283
14: CONTROL ACCOUNTS

Lecture example 3 Preparation question

Ryan Co purchases goods worth $5,000 from Austin Co. Ryan Co will receive a 5% settlement
discount if the goods are paid for within seven days. Ryan Co has every intention of taking
advantage of the settlement discount.
Required
(a) Show the initial recording of the purchase.
(b) Record the payment for the goods assuming Ryan pays within seven days.
(c) Record the payment for the goods if payment is made after seven days.

Solution

284
14: CONTROL ACCOUNTS

Sales tax and discounts


4.5 Sales tax is calculated on the amount after all discounts, regardless of whether the
discount is taken or not.

Lecture example 4 Exam standard for 2 marks

Brick buys goods with a list price of $50,000 from Cement. Brick receives a trade discount of 12%
from Cement and a further discount of 4% if payment is made within 10 days. Sales tax is at 15%.
Required
What amount should Brick show in Cement's payables ledger to record this purchase?
A $48,576
B $50,336
C $50,600
D $57,500

Solution

5 Control account reconciliations


5.1 As mentioned in Section 2 if we add up the balances in the receivables and payables
ledgers, they should agree to the balances per the RLCA and PLCA.
If not, an error must have occurred at some point in the system.
The easiest way to identify the error is to perform a reconciliation between the two amounts.

285
14: CONTROL ACCOUNTS

5.2 Proforma control account reconciliation


RLCA
$ $
Balance b/d X Transposition error in posting X
Sales day book undercast X
Sales omitted from SDB X Balance c/d X

X X

Balance b/d X

Reconciliation Statement
$ $ $
+ –
Total per listing of receivables ledger X
balances

Adjustments
Balance omitted X
Credit balance listed as debit (2X)
X X X

Balance as per adjusted control account X

Lecture example 5 Technique demosntration

(a) Required
Post the following transactions to and balance off the receivables ledger control account.
(1) Opening balance $614,000
(2) Credit sales made during the month $302,600
(3) Receipts from customers $311,000
(4) Bad debts were written off $32,000
(5) Discounts allowed for prompt payment $3,400
(6) Contras against amounts due to suppliers in payables ledger $8,650
(b) The receivables ledger list of balances totals to $563,900.
You have found the following errors:
(i) The total of the sales day book was undercast by $3,600.
(ii) A credit balance of $450 was included in the list of balances as a debit.
(iii) A customer balance of $2,150 was left out when the receivables ledger list of
balances was totalled.
Required
Reconcile the receivables ledger control account to the receivables ledger list of balances.

286
14: CONTROL ACCOUNTS

Solution

287
14: CONTROL ACCOUNTS

6 Chapter summary
Section Topic Summary

1 Recap The balance of the receivables ledger control account


and the payables ledger control account in the nominal
ledger show the total owed by all credit customers and
due to all credit suppliers.
The purpose of the memorandum ledgers is to show
the balance on each individual customer or supplier
account.
2 The flow of information Given that the nominal ledger and the memorandum
ledgers are updated from the same source
documentation, at any point in time the balance on the
control accounts should equal the total of all the
balances in the memorandum ledgers.
Where the two balances are not the same an error must
have arisen and a reconciliation should be performed to
identify the errors (section 5).
3 Other entries If an entity has a customer is also a supplier the two
parties may choose to settle their accounts by making a
contra entry. The contra is always for the lower of the
two balances.
If a customer returns goods having paid for them or
overpays for goods then the entity will owe money back
to that customer and the customer will have a credit
balance on their account.
If a customer is late in settling their account the entity
may decide to charge them interest on the overdue
account. This will increase the balance owed.
4 Discounts Sometimes a business may offer discounts to attract
custom. There are two types of discounts: trade
discounts and settlement discounts.
Sales and purchases are recorded after trade
discounts but before settlement discounts.
Sales tax is calculated on the amount after all
discounts, regardless of whether the discount is taken
or not.
5 Control account As detailed in section 2 if the balance on the control
reconciliations account does not agree to the total of all the balances
on the memorandum ledger then an error must have
occurred and a reconciliation will need to be carried out
to identify the differences.

288
14: CONTROL ACCOUNTS

7 Double Entry Summary for Chapter 14


7.1 Contra entry adjustment:

Dr Payables ledger control account (SOFP)


Cr Receivables ledger control account (SOFP)

7.2 Adjustment to record settlement discounts allowed to customers:

Dr Discounts allowed (I/S)


Cr Receivables ledger control account (SOFP)

7.3 Adjustment to record settlement discounts received from suppliers:

Dr Payables ledger control account (SOFP)


Cr Discounts received (I/S)

289
14: CONTROL ACCOUNTS

Chapter Summary • The RLCA and the RL and the PLCA and the PL are
showing the same information and so the balances should
reconcile

Reconciliations

Receivables ledger control account Receivables ledger


Payables ledger control account Payables ledger

• RLCA: The total owed by all • RL: a list of the amounts owed by
credit customers at a particular each individual credit customer at
point in time. a particular point in time
• PLCA: Total owed to all credit • PL: a list of the amounts owed to
suppliers at a particular point in each individual credit supplier at
time a particular point in time
Control accounts

Contra entries Returns, credit notes, refunds Discounts


and over payments allowed and received
'Where a business has a customer which
is also a supplier' • If a customer returns goods having already • Discounts allowed are offered by a
• A contra will always be for the lower of paid for them or over pays an invoice they business to their customer (an expense)
the two amounts and will always will show a credit balance on their account • Discounts received are received by a
reduce both receivable and payables: • The business may issue the customer with business from their supplier (sundry
Dr PLCA a credit note which they can use to pay for income)
Cr RLCA future purchases or the customer may
• The memorandum ledgers must also request a refund
be updated for the contra entry • Both the control accounts and the
memorandum ledgers must be updated for
these entries

Trade discounts Settlement discounts

• Offered as an incentive to
• Given at the time of sale/purchase settle a debt early
• For example: bulk buying discounts • For example: 3% discount if
• Never appear in the financial settled within 10 days
statements • May or may not be taken
Sales tax considerations • Sales and purchases are
recorded after trade
discounts but before
• Sales tax is calculated after all discounts, regardless of settlement discounts
whether they are taken or not
• The rate of sales tax will be provided in the exam question

290
Chapter 14: Questions

291
14: QUESTIONS

Data for Questions 14.1 and 14.2


Womble & Sons have an accounting year ended 31 July 20X8. At that date the balance on the receivables ledger
control account was $130,000, but the total of the individual accounts in the receivables ledger came to
$127,240.
Upon investigation the following facts were discovered:
(i) The sales day book total for week 22 had been overcast by $600.
(ii) A credit balance of $420 on Orinoco’s account had been incorrectly treated as a debit entry when listing
the receivables ledger.
(iii) A contra of $3,000 has been entered in Bungo’s account in the receivables ledger but no other entry had
been made.

14.1 The adjusted balance on the receivables ledger control account is:
A $125,560
B $126,400
C $127,240
D $129,400 (2 marks)

14.2 The adjusted balance on the receivables ledger is:


A $125,560
B $126,400
C $127,240
D $129,400 (2 marks)

14.3 A page of the sales day book is undercast by $250. The journal necessary to correct the error is:
A Debit trade receivables $500, Credit sales $500
B Debit sales $500, Credit trade receivables $500
C Debit trade receivables $250, Credit sales $250
D Debit sales $250, Credit trade receivables $250 (2 marks)

14.4 Winn Co has opening trade payables of $24,183 and closing trade payables of $34,665. Purchases for
the period totalled $254,192 ($31,590 relating to cash purchases).
What were total payments recorded in the payables ledger for the period? $ (2 marks)

14.5 The double entry to record a discount granted by a supplier is:


A Debit trade payables, Credit discounts allowed
B Debit trade payables, Credit discounts received
C Debit discounts received, Credit trade payables
D Debit trade payables, Credit purchases (2 marks)

292
14: QUESTIONS

14.6 The following receivables ledger reconciliation has been prepared by the bookkeeper of Julian Co as at
31 October 20X7:
$
Total per listing of receivables ledger balances 26,170
Debit balance omitted 1,740
Credit listed as debit (1,220)
Unexplained difference 300
Balance per control account 26,990
Which of the following errors could have produced the ‘unexplained difference’?
A A refund of $300 was omitted from the receivables ledger.
B The sales day book for October was undercast by $300.
C The trade receivables column of the cash receipts book was overcast by $300.
D A payables ledger contra of $300 was not entered in the memorandum records. (2 marks)

14.7 Justin has attempted to write up his own nominal ledger but is very confused about debits and credits. He
realises he has made some mistakes and has asked you to correct the following receivables ledger
control account:
Receivables ledger control account
$ $
Balance b/d 12,460 Cash sales 4,430
Sales on credit 15,520 Cheques from credit customers 11,650
Purchase ledger contra 1,600 Discounts allowed 890
Balance c/d 12,610
29,580 29,580

The opening balance is correct. What should the closing balance be?
A $9,410
B $13,840
C $15,620
D $17,040 (2 marks)

14.8 Which of the following is not a valid reason for a credit balance on a customer's account in the
receivables ledger?
A Over payment
B Cheque dishonoured by bank
C Returned goods credited to account (1 mark)

293
14: QUESTIONS

14.9 During April a company receives an invoice for $12,000 relating to goods bought on credit. These
purchases qualify for a 5% trade discount which has not yet been taken into account. The company also
sells goods with a list price of $20,000. A 6% trade discount is to be offered on these goods. Sales tax is
applicable to all items and is at 15%. Sales tax is not included in the above amounts. If there is no
opening balance on the sales tax account at the beginning of April, what is the closing balance at the end
of April?
A $1,110 Cr
B $1,110 Dr
C $1,200 Cr
D $1,200 Dr (2 marks)

14.10 The following transactions were recorded in a company’s books during one week of its trading year:
$
Trade purchases (at list price) 4,500
Sales on credit (at list price) 6,000
Purchase of a van 10,460
A trade discount of $300 was given on the sales. All figures are given exclusive of sales tax at 15%.
If the balance on the sales tax account was $2,165 credit at the beginning of the week, what is the
balance at the end of the week? $ (2 marks)
14.11 Thomas
Thomas is a sole trader. He has been reading a book on basic bookkeeping but his grasp of the subject is
weak.
He has produced the following receivables ledger control account but is not sure whether his closing
balance is correct.
Receivables ledger control account
$ $
Balance b/d 1.1.X6 12,240 Cheques received from customers 74,730
Discounts allowed 2,165 Cheques dishonoured by customers 425
Cash paid to customers with Irrecoverable debts 470
credit balances 180 Allowance for receivables 1,470
Sales 71,250 Cash received from customers 870
Returns inwards 2,250
Purchase ledger contras 230 Balance c/d 31.12.X6 10,350
88,315 88,315

Balance b/d 1.1.X7 10,350


Required
Produce a corrected receivables ledger control account.

294
14: QUESTIONS

14.12 Duff
On 31 December 20X7 the balance on Duff’s receivables ledger control account was $1,070, but the
receivables ledger balances totalled only $890.
You ascertain the following:
(1) The sales day book was overcast by $100 on 1 December 20X7.
(2) Receivables ledger balances totalling $70 had been omitted from the list.
(3) A contra entry of $20 had been made between the payables ledger and receivables ledger
accounts of Jones & Co, but no other entry had been made.
(4) The only posting made in respect of sales on 15 December 20X7, $50 in total, had been to
individual ledger accounts.
(5) $60 worth of goods had been returned by Smith Co in November; this had been recorded only in
the control account.
(6) The ledger account balance of Davis & Co had been listed as $90, but was in fact $190.
Required
Prepare a reconciliation between the receivables ledger control account and the receivables ledger.

295
14: QUESTIONS

296
Chapter 14: Answers

297
14: ANSWERS

14.1 B Receivables ledger control account

$ $
Balance b/d 130,000 SDB overcast 600
Contra 3,000
∴balance c/d 126,400
130,000 130,000

14.2 B Receivables Ledger $

Balance per list of balances 127,240


Credit balance treated as a debit (840)
(2 × $420)
126,400

14.3 C

14.4 $212,120 Trade payables

$ $
Bal b/d 24,183
∴ Payments 212,120
Bal c/d 34,665 Purchases
($254,192 – $31,590) 222,602

246,785 246,785

Bal b/d 34,665

14.5 B

14.6 A B – Day book total has no effect on the receivables ledger, where individual invoice
amounts will be entered.
C – Cash book total has no effect on receivables ledger.

D – If a contra had been omitted, the receivables ledger total would have to be reduced by
$300.

14.7 B Receivables ledger control account

$ $
Balance b/d 12,460 Purchase ledger contra 1,600
Sales on credit 15,520 Cheques from credit customers 11,650
Discounts allowed 890
Balance c/d 13,840
27,980 27,980

Note: Cash sales are not recorded in the control account

14.8 B This would leave a debit balance as the original debt would be reinstated.

298
14: ANSWERS

14.9 A $
Input sales tax
$12,000 × 95% × 15% 1,710

Output sales tax


$20,000 × 94% × 15% (2,820)

Closing balance on sales tax account at end of April (1,110) Cr

14.10 $776 Cr

$2,165 + [15% × ($6,000 – $300)] – [15% × $4,500] – [15% × $10,460] = $776


14.11 Thomas
Receivables ledger control account

$ $
Balance b/d 12,240
Cash – cheque dishonoured 425 Cheques 74,730
Cash – credit balances 180 Irrecoverable debts 470
Sales 71,250 Cash received 870
Discounts allowed 2,165
Returns inwards 2,250
Purchase ledger contras 230
Balance c/d 3,380
84,095 84,095
14.12 Duff
Receivables ledger control account

$ $
Balance b/d 1,070 SDB Overcast (1) 100
Sales 15.12.X7 (4) 50 Purchase ledger contra (3) 20
Amended balance c/d 1,000
1,120 1,120

Balance b/d 1,000

Receivables ledger reconciliation statement


$
Total balance per receivables ledger 890
$ $
Adjustments: + –
Balances omitted (2) 70
Goods returned (5) 60
Balance understated (6) 100
170 60
110
Amended total 1,000

299
14: ANSWERS

END OF CHAPTER

300
Bank reconciliations 15
Syllabus Guide Detailed Outcomes
Having studied this chapter you will be able to:
• Understand the purpose of bank reconciliations.
• Identify the main reasons for differences between the cash book and the bank statement.
• Correct cash book errors and/or omissions.
• Prepare bank reconciliation statements and identify the bank balance to be reported in the final accounts.
• Derive bank statement and cash book balances from given information.

Exam Context
Exam questions are likely to ask you to perform calculations to correct a bank reconciliation. Alternatively they may ask
you to state whether differences between the cash book and the bank statement should be adjusted in the cash book or
in the reconciliation statement.

Qualification Context
This chapter covers a topic which is only examined in Paper F3.

301
15: BANK RECONCILIATIONS

Overview

Bank reconciliations

Cash book balance Bank statement balance

Differences

Timing differences Errors by the business Errors by the bank

302
15: BANK RECONCILIATIONS

1 Introduction
1.1 This chapter is designed to enable you to explain and apply the approach to identifying and
correcting errors through the use of bank reconciliations.

1.2 The cash book is used to record the detailed transactions of receipts and payments into and
out of the bank account. These are then posted to the nominal ledger periodically using
double entry. At the end of each accounting period, the balance on the cash book should
equal the balance in the nominal ledger cash account.

1.3 Bank statements provide an independent record of the balance on the bank account but this
balance is unlikely to agree exactly to the cash book balance – therefore a reconciliation is
required.

Differences between the cash book balance and the bank statement
1.4 Differences essentially occur for three reasons:
(a) Timing differences:
(i) unrecorded lodgements (money paid into the bank by the business but not
yet appearing as a receipt on bank statement)
(ii) outstanding/unpresented cheques (cheques paid out by business which
have not yet appeared on bank statement).
(b) Errors by the business (i.e. in the cash book):
(i) omissions, such as:
standing orders
direct debits
bank charges
interest
(ii) transposition errors
(iii) casting errors
(c) Errors by the bank.

A word of warning
1.5
In the books of the business:
POSITIVE BANK BALANCE = ASSET = DEBIT
NEGATIVE BANK BALANCE (OVERDRAFT) = LIABILITY = CREDIT
But from the bank’s point of view:
POSITIVE BALANCE = LIABILITY = CREDIT (the bank owes you your money)
NEGATIVE BALANCE (OVERDRAFT) = ASSET = DEBIT
(you owe the bank ∴ this is an asset for the bank)

303
15: BANK RECONCILIATIONS

2 Preparing a bank reconciliation


Procedures
2.1 (a) Compare the bank statement to the cash account and tick off all items which agree.
(b) Remaining items must represent timing differences or errors – decide which!

Example of how to set out a bank reconciliation


2.2
Cash account
$ $
Balance b/d X Dishonoured cheque X
Bank charges X
Standing orders X
Under cast error in balance b/d X Direct debits X
Balance c/d X
X X

$
Balance per bank statement X
plus unrecorded lodgements X
less outstanding cheques (X)
plus/less bank errors X/(X)

Balance per adjusted cash account X

Practical tips
2.3 (a) On reconciliation, put overdrafts and payments in brackets.
(b) It is the corrected cash account balance which is shown on the statement of financial
position. This figure will be the recalculated 'Balance c/d' on the cash account (or the
total at the end of the reconciliation statement – which should be identical!).

304
15: BANK RECONCILIATIONS

Lecture example 1 Preparation question

The cash account of Graham showed a debit balance of $204 on 31 March 20X8. A comparison
with the bank statements revealed the following.
$
(1) Cheques drawn but not presented 3,168
(2) Amounts paid into the bank but not credited 723
(3) Entries in the bank statements not recorded in the cash account
(i) Standing order payments 35
(ii) Interest on bank deposit account 18
(iii) Bank charges 14
(4) Balance on the bank statement at 31 March 20X8 2,618
Required
Make any necessary adjustments to the cash book balance and complete the bank reconciliation
statement as at 31 March 20X8.

Solution
Adjustment of cash book balance
Cash account
$ $

Bank reconciliation statement

$
Balance per bank statement 31 March 20X8
Unrecorded lodgements
Outstanding cheques

Balance per cash account at 31 March 20X8

305
15: BANK RECONCILIATIONS

Lecture example 2 Exam standard for 2 marks

Whilst preparing a bank reconciliation statement at 31 December. The following items caused a
difference between the bank statement balance and the cash book balance.
(1) Bank interest charged to the account in error
(2) Direct debit for $500 for insurance
(3) Bank charges of $70
(4) Cheque paid to a supplier on 29 December
(5) Receipt from a trade receivable by electronic transfer
Required
Which of these items will result in an adjustment to the balance per the bank statement?
A 2, 3, and 5
B 1 and 4
C 1, 4, and 5
D 1, 3 and 5

Solution

306
15: BANK RECONCILIATIONS

3 Chapter summary
Quick Quiz Section Topic Summary
1 Introduction A business maintains a cash book to tell it how much
cash it has at a particular point in time. It should
reconcile this balance to the bank statement in order to
ensure the cash book information is accurate.
Differences between the cash book balance and the
bank statement balance will arise for three reasons:
timing differences, errors by the business and
errors by the bank.
2 Preparing a bank The bank reconciliation is produced by checking all of
reconciliation the items on the bank statement to the cash book to
ensure that they have all been recorded.
Any items not in the cash book will then need to be
recorded and the cash book updated.
The balance per the bank statement must then be
adjusted for any timing differences (unrecorded
lodgements and outstanding cheques) or errors by the
bank.

307
15: BANK RECONCILIATIONS

Chapter Summary

Bank reconciliations

Cash book balance Bank statement balance

• Business's record of the amount • Bank's record of the amount of


of cash held by the business at cash held by the business at
any point in time any point in time

Differences

Timing differences Errors by the business Errors by the bank

• Items shown in the cash book • Items on the bank statement which • Examples:
but not currently on the bank have been omitted from the cash - cheque incorrectly debited to the
statement book business's account
• Examples: • Examples: - lodgement incorrectly credited to
- unrecorded lodgements - bank charges the business's account
- outstanding cheques - direct debits • Adjust bank statement balance
• Adjust bank statement balance • Adjust in the cash book

308
Chapter 15: Questions

309
15: QUESTIONS

Data for Questions 15.1 and 15.2


In the books of Ted Co the bank account shows a balance overdrawn of $6,530 as at 31 December 20X8. On
comparing the bank statements with the cash book the following items are discovered:
(i) Bank charges of $100 and overdraft interest of $50 have been omitted.
(ii) Cheques received from customers totalling $1,900 have not yet been cleared by the bank.
(iii) Cheques drawn in favour of suppliers amounting to $2,300 are outstanding at the year end.
(iv) A credit transfer from a customer of $2,000 was not recorded.
(v) A direct debit to a supplier of $1,000 was omitted.

15.1 What figure will be shown in the statement of financial position as at 31 December 20X8 for ‘bank
overdraft’?
A $5,480
B $5,680
C $6,130
D $7,380 (2 marks)

15.2 Assuming that the above items are all that is required to reconcile the cash book balance to the balance
per the bank statement, what balance did the bank statement show as at 31 December 20X8?
A $5,280 overdrawn
B $6,080 overdrawn
C $7,780 overdrawn
D $8,580 overdrawn (2 marks)
15.3 Rectify
A summary of the cash book of Rectify Co for the year to 31 May 20X5 is as follows:
Cash Book
$ $
Opening balance b/d 805 Payments 146,203
Receipts 145,720 Closing balance c/d 322
146,525 146,525
After some investigation of the cash book and vouchers you discover that:
(1) bank charges of $143 shown on the bank statement have not yet been entered in the cash book;
(2) a cheque drawn for $98 has been entered in the cash book as $89, and another drawn at $230
has been entered as a receipt;
(3) a cheque received from a customer for $180 has been returned by the bank marked ‘refer to
drawer’, but it has not yet been written back in the cash book;
(4) an error of transposition has occurred in that the opening balance of the cash book should have
been brought down at $850;
(5) cheques paid to suppliers totalling $630 have not yet been presented at the bank, whilst payments
in to the bank of $580 on 31 May 20X5 have not yet been credited to the company’s account;
(6) a cheque for $82 has been debited to the company’s account in error by the bank;
(7) the company owes $430 to the electricity board;
(8) standing orders appearing on the bank statement have not yet been entered in the cash book:
(i) interest for the half year to 31 March on a loan of $20,000 at 11% pa;
(ii) hire purchase repayments on the managing director’s car – 12 months at $55 per month;
(iii) dividend received on a trade investment – $1,147;

310
15: QUESTIONS

(9) a page of the receipts side of the cash book has been undercast by $200;
(10) the bank statement shows a balance overdrawn of $870.
Required
Prepare a bank reconciliation as at 31 May 20X5.

311
15: QUESTIONS

312
Chapter 15: Answers

313
15: ANSWERS

15.1 B $
Unadjusted cash book balance (6,530)

Corrections to cash book


(i) interest (100 + 50) (150)
(iv) unrecorded cash received 2,000
(v) payment to supplier omitted (1,000)
Adjusted cash book balance (5,680)

15.2 A $
Adjusted cash book balance (5,680)
Less: unrecorded lodgements (1,900)
Add: outstanding cheques 2,300
Balance per bank statement (5,280)
15.3 Rectify
Bank
$ $
Balance b/d 322 Bank charges (1) 143
Error in opening balance (4) (850 – 805) 45 Cheque drawn entered as $89 (2) 9
(98 – 89)
Dividend received (8iii) 1,147
Cheque drawn entered as receipt (2) 460
Undercast (9) 200
(2 × $230)
Balance c/d 838
Cheque returned written back (3) 180
Loan interest (8i) 1,100
HP repayments (8ii) 660
2,552 2,552

Bank reconciliation statement as at 31 May 20X5


$
Balance per bank statement (870) O/D

Add: lodgements not yet credited 580

Less: outstanding cheques (630)

Add: cheque wrongly debited by bank 82

Balance per cash book (838) O/D

END OF CHAPTER

314
Correction of errors 16
Syllabus Guide Detailed Outcomes
Having studied this chapter you will be able to:
• Identify the types of error which may occur in bookkeeping systems.
• Identify errors which would be highlighted by the extraction of a trial balance.
• Prepare journal entries to correct errors.
• Calculate and understand the impact of errors on the income statement and statement of financial position.
• Understand the purpose of a suspense account.
• Identify errors leading to the creation of a suspense account.
• Record entries in a suspense account and make journal entries to clear it.

Exam Context
Questions on this area are likely to focus on three main areas. You may be asked to identify which explanations could
have led to a particular difference or be asked to identify the journal entry to correct an error. You may also need to
determine the effect errors may have on the profit figure.

Qualification Context
This topic is only tested in Financial Accounting.

315
16: CORRECTION OF ERRORS

Overview

Types of error

Correction of errors

Suspense account Adjustments to profit

316
16: CORRECTION OF ERRORS

1 Introduction
1.1 Chapter 6 showed us how the trial balance was extracted from the ledger accounts and that
it should balance, i.e. total debits should equal total credits.

1.2 If the trial balance doesn't balance then an error has definitely been made and must be
corrected.

2 Types of error
2.1 The following errors will still allow the trial balance to balance.

Type of error Example


Section 1
Error of omission

Error of commission

Error of principle

Compensating error

2.2 The trial balance will not balance if total debits do not equal total credits.
This could be due to the following:
(1) Transposition error

(2) An entry has been posted where


(a) debits ≠ credits

(b) a debit entry has been posted and no corresponding credit made (or vice
versa)

317
16: CORRECTION OF ERRORS

(c) two debit entries or two credit entries have been posted.

These errors will be corrected by creating a suspense account and making a journal entry
to correct the error.

3 Suspense accounts
3.1 A suspense account is a temporary account. They never appear in the final accounts.

3.2 It is used for two main reasons:


(1) To account for a debit or credit entry when the accountant is unsure as to where it
should go
(2) To make a preliminary trial balance balance when an error has been detected.

3.3 Steps to clear a suspense account.


(1) Determine the original accounting entry which was made.
(2) Decide what entry should have been made.
(3) Make the required adjustment.

3.4 Illustration
W Co sold goods with a value of $2,500 to James, a credit customer. When recording the
sale W Co posted the transaction to the correct accounts but made two debit entries.
Steps
(1) Entry made was:
Dr Trade receivables $2,500
Dr Sales $2,500
(2) Entry should have been:
Dr Trade receivables $2,500
Cr Sales $2,500
(3) Correction:
The trade receivables entry is correct but sales have been debited by $2,500 when
they should have been credited by that amount.
The correction is therefore twice the original error:
Dr Suspense account $5,000
Cr Sales (2 × $2,500) $5,000
Being: correction of sales posting.

318
16: CORRECTION OF ERRORS

Lecture example 1 Technique demonstration

Dan, the bookkeeper of Tiffany's, has made his usual mess of things and produced the following
attempt at a trial balance for the year ended 30 April 20X7.

$ $
Property, plant and equipment
At cost 60,000
Provision for depreciation 31,000
Capital at 1 May 20X6 53,000
Profit for the year 12,300
Inventory, at cost 14,000
Receivables ledger control account 9,600
Payables ledger control account 6,500
Balance at bank 1,640
85,240 102,800
As chief accountant you discover the following:
(1) A rent payment of $350 in March 20X7 had been debited in the receivables ledger control
account.
(2) Discounts allowed of $500 during the year ended 30 April 20X7 had not been recorded in
the books.
(3) No entry had been made for the refund of $2,620 made by cheque to V Woolf in March
20X7, in respect of defective goods returned to Tiffany. V Woolf, who had already paid for
the goods, returned them on 28 February 20X7.
(4) The total column of the cash receipts book had been overcast by $1,900 in March 20X7.
(5) The purchase of stationery for $1,460 cash in June 20X6 has been correctly entered in the
cash account, but no entry has been made to the appropriate expense account.
(6) Capital of $35,000 was recorded incorrectly as $53,000.
Required
Prepare
(a) Journal entries to correct the above errors;
(b) A suspense account showing how it is cleared.

319
16: CORRECTION OF ERRORS

Solution

320
16: CORRECTION OF ERRORS

4 Adjustments to profit
4.1 When errors are corrected they may affect the business' profit for the year figure.

4.2 For example in Lecture example 1, item 5 tells us that a stationery expense of $1,460 has
not been recorded in the expense account.
The profit for the year figure in the trial balance of $12,300 is therefore too high and needs
to be corrected.

4.3 This is done by using a statement of adjustments to profit.

Proforma
4.4
$ $ $
+ –
Original profit X
Adjustment:
(a) over depreciation X
(b) unrecorded expense X
(c) unrecorded sale X
X (X) X
Adjusted profit X

Lecture example 2 Technique demonstration

Required
Prepare a statement of adjustments to profit for Lecture example 1.

Solution
Statement of adjustments to profit for the year ended 30 April 20X7.

Increases Decreases
$ $ $
Draft profit
Adjustments

Revised profit

321
16: CORRECTION OF ERRORS

Lecture example 3 Exam standard for 2 marks

Z Co's income statement showed a profit of $112,400 for the year ended 30 September 20X7. The
following errors were later discovered:
(1) Sales returns of $2,700 had been recorded as a new sale.
(2) A machine which had been held for two years and had originally cost $15,000 was
depreciated this year using a 33 31 % reducing balance basis. Z Co's policy is to depreciate
machines over four years.
Required
What would be the net profit after adjusting for these errors?
A $103,250
B $105,750
C $105,950
D $108,450

Solution

322
16: CORRECTION OF ERRORS

5 Chapter summary
Section Topic Summary
Quick Quiz
2 Types of error There are four types of errors: errors of omission,
commission, principle and compensating errors which
will still allow the trial balance to balance.
If an error is made however where debits ≠ credits then
the trial balance will not balance.
3 Suspense accounts Where the trial balance does not balance a suspense
account will be inserted and the errors, once identified,
will be corrected via a journal entry.
A suspense account should never appear in the final
financial statements.
4 Adjustments to profit Where the process of correcting errors requires
changes to income and expense accounts the
business’ profit will be affected. In this case a
statement of adjustments to profit can be prepared
to determine the revised profit figure.

323
16: CORRECTION OF ERRORS

Chapter Summary

Types of error

• Error of omission: Transaction not recorded


• Error of commission: Debits and credits balance but the entry is made to the wrong account
For example, an expense is debited to the rent account rather than the electricity account
• Error of principle: Debits and credit balance but the entry is made to the wrong 'type' of account
For example, machine repairs debited to the machine asset account
• Compensating error: Two separate errors are made which correct each other
• Transposition error: Here debits ≠ credits and so the trial balance will not balance
For example, the posting of a credit sale as
Dr Trade receivables $210
Cr Sales $120

Correction of errors

Suspense account Adjustments to profit

'A temporary account which never appears in 'When errors are corrected they may affect the
the financial statements' business' profit'
• Used when: • Only errors relating to items of income or
– an accountant is unsure of a double entry expenses will affect profit
– a preliminary trial balance does not
balance
• Must be cleared out
• Steps:
(1) What entry was made?
(2) What entry should have been made?
(3) What entry is required to correct the
entries?

324
Chapter 16: Questions

325
16: QUESTIONS

16.1 Which of the following errors could result in a suspense account being required to balance the trial
balance?
A Cash received from receivables treated as a cash sale
B Payments to suppliers of $513 recorded as $531 in the payables ledger
C A supplier’s invoice for $19 recorded as $91 in the purchases account (1 mark)
16.2 Duncan corrected the following errors before producing his final balance sheet. What was the balance on
the suspense account before he did this?
(i) Sales day book for March overcast by $63.
(ii) Cash receipts from receivables of $713 posted to the receivables ledger control account as $731.
(iii) Cash received from the issue of $1,000 debentures at par had been posted to a suspense
account.
A $982 Dr
B $982 Cr
C $1,018 Dr
D $1,018 Cr (2 marks)

16.3 Russell’s bookkeeper transposed some figures when the week’s cash payments were being posted to the
nominal ledger. Payments for staff wages of $125 were posted to the wages account as $152 and
payments of $31 for stationery were posted to the stationery expense account as $13. The entry required
to correct this is
A Dr stationery $18 Dr suspense $9 Cr wages $27
B Dr wages $27 Dr stationery $18 Cr suspense $45
C Dr wages $27 Cr stationery $18 Cr suspense $9
D Dr suspense $45 Cr wages $27 Cr stationery $18 (2 marks)

16.4 Which of the following errors would cause a trial balance imbalance?
(i) The discounts received column of the cash payments book was overcast.
(ii) Cash paid for the purchase of office furniture was debited to the general expenses account.
(iii) Returns inwards were included on the credit side of the trial balance.
A (i) only
B (i) and (ii)
C (iii) only
D all of the errors (2 marks)

16.5 If sales of $150 has been wrongly entered on the debit side of the purchases account, but correctly
entered in the trade receivables account, the totals on the trial balance would show:
A The debit side to be $150 more than the credit side
B The debit side to be $300 more than the credit side
C The debit side to be $150 less than the credit side
D The debit and credit sides to be equal in value (2 marks)

326
16: QUESTIONS

16.6 Platinum Co
Platinum Co, a manufacturer of electrical goods, has just produced its draft accounts for the year ended
30 September 20X7. These show a draft profit of $28,960.
Unfortunately, the accountant has since discovered the following matters which require consideration
before the final accounts can be prepared:
(1) Returns outwards to Metals Co in June 20X7 of $490 have been treated as returns inwards in
error in the nominal ledger.
(2) R. Silverman, a customer owing $1,850 has gone bankrupt. Full allowance had been made
against this amount in Platinum's accounts for the year ended 30 September 20X6.
(3) An item of equipment with a net book value of $6,000 (cost $10,000) was sold for $5,000 in
September 20X7. The proceeds were included in cash and credited to the motor expenses
account. No other entries were made.
(4) An amount owing from Aluminium Co of $780 was written off in January 20X7. The amount was
removed from trade receivables and debited to the sales account.
Required
Calculate the corrected profit for the year ended 30 September 20X7.

327
16: QUESTIONS

328
Chapter 16: Answers

329
16: ANSWERS

16.1 C

16.2 B (i) No suspense account effect


(ii) Entry made was Dr Bank $713, Cr RLCA $731 and therefore Dr Suspense $18
(iii) Entry made was Dr Bank $1,000, Cr Suspense $1,000
Original suspense account balance is therefore $18 Dr + $1,000 Cr = $982 Cr

16.3 A

16.4 C

16.5 B
16.6 Platinum Co
(a) Statement of corrected profit for the year ended 30 September 20X7.
$ $ $
Draft profit + – 28,960

(1) Returns outwards ($490 x 2) 980


(2) No effect
(3) Disposal of machine 6,000
($5,000 + $1,000)
(4) No effect

980 (6,000) (5,020)


Revised profit 23,940

END OF CHAPTER

330
17
Home study chapter –
Preparation of
financial statements for
sole traders

Syllabus Guide Detailed Outcomes


Having studied this chapter you will be able to:
• Prepare extracts of an opening trial balance.
• Prepare journal entries to correct errors.
• Record entries in a suspense account
• Make journal entries to clear a suspense account
• Prepare extracts of a statement of financial position and income statement from given information.

Exam Context
This chapter recaps some of the key skills you have learnt in the chapters covered to date. Whilst you will not be asked
to produce a statement of financial position or an income statement in the real exam any of the adjustments in this
chapter could be tested as an individual question. This chapter will also help you to see how financial accounting fits
together.

Qualification Context
The skills to produce a statement of financial position and an income statement are tested in detail in the Fundamentals
level paper, Financial Reporting (F7).

331
17: HOME STUDY CHAPTER – PREPARATION OF FINANCIAL STATEMENTS FOR SOLE
TRADERS

Overview

Preparation of financial
statements for sole traders

Trial balance

Adjustments Suspense account

Income Statement and


Statement of financial
position

332
17: HOME STUDY CHAPTER – PREPARATION OF FINANCIAL STATEMENTS FOR SOLE
TRADERS

1 Introduction
1.1 The purpose of this chapter is to recap some of the skills covered in Chapters 1–16.

1.2 You will not be required to answer a question in the format of Lecture example 1 in the
exam. However completing this exercise will revise your understanding of topics covered so
far and enable you to see the end product – a business' transactions ordered into a set of
financial statements.

Lecture example 1 Technique demonstation

You have been given the information below and asked to prepare the accounts of Mugg for the
year ended 31 December 20X7.
Trial balance as at 31 December 20X7.
Dr Cr
$ $
Capital account at 1 January 20X7 2,377
Rent 500
Inventories 1 January 20X7 510
Electricity 240
Insurance 120
Wages 1,634
Trade receivables 672
Sales 15,542
Repairs 635
Purchases 9,876
Discounts received 129
Drawings 1,200
Petty cash 5
Bank 762
Motor vehicles at cost 1,740
Furniture and fixtures at cost 830
Accumulated depreciation at 1 January 20X7
– Motor vehicles 435
– Furniture and fixtures 166
Travel and entertaining 192
Trade payables 700
Suspense account 433

19,349 19,349
The following information is also available:
(1) Closing inventories, valued at cost, amounts to $647;
(2) Mugg has drawn $10 a month and these drawings have been charged to wages;
(3) Depreciation is to be provided at 25% on cost on motor vehicles, and 20% on cost on
furniture and fixtures;
(4) Bad debts totalling $37 are to be written off;

333
17: HOME STUDY CHAPTER – PREPARATION OF FINANCIAL STATEMENTS FOR SOLE
TRADERS
(5) $180 received from a credit customer was correctly entered in the trade receivables account
and credited to the bank account;
(6) Mugg has taken goods from inventories for his own use. When purchased by his business
these goods cost $63 and they would have been sold for $91;
(7) The annual rental of the business premises is $600, and $180 paid for electricity in August
20X7 covers the 12 months to 30 June 20X8;
(8) Discounts allowed of $73 have only been recorded in the trade receivables account.
Required
(a) Prepare journal entries to record items (1) – (8).
(b) Clear the suspense account.
(c) Produce an income statement for the year ended 31 December 20X7 and a statement of
financial position as at that date.

Solution
(a) Journals

(1)

(2)

(3)

(4)

334
17: HOME STUDY CHAPTER – PREPARATION OF FINANCIAL STATEMENTS FOR SOLE
TRADERS

(5)

(6)

(7)

(8)

(b)
Suspense account
$ $

335
17: HOME STUDY CHAPTER – PREPARATION OF FINANCIAL STATEMENTS FOR SOLE
TRADERS
(c) Mugg
Income statement for the year ended 31 December 20X7
$ $
Sales
Less: cost of sales
Opening inventories
Purchases
Less: closing inventories

Gross profit
Discounts received
Less: expenses:
Rent
Electricity
Insurance
Wages
Repairs
Depreciation
Travel and entertaining
Bad debts
Discounts allowed

Profit for the period


Mugg
Statement of financial position as at 31 December 20X7
Accumulated
Cost depreciation NBV
$ $ $
Non-current assets
Motor vehicles
Furniture and fixtures

Current assets
Inventories
Trade receivables
Prepayments
Cash and bank balances

Capital
Capital as at 1 January 20X7
Profit for the period
Less: drawings

Current liabilities
Trade payables
Accruals

336
17: HOME STUDY CHAPTER – PREPARATION OF FINANCIAL STATEMENTS FOR SOLE
TRADERS

2 Chapter summary
Quick Quiz Section Topic Summary
1 Introduction The statement of financial position and the income
statement are the end product produced by a business.
All the business’ transactions need to be categorised
into the books of prime entry and posted to the nominal
ledger. The trial balance is then extracted and some
adjustments may need to be made before the financial
statements are drawn up.
You will not have to produce a statement of financial
position or income statement; however this chapter
should reinforce your understanding of Chapters 1 – 16.

337
17: HOME STUDY CHAPTER – PREPARATION OF FINANCIAL STATEMENTS FOR SOLE
TRADERS

Chapter Summary

Preparation of financial
statements for sole traders

Trial balance

'A list of the balances brought down on each ledger account'

Adjustments Suspense account

'A temporary account which never


• For example:
appears in the financial
- closing inventories
statements'
- depreciation
- bad and doubtful
debts
Income Statement and
- accruals and
prepayments Statement of financial
- correction of errors position

338
Chapter 17: Question

339
17: QUESTION

17.1 Drawings are an expense of the business. Is this statement true or false?
A True
B False (1 mark)

340
Chapter 17: Answer

341
17: ANSWER

17.1 B

END OF CHAPTER

342
Incomplete records 18
Syllabus Guide Detailed Outcomes
Having studied this chapter you will be able to:
• Understand and apply techniques used in incomplete record situations:
(i) use of accounting equation
(ii) use of ledger accounts to calculate missing figures
(iii) use of cash and/or bank summaries
(iv) use of profit percentages to calculate missing figures

Exam Context
Questions on this chapter will require you to identify missing figures, for example sales, closing inventories and
drawings. The Pilot Paper included two questions asking you to derive the value of closing inventories using information
about the gross profit margin earned by the business.

Qualification Context
This topic is only tested in Financial Accounting.

Business Context
Some sole traders do not keep very detailed accounting records. They still however need to produce accounts so they
know how their business is performing and also how much tax to pay to the tax authorities. The preparation of accounts
from incomplete records can generate a lot of income for smaller accountancy practices.

343
18: INCOMPLETE RECORDS

Overview

Margin Cost structures Mark-up

Incomplete records

Techniques for solving


incomplete records

Derive missing figures from


given information

Sales Purchases Drawings Inventory

344
18: INCOMPLETE RECORDS

1 Issue
1.1 Individuals running small businesses such as a newsagent or greengrocer may not keep all
of the accounting records we have studied or have a detailed understanding of double entry
bookkeeping.

1.2 They still need to know how the business is performing and so will produce financial
statements. If some necessary information isn't maintained by the business, it will need to
be derived from other available information.

2 Cost structures
2.1 Cost structure information is usually expressed in one of two ways, either as a margin or a
mark-up.
(a) Margin: here gross profit is expressed as a percentage of sales, for example a
margin of 25% gives:
Sales 100%
Cost of sales 75%
Gross profit 25%
(b) Mark-up: here gross profit is expressed as a percentage of cost of sales, for
example a mark-up of 35% gives:
Sales 135%
Cost of sales 100%
Gross profit 35%

2.2 Remember that:

Cost of sales = opening inventories + purchases – closing inventories

Lecture example 1 Preparation question

W Co has on average a profit margin of 40%. In 20X7 sales total $476,000.


Required
What is cost of sales? $
Workings

345
18: INCOMPLETE RECORDS

Lecture example 2 Preparation question

Y Co operates with a standard mark-up of 30% and has the following information available for
20X7.
$
Sales 221,000
Opening inventories 43,000
Closing inventories 47,500
Required
What is the value for purchases in 20X7? $

Workings

Lecture example 3 Exam standard for 2 marks

On 1 January 20X7 J Co had inventory of $620,000. Sales for the month amounted to $985,000
and purchases were $700,000. At the end of January a fire in the warehouse destroyed some
inventory items. The owners salvaged inventory valued at $180,000. J Co operates with a mark up
of 25%.
What is the cost of inventory destroyed in the fire?
A $335,000
B $352,000
C $401,250
D $532,000

Solution

346
18: INCOMPLETE RECORDS

3 Other techniques for solving incomplete records

Lecture example 4 Preparation question

A Co has recorded the following details relating to trade payables:


$
Balance at 1.1.X7 38,450
31.12.X7 43,825
Cash paid from till 430
Payments from bank 167,224
Required
Based on the information above what was the value of purchases made during the year?

$
Workings

Trade payables
$ $

347
18: INCOMPLETE RECORDS

Lecture example 5 Preparation question

B Co maintains a cash float of $50. In 20X7, all receipts from credit customers were banked, after
the following payments from the till had been made:
$
General expenses 4,500
Drawings 6,250
Total bankings in the year amounted to $28,454, and opening and closing trade receivables were
$1,447 and $1,928 respectively.
Required
Based on the information above what was the value of sales made during the year? $
Workings
Cash
$ $

Trade receivables
$ $

348
18: INCOMPLETE RECORDS

Lecture example 6 Exam standard for 2 marks

Bob owns and manages B Co although he does not keep detailed accounting records.
All of Bob's sales are for cash. He pays certain expenses from his till and then banks the remaining
funds.
Bob maintains a $1,000 float and operates with a margin of 20%. He has provided you with the
following information.
$
Purchases of goods (on credit) 20,000
Wages for clerical assistant (per week; there are 52 weeks in the year) 100
Stationery 500
Electricity 1,200
Bankings 12,800
Opening inventories 2,000
Closing inventories 3,000
Bob is unsure of the level of drawings taken during the year but estimates they were between $60
and $90 per week.
Required
What were Bob's drawings during the year? $

Workings

349
18: INCOMPLETE RECORDS

4 Goods drawn by proprietor


4.1 The owners of the business may at times take goods or cash from the business for their own
use. We have seen these before as drawings.
In incomplete records questions these drawings need to be included.
Cash drawings
Dr Drawings
Cr Cash
Goods taken for own use
Dr Drawings
Cr Purchases

These are recorded at the cost to the business not at sale price.
They are taken out of purchases and not recorded against inventories.
Note: If you are using a trade payables T account to calculate purchases remember to
adjust purchases for any goods taken by proprietor.

Example
4.2 During the year ended 31 December 20X7, Peter Albert, a sole trader, carried out the
following transactions:
$
Sales (40 units @ $100) 4,000
Purchases (45 units @ $60) 2,700
His inventories (at cost) were:
$
1 January 20X7 (5 units @ $60) 300
31 December 20X7 (8 units @ $60) 480
During the year he had withdrawn two units for his own use. Firstly, ignoring the drawings,
an outline trading account would appear as follows:
$ $
Sales 4,000
Cost of sales
Opening inventories 300
Purchases 2,700
3,000
Less: closing inventories (480)
2,520
Gross profit 1,480
How should the drawings of goods be treated?

350
18: INCOMPLETE RECORDS

It should be fairly obvious that the debit entry will be to drawings on the balance sheet, but
what about the credit entry? It will not, as you might initially think, go to inventories (because
these goods were not in hand at the year end so they are not included in the value of $480)
but rather to purchases (as this is where they will have been previously recorded).
In the trading account, this credit entry is often shown as a separate deduction from cost of
sales, i.e.:
$ $
Sales 4,000
Cost of sales
Opening inventories 300
Purchases 2,700
Less: goods drawn by proprietor
2 units @ $60 (120)
2,880
Less: closing inventories (480)
2,400
Gross profit 1,600

Points to note
4.3 (a) Drawings of goods are recorded at cost.
(b) Gross profit figure now makes sense, i.e. profit of $40 per unit × 40 units sold.

351
18: INCOMPLETE RECORDS

5 Chapter summary
Section Topic Summary
Quick Quiz
1 Issue Not all businesses keep proper accounting records,
however all businesses need to know how much profit
they have made in a particular year so that they can
pay the relevant amount of tax over to the tax
authorities.
Where a business does not have sufficient records to
produce financial statements they need to piece
together the missing information.
2 Cost structures A margin is where a business expresses gross profit as
a percentage of sales.
A mark-up is where gross profit is expressed as a
percentage of cost of sales.
3 Other techniques for Other techniques that may be used in solving
solving incomplete incomplete records questions involve putting all known
records information in to one or two ledger accounts and
balancing off to derive the required information. These
questions are essentially a test of double entry skills.
4 Goods drawn by A business is a separate entity from its owner which
proprietor means that any monies or goods taken out of the
business for personal use must be classified as
drawings. Drawings of goods are always recorded at
cost.

6 Double Entry Summary for Chapter 18


6.1 Adjustment to record cash drawings:

Dr Drawings (SOFP)
Cr Cash (SOFP)

6.2 Adjustment to record drawings of goods:

Dr Drawings (SOFP)
Cr Purchases (I/S)

352
18: INCOMPLETE RECORDS

Chapter Summary

Margin Cost structures Mark-up

'Gross profit expressed as a 'Gross profit expressed as a


percentage of sales' percentage of cost of sales'
• For example, a 20% margin: • For example, a 25% mark-up:
Sales $1.00 Sales $1.25
COS $0.80 COS $1.00
Gross profit $0.20 Gross profit $0.25
Incomplete records

Techniques for solving


incomplete records

Derive missing figures from


given information

Sales Purchases Drawings Inventory

• Derive sales figure by: • Derive purchases figure by: • Put all known information into • Derive using cost structure
- putting all known - putting all known information a cash T account information
information into a trade into a trade payables T • If the question states that • Drawings of inventory are
receivables T account account drawings were between $50 always valued at their cost
- using cost structure - using cost structure and $80 per week this and not their selling price
information to work from information to derive indicates that drawings are
cost of sales back to purchases as part of the cost the missing figure
sales of sales figure

353
18: INCOMPLETE RECORDS

354
Chapter 18: Questions

355
18: QUESTIONS

18.1 If a business has sales of $6,000 and a margin of 20%, what is the gross profit? $
(1 mark)

18.2 A trader has budgeted sales for the coming year of $300,000. He achieves a constant mark-up of 25% on
cost. He plans to reduce his inventory level by $14,000 over the year. How much will his purchases for
the year be?
A $211,000
B $239,000
C $226,000
D $254,000 (2 marks)

18.3 A business has opening inventories of $273 and makes purchases during the year of $2,781. The
proprietor removes goods costing $87 for his own use. The business achieves a constant mark-up of 20%
on cost and records sales for the year of $3,360.
What is the cost of closing inventories? $ (2 marks)

18.4 Jethro sold goods for $157,470 during the year ended 31 October 20X7. Inventories at that date were
valued at $8,920 more than at the previous year end. Jethro prices his goods to give a mark-up of 45%.
What was the total value of purchases in the year ended 31 October 20X7?
A $77,689
B $95,529
C $99,680
D $117,520 (2 marks)

356
Chapter 18: Answers

357
18: ANSWERS

18.1 $1,200
$6,000 × 0.2 = $1,200

18.2 C $
100
Cost of sales = $300,000 × 240,000
125

Less: decrease in inventories (14,000)


226,000

18.3 $167
Cost of sales $
100
$3,360 × 2,800
120

Less: opening inventories (273)


purchases (2,781 – 87) (2,694)
∴ closing inventories 167

18.4 D Cost of sales = $157,470 x 100/145 = $108,600


∴ purchases = $108,600 + $8,920
= $117,520

END OF CHAPTER

358
Partnerships 19
Syllabus Guide Detailed Outcomes
Having studied this chapter you will be able to:
• Understand and identify the typical content of a partnership agreement, including profit sharing terms.
• Understand the nature of capital accounts, current accounts and division of profits.
• Calculate and record the partners' shares of profits/losses.
• Account for guaranteed minimum profit shares.
• Calculate and record
(i) partners' drawings
(ii) interest on drawings
(iii) interest on capital
(iv) partner salaries
• Prepare an extract of a capital account and a current account.
• Prepare extracts of the income statement, including division of profit, and statement of financial position of a
partnership.
• Define goodwill in relation to partnership accounts and identify the factors leading to the creation of goodwill.
• Calculate the value of goodwill from given information.

Exam Context
Questions on this topic are likely to require you to calculate a partner's profit share. This may include dealing with
partners' salaries, interest on capital and drawings and loan interest. You may also need to allocate goodwill to partners
when a new partner is admitted.

Qualification Context
Partnerships are only examined in this paper.

Business Context
Many individuals set up business as a sole trader – as they expand they need new finance. One way of obtaining this is
to go into partnership with someone else. That other person could provide some of the finance needed. They may also
bring new ideas to the table. Becoming a partnership will mean that the sole trader will share some of their risk but they
will also need to share their profits too!
It is always recommended that a partnership agreement is drawn up to retain a legal record of how the partnership will
operate.

359
19: PARTNERSHIPS

Overview

Partnership agreements Appropriation account

Partnerships

Capital accounts Current accounts Other issues

Guaranteed minimum Loans Goodwill


profit share

360
19: PARTNERSHIPS

1 Definition
1.1 Partnership: The relationship which exists between two or more persons carrying on
a business with a view to profit.

1.2 Partnerships are similar to sole traders. With a sole trader the owner will run the business
and any profits belong to him. The sole trader also bears the risk that the business may not
be successful.
In a partnership, the owners (partners) run the business together and share profits and risk.

1.3 Most partnerships have unlimited liability which means the partners are personally liable
for the debts of the business.
Liability is also joint and several so if one partner cannot meet the partnership's obligations
the other partners must make up any shortfall.

1.4 Limited liability partnerships (LLPs) exist nowadays to limit partner liability. These are
outside the scope of the F3 syllabus.

2 Partnership agreements
The partners will need to agree the terms under which the partnership will operate, and
decide, for example how much capital each partner will contribute and what share of profits
they will be entitled to.
This is done by way of a partnership agreement which usually covers the following areas:
2.1
Area Consideration
Capital • how much each partner pays in
• whether a "Fixed Capital" level is specified
Profit sharing ratio • allocation of profit
(PSR) • more to senior partners?
• equal shares?
• guaranteed minimum profit share?
Salaries • whether or not partners are entitled to salaries
• it is an appropriation of profit
• it is not an income statement expense
Interest on capital • whether or not allowed
• paid on capital injected
• interest rate
Drawings • may set a limit
• may set an interest charge

361
19: PARTNERSHIPS

3 Accounting for partnerships


3.1 There are two key differences between accounting for a sole trader and a partnership.
These are illustrated below.

3.2 Income statement


Sole trader Partnership
$ $
Sales X Sales X
Cost of sales (X) Cost of sales (X)
Gross profit X Gross profit X
Less: expenses (X) Less: expenses (X)
Profit for period X Profit for period X

All belongs to Shared between partners


sole trader according to the
partnership agreement

Appropriating the profit for the period


3.3 The profit for the period is appropriated (shared out) between the partners according to
their partnership agreement.
Steps
(1) Allocate the partner salaries
(2) Allocate any interest on capital
(3) Charge any interest on drawings
(4) Allocate remaining profit balance in profit sharing ratio
This is done using an appropriation account.
Appropriation account
Salaries Profit before appropriation X
Partner A X Interest on drawings
Partner B X Partner A X
Interest on capital Partner B X
Partner A X
Partner B X
* PSR
Partner A X
Partner B X

X X

* PSR is always the last entry, splitting the residual profit after all other allocations

362
19: PARTNERSHIPS

3.4 Statement of financial position


Sole trader Partnership
$ $
Proprietor’s interest Capital accounts
Capital X Partner A X
Profit X Partner B X
Less: drawings (X) X
X Current accounts
Partner A X
Partner B X
X

Amount owed back Amount owed back to the


to the owner by the partners by the business
business

3.5 Capital accounts


These represent the capital invested in the business by each individual partner. The
balances in these accounts will remain relatively static.
The capital account can be shown as one T account subdivided into columns.
For example, if Partner A contributed $5,000 and Partner B $8,000, the capital account
would show.
Capital account
Ptnr Ptnr Ptnr Ptnr
A B A B
$ $ $ $
Bal b/d 5,000 8,000

3.6 Current accounts


These record each partner's day to day transactions with the business.
The main entries in the current account will be the partners’ appropriation of profits (salary,
interest on capital and profit share) less drawings they have taken from the business and
any interest charged on those drawings.
Current account
Ptnr Ptnr Ptnr Ptnr
A B A B
$ $ $ $
Drawings 2,900 970 Bal b/d 1,000 1,500
Interest on drawings 100 30 Salaries 1,500 –
Interest on capital 500 800
Bal c/d 4,000 5,000 Profit share 4,000 3,700
7,000 6,000 7,000 6,000

363
19: PARTNERSHIPS

Lecture example 1 Preparation question

(a) On 1 January 20X4 Tick, Cast and Balance entered into partnership together as chartered
certified accountants. They agreed that Balance would receive a salary of $15,000 p.a., they
would all be allowed interest on capital of 12% p.a. and they would share profits in the ratio:
Tick five tenths, Cast three tenths, Balance two tenths. They paid in the following capital
amounts:
Tick $50,000
Cast $30,000
Balance $20,000
In the year to 31 December 20X4 their profit for the period was $50,000.
During the year they had made drawings in cash as follows:
30.6.20X4 Tick $6,000
30.9.20X4 Cast $4,000
31.12.20X4 Balance $8,800
Required
(i) Write up their capital accounts in columnar form.
(ii) Write up the appropriation account.
(iii) Write up their current accounts in columnar form.
(iv) Show the partners' balances on the statement of financial position.

Solution
(i)
Capital Accounts
Tick Cast Balance Tick Cast Balance
$ $ $ $ $ $

364
19: PARTNERSHIPS

(ii)
Appropriation account for the year ended 31 December 20X4
$ $

(iii)
Current accounts
Tick Cast Balance Tick Cast Balance
$ $ $ $ $ $

365
19: PARTNERSHIPS

(iv) Tick, Cast and Balance

Statement of financial position as at 31 December 20X4 (extract)


$ $

Capital accounts

Tick

Cast

Balance

Current accounts

Tick

Cast

Balance

(b) What would your answer be to (ii) and (iii) if the agreement had also provided for interest to
be charged on drawings at the rate of 10% p.a.?
(ii)
Appropriation account for the year ended 31 December 20X4
$ $

366
19: PARTNERSHIPS

(iii)
Current accounts
Tick Cast Balance Tick Cast Balance
$ $ $ $ $ $

367
19: PARTNERSHIPS

4 Guaranteed minimum profit share


4.1 It may be that the partnership agreement specifies that one or more partners must receive a
minimum share of profits.

4.2 If when the appropriation of profits is made this level is exceeded, there is nothing further to
do.

4.3 If, however, there is a shortfall then this will be made up by the remaining partners in their
profit sharing ratio.

4.4 Illustration
A, B and C are in partnership and share profits in the ratio 2:2:1.
The partnership made a profit for the year of $50,000. A and B each receive a salary of
$12,000. Interest due on the partners' capital is $2,000, $1,700 and $1,500 respectively.
No interest is charged on drawings.
C has a guaranteed minimum profit share of $7,000.
A B C Total
$ $ $ $
Salaries 12,000 12,000 – 24,000
Interest on capital 2,000 1,700 1,500 5,200
Profit share (2:2:1) 8,320 8,320 4,160 20,800
Subtotal 22,320 22,020 5,660 50,000
Guaranteed minimum profit
share shortfall (2:2) (670) (670) 1,340 –

21,650 21,350 7,000 50,000

5 Loans
5.1 Unlike sole traders, a partner can make a loan to the partnership.

Reasons for making a loan


5.2 (a) Partnership may be short of funds.
(b) Partner is unwilling to tie cash up for long period.
(c) Partner wants to earn interest.
(d) Partner retires but partnership does not have enough cash to buy out his share.

Accounting treatment
5.3 The loan is shown as a non-current liability on the statement of financial position and not
in the partner's capital account.

5.4 The interest incurred on the loan is shown as an expense in the income statement (just
like bank interest). It will need to be deducted from the profit figure before any appropriation
is made if it has not already been accounted for.

368
19: PARTNERSHIPS

5.5 If the loan interest has not been paid by the end of the year, the liability will be shown in the
relevant partner’s current account.
The double entry would be:
Dr Loan interest expense (I/S)
Cr Current account (SOFP)

Lecture example 2 Exam standard for 2 marks

X, Y and Z are in partnership sharing profits in the ratio 6:3:1.


Y made a loan of $10,000 to the partnership on 1 July 20X7. The loan carries interest at 12% but
this has not yet been accounted for.
X and Z receive salaries of $15,000 and $8,000 respectively and interest due on capital to each
partner is $400.
The profit for the year to 31 December 20X7 was $67,000.
Required
What is the amount of profit appropriated to each partner for the year ended 31 December 20X7?
$

Workings

369
19: PARTNERSHIPS

6 Changes to the partnership agreement


6.1 Profits are always appropriated according to the partnership agreement; therefore if the
terms of the agreement change during the period this will affect the profit appropriation.

6.2 Always use the old partnership agreement to appropriate the profits for the first part of the
year and the new partnership agreement for the latter part of the year.

6.3 Assume profits accrue evenly unless the question specifies otherwise.

Lecture example 3 Exam standard for 2 marks

Melanie, Sarah and Angela are in partnership, compiling their accounts for the year to
31 December each year. The partnership agreement states the following:
Until 30 June 20X3

Annual salaries Sarah $40,000


Angela $20,000

Profit sharing ratio Melanie: Sarah: Angela is 60:20:20:


From 1 July 20X3
Salaries to be discontinued, profit sharing ratio to be: 50:30:20
The profit for the year ended 31 December 20X3 was $400,000 before charging partners' salaries,
accruing evenly through the year and after charging an expense of $40,000, which it was agreed
related wholly to the first six months of the year.
Required
How should the profit for the year be divided among the partners? Use a separate page for your
workings.

Melanie Sarah Angela


$ $ $
A 182,000 130,000 88,000
B 200,000 116,000 84,000
C 198,000 118,000 88,000
D 180,000 132,000 88,000

370
19: PARTNERSHIPS

7 Changes to the partnership – goodwill


7.1 When a partner retires from the partnership or a new partner is admitted to the partnership it
is usual for the partners to value the business.

7.2 It is likely that over time the value of items such as property, plant and equipment will
increase over their net book value.
However, hopefully the business will also have built up a good reputation and a loyal
customer base and the business itself will be worth more than its individual assets.

7.3 The worth of a business over and above its individual assets is called goodwill.

Section 2.8-2.10 7.4 When a partner retires it is important that he is paid a sum that represents not just the
money he invested but also his share of the extra value created in the business, i.e. his
share of goodwill.
Illustration
A partnership's statement of financial position may have the following assets and liabilities:
$'000
Property 200
Other assets 120
320
Liabilities (100)
220
The business therefore has a 'book value' of $220,000.
However when the partnership was valued as a whole it was judged to be worth $350,000:
an increase of $130,000.
$80,000 of this increase was believed to be attributable to the increase in the value of the
property, but the other $50,000 was due to the business's superb regional reputation and
wealthy customer base.
This $50,000 is known as goodwill.
Goodwill is therefore added to the partners' accounts according to the existing or old profit
sharing ratio.

7.5 Similarly, when a new partner joins, he will pay in a sum of money (capital). It is important
that the original partners value the partnership so they know its worth and can determine
how much the partner should contribute.

7.6 Goodwill is an extremely subjective figure and so it is not left in the partnership's statement
of financial position, but is removed.
This is done using the new profit sharing ratio.

371
19: PARTNERSHIPS

Lecture example 4 Exam standard

A and B have been in partnership for many years and have built up a business with a balance
sheet value of $150,000.
The partnership now needs new finance to continue to grow and a new partner, C, is to be
admitted on 31 March at which time the value of A and B's partnership is $210,000.
A and B contributed capital of $80,000 and $97,000 respectively when the partnership was
created. They share profits in the ratio 5:7.
C will pay capital of $100,000 into the partnership after which profits will be shared equally
between the three partners. The partnership does not include goodwill in its balance sheet.
Required
Calculate the balance of each of the partner’s capital accounts after C has been admitted to the
partnership.

Solution

372
19: PARTNERSHIPS

8 Chapter summary
Quick Quiz Section Topic Summary
1 Definition A partnership exists where two or more persons are
carrying on a business with a view to a profit.
2 Partnership The purpose of a partnership agreement is to specify
agreements how the partnership operates in terms of the how much
capital the partners pay in and whether they are paid
interest on capital; whether they are entitled to a
salary; whether interest is charged on drawings and
the profit sharing ratio.
3 Accounting for The final profit for the year is appropriated according to
partnerships the partnership agreement. The profit for the year will
be allocated to cover the partners’ salaries and any
interest on capital/ drawings. The balance will then be
shared between the partners according to the profit
sharing ratio. Note that partners’ salaries are not an
expense of the business but an appropriation of
profit.
Capital accounts represent the capital paid in by each
partner and are generally static.
Current accounts record the partners’ day to day
transactions with the business.
4 Guaranteed minimum The partnership agreement may stipulate that a
profit share particular partner is guaranteed a minimum profit share.
Where this is the case any shortfall will generally be
made up by the other partners according to their
profit sharing ratio.
5 Loans Should a partner make a loan to the business then this
is shown as a liability on the statement of financial
position and the interest is recorded as an expense in
the income statement. Any interest owed to the
partner at the end of the year will be credited to their
current account.
6 Changes to the The partners may change the partnership agreement
partnership agreement during the year. If this is the case then profits are
assumed to accrue evenly during the year. The first
period of profits should be appropriated using the old
partnership agreement and the second period using the
new partnership agreement.

373
19: PARTNERSHIPS

Section Topic Summary


7 Changes to the Whenever a new partner is admitted or an existing
partnership - goodwill partner retires the partnership will be valued. The worth
of the partnership over and above the statement of
financial position valued is called goodwill. This is
allocated to the partners according to their profit sharing
ratio.

374
'The sharing out of the profit19:
for the period according to
PARTNERSHIPS
the partnership agreement'
• Steps:
(1) Allocate partner salaries
Chapter Summary (2) Allocate interest on capital
(3) Charge interest on drawings
• Capital and interest on capital (4) Allocate residual profit using PSR
• Drawings and interest on drawings • Check to see whether any loan interest due to
• Salaries partners has been deducted from the profit for the
• Profit sharing ratio (PSR) period figure

Partnership agreements Appropriation account

Partnerships

Capital accounts Current accounts Other issues

• Record the partners' day to day


• Record the capital invested in the
transactions with the business
business by each partner
• Entries include:
• Tend to remain static
- salaries
- interest on capital
- profit share
- drawings
- interest on drawings

Guaranteed minimum Loans Goodwill


profit share

• Show as a non-current 'The value of a business as a


• The partnership agreement may specify whole over and above the
liability in the balance sheet
that one or more partners receive a value of its individual assets'
• Interest is an expense in the
minimum share of profits
income statement • Goodwill is allocated to the
• Any shortfall is made up by the partners using the old PSR
• Any unpaid loan interest is
remaining partners according to their
shown as a credit to the • It is not retained in the
PSR
partner's current account balance sheet but is
removed using the new
PSR

375
19: PARTNERSHIPS

376
Chapter 19: Questions

377
19: QUESTIONS

Data for Questions 19.1 and 19.2


John, Paul and David have been in a partnership for one year providing advice on landscape gardening. The
partnership agreement provides for the following:
• a salary to John of $5,000 per annum.
• interest on capital balances @ 10% per annum.
• interest on drawings @ 5% per annum.
• profit sharing ratio of 2:2:1 respectively.
The capital accounts as at 31 March 20X3 showed the following balances:
John $30,000
Paul $25,000
David $20,000
The partners made the following drawings during the year:
John $6,000 on 30 June 20X2
Paul $2,000 on 31 December 20X2
David $1,500 on 31 March 20X3
On 30 September 20X2 Paul lent the partnership $100,000. Interest (which has not been included in the
accounts) is to be charged at 4% per annum. Loan interest is to be included in the current account. The profit for
the year ended 31 March 20X3 was $40,000.

19.1 What is John’s share of the residual profits? $ (2 marks)


19.2 What will be the balance on Paul’s current account at 31 March 20X3? $ (2 marks)

19.3 Which of the following is not true?


A Partners are jointly and severally liable
B Partner salaries are an appropriation of profits
C Interest on drawings is an appropriation of profits (1 mark)

378
19: QUESTIONS

19.4 A, B and C
A, B and C are in partnership, agreeing to share profits in the ratio of 4:2:1. They have also agreed to
allow interest on capital at 8% per annum, a salary to C of $5,000 per annum, and to charge interest on
drawings made in advance of the year end at a rate of 10% per annum.
The statement of financial position as at 30 June 20X8 disclosed the following:
$ $
Capital accounts A 50,000
B 30,000
C 10,000
90,000
Current accounts A 2,630
B 521
C (418)
2,733
Loan account (5% interest) A 15,000
107,733
Drawings were: A $6,400, B $3,100, C $2,000, with all sums being withdrawn on 1 July 20X8.
Profit for the year to 30 June 20X9 was $24,750, before charging interest on A's loan. The partnership
made a payment to A for loan interest on 29 June 20X9 but has not recorded this in its books.
Required
Prepare the current accounts and the appropriation account for the partners as at 30 June 20X9.

379
19: QUESTIONS

380
Chapter 19: Answers

381
19: ANSWERS

19.1 $10,300
$
Profit per accounts 40,000
Less loan interest
6
4% × $100,000 × 12
(2,000)
38,000

John Paul David Total


$ $ $ $
Salary 5,000 – – 5,000
Interest on capital 3,000 2,500 2,000 7,500
Interest on drawings (225) (25) – (250)
PSR (2:2:1) 10,300 10,300 5,150 25,750 (β)
Total profit 18,075 12,775 7,150 38,000

19.2 $12,775

Current account – Paul

Drawings 2,000 Interest on capital 2,500


Interest on drawings 25 Loan interest 2,000
c/d 12,775 PSR 10,300
14,800 14,800
b/d 12,775

19.3 C Interest on drawings increases available profits to share and is therefore not an appropriation of
profit.

Partner salaries are an appropriation of profit, not an expense.


19.4 A, B and C

Current accounts
A B C A B C
$ $ $ $ $ $

Balance b/d 418 Balance b/d 2,630 521


Drawings 6,400 3,100 2,000 Salary 5,000
Interest on drawings 640 310 200 Interest on capital 4,000 2,400 800
Balance c/d 6,990 3,211 5,032 Share of profit 7,400 3,700 1,850
14,030 6,621 7,650 14,030 6,621 7,650
Balance b/d 6,990 3,211 5,032

382
19: ANSWERS

Appropriation account

$ $
Salary C 5,000 Profit (W1) 24,000
Interest A 4,000 Interest A 640
on capital B 2,400 on drawings B 310
C 800 C 200
7,200 1,150

Share of profits in PSR


A(4/7) 7,400
B(2/7) 3,700
C(1/7) 1,850
β 12,950
25,150 25,150

(W1) Profit 24,750


Interest on loan (750)
24,000

383
19: ANSWERS

END OF CHAPTER

384
Introduction to
company accounting 20
Syllabus Guide Detailed Outcomes
Having studied this chapter you will be able to:
• Understand the capital structure of a limited liability company including ordinary shares, preference shares and
loan notes.
• Record movements in the share capital and share premium accounts.
• Define a bonus issue and a rights issue, their advantages and disadvantages and show how they are recorded in
the balance sheet.
• Identify and record the other reserves which may appear in the company statement of financial position.
• Record dividends in ledger accounts and the financial statements.
• Calculate and record finance costs in ledger accounts and the financial statements.

Exam Context
Questions on this chapter are likely to focus on the calculation of share capital movements (new issues, bonus issues
and rights issues), dividends and finance costs and their associated journal entries. You may also see a question
comparing a sole trader and a limited company as was included in the Pilot Paper.

Qualification Context
The knowledge covered in this chapter is developed further in the Fundamentals level paper Financial Reporting (F7).
This paper looks in more detail at whether shares and borrowings should be classified as debt or equity and also at how
they should be valued. The area of income taxes is also extended to include adjustments for deferred tax as well as
current tax.

Business Context
When a company is seeking to raise finance it will evaluate its current financing structure and gearing levels before
deciding how to secure additional funds. It will also consider the degree of risk attached to each method of financing and
will weigh up the cost in terms of interest payments versus future dividends. A company will also receive tax relief on its
interest payments (but not on dividends) and so the tax implications will form part of the final decision.

385
20: INTRODUCTION TO COMPANY ACCOUNTING

Overview

Finance costs

Reserves Long term borrowings Income taxes

Introduction to
company accounting

Shares

Accounting treatment

Issue at a premium Bonus issue Rights issue

Dividends

386
20: INTRODUCTION TO COMPANY ACCOUNTING

1 Introduction
1.1 We have seen how financial statements are produced for sole traders and partnerships.
These accounts are not subject to any specific regulation and so there is some flexibility as
to how they are presented.

1.2 Companies use exactly the same bookkeeping process as sole traders and partnerships;
however, the financial statements they produce are subject to regulation and must follow a
prescribed format.
Many of the differences are due to the terminology used by company financial statements.

2 Proforma financial statements


2.1 Income statement for the year ended 31 March 20X7
$'000
Revenue X
Cost of sales (X)
Gross profit X
Other income X
Distribution costs (X)
Administrative expenses (X)
Finance costs (X)
Profit before tax X
Income tax expense (X)
Profit for the year X

387
20: INTRODUCTION TO COMPANY ACCOUNTING

2.2 Statement of financial position as at 31 March 20X7


$'000
ASSETS
Non-current assets
Property, plant and equipment X
Other intangible assets X
X
Current assets
Inventories X
Trade receivables X
Other current assets X
Cash and cash equivalents X
X
Total assets X

EQUITY AND LIABILITIES


Equity
Share capital X
Share premium account X
Revaluation reserve X
Retained earnings X
X
Non-current liabilities
Long term borrowings X
Long term provisions X
X

Current liabilities
Trade payables X
Short term borrowings X
Current tax payable X
Short term provisions X
Total equity and liabilities X

2.3 These proformas will be covered in more detail in Chapter 21.

3 Share capital
Share capital
3.1 It is necessary to be able to distinguish between the following types of share capital:
(a) Authorised share capital – maximum number of shares the company may issue.
Section 2.3
(b) Issued share capital – number of shares actually issued to shareholders.
(c) Called up share capital – the amount of issued share capital the company has
asked shareholders to pay for to date.
(d) Paid up share capital – amount of called up share capital which has been paid
for.

388
20: INTRODUCTION TO COMPANY ACCOUNTING

Types of shares
3.2
Ordinary share Preference share
• Equity share • Fixed rate of dividends (eg 7%
preference share)
• Ordinary shareholders – own business • Receive dividend in priority to ordinary
shareholders
• Usually have voting rights • On winding up, receive capital in priority
• No right to a dividend, receive what
directors decide to pay

4 Share capital: accounting treatment


Issue of new shares
4.1 Rab Co started business on 1 January 20X6 issuing 100,000 ordinary shares of 50c each
for 50c per share. The initial statement of financial position would be:
$
Cash 50,000

Share capital – 50c ordinary shares 50,000

Issue of new shares at a premium


4.2 Where shares are issued for more than their nominal value, the excess must be credited to
a share premium account.

Lecture example 1 Preparation question

On 1 June 20X6 Rab Co issued a further 200,000 ordinary shares of 50c each for 80c per share.
Required
Show how this issue of shares would be accounted for and what the statement of financial position
would look like immediately after the issue.

389
20: INTRODUCTION TO COMPANY ACCOUNTING

Solution
Dr Cr
$ $
Dr Cash
Cr Share capital
Cr Share premium account
Rab Co statement of financial position (extract) as at 1 June 20X6
Equity
$
Share capital – 50c ordinary shares
Share premium account

Bonus issue (capitalisation issue)


4.3 This is used when a company wishes to increase its share capital without needing to raise
additional finance by issuing new shares. Any reserve may be used including the share
premium account.

4.4
Advantages Disadvantage
• Bonus issue can be made from the share • The rationale for a bonus issue is not
premium account which has few other always understood by shareholders
uses
• Will allow the share price to fall (without
disadvantaging shareholder wealth) to
make the company's shares more
affordable to new investors
• Shareholders will now own more shares
and could sell part of their holding

4.5 A bonus issue is always done at nominal value.

390
20: INTRODUCTION TO COMPANY ACCOUNTING

Lecture example 2 Preparation question

Rab Co
Statement of financial position (extract)
$
Share capital – 50c ordinary shares 150,000
Share premium account 60,000
Retained earnings 200,000
410,000
Several years later Rab Co is to make a bonus issue on a 1 for 4 basis.
Required
Show how this issue of shares would be accounted for and prepare the statement of financial
position of Rab Co immediately after the issue.

Solution
Dr Cr
$ $
Dr Share premium account
Cr Share capital

Rab Co
Statement of financial position (extract)
$
Share capital – 50c ordinary shares
Share premium account
Retained earnings

Rights issue
4.6 (a) A rights issue is an issue of shares for cash (unlike a bonus issue) to existing
shareholders.
(b) ‘Rights’ are offered to the existing shareholders who can sell them if they wish.

391
20: INTRODUCTION TO COMPANY ACCOUNTING

4.7
Advantages Disadvantages
• More cost effective way for the • Lack of shareholder interest may reflect
company to raise finance than a badly on the company
fresh issue to the public
• A more time efficient way to issue • Unwelcome predators may try to acquire
shares shares where not all rights are taken up
• If all rights are taken up • Effect on future dividend policy as company
shareholders will maintain their will have issued more shares under the
existing percentage shareholding rights issue than it would have under a fresh
issue to the public

Lecture example 3 Preparation question

One year later, Rab Co is to make a rights issue on a 1 for 5 basis. The rights price is $1.50. All
shareholders take up their rights.
The following statement of financial position extract shows the position before the issue
Rab Co
Statement of financial position (extract)
$
Share capital – 50c ordinary shares 187,500
Share premium account 22,500
Retained earnings 230,000
440,000
Required
Show how this issue of shares would be accounted for and prepare the statement of financial
position of Rab Co immediately following the issue.

Solution
Dr Cr
$ $
Dr Cash
Cr Share capital
Cr Share premium account
Rab Co
Statement of financial position (extract)
$
Share capital – 50c ordinary shares
Share premium account
Retained earnings

392
20: INTRODUCTION TO COMPANY ACCOUNTING

5 Reserves
5.1 The following reserves are commonly found in limited liability company accounts.
(a) The share premium account:
(i) Typical permitted uses:
(1) to issue bonus shares;
(2) to write off share issue expenses.
(b) The revaluation reserve (see Chapter 9):
(c) Other reserves:
as designated by the individual company, for example a 'general reserve'.
(d) Retained earnings:
cumulative undistributed profits less any losses.

6 Dividends
Definition
6.1 Dividends – a sharing out/appropriation of retained earnings to owners/shareholders.

Illustration
6.2 Suppose a company with 1,000 ordinary $1 shares in issue made a profit of $500 in its first
year. The company has two choices as to what can be done with this profit:
(a) distribute it as a dividend to the shareholders;
(b) retain it in the business.
If this company decides to pay a dividend of 10c per share and retain the remaining profits,
the financial statements would appear as follows:
Income statement for the year ended 31 December 20X7
$
Profit for the period 500
Statement of financial position as at 31 December 20X7 (extract)
$
Share capital – $1 shares 1,000
Retained earnings (500 – 100) 400
1,400

6.3 Dividends are charged directly to retained earnings as they are an appropriation of profits
earned to date. They are not an expense of the income statement.

6.4 The double entry is:


Dr Retained earnings (SOFP)
Cr Dividends payable (SOFP)

393
20: INTRODUCTION TO COMPANY ACCOUNTING

6.5 A company may pay dividends in two stages:


(a) Interim (mid year)
(b) Final (end year)
In reality the directors will wait until they know the company's full year profit before declaring
the final dividend.
The final dividend will only be accounted for in the current year if it is declared before the
year end. Otherwise it will be disclosed in a note to the financial statements
(see Chapter 22).

Lecture example 4 Preparation question

ABC Co has the following share capital:


100,000 6% $1 preference shares
200,000 50c ordinary shares
Retained earnings at the beginning of the year were $125,000.
During the year ended 31 December 20X7 it made the following profit:
$
Profit before tax 60,000
Income tax expense 10,000
Profit for the period 50,000
Dividends paid and declared during the year were as follows:
Interim dividend paid 5c per share
Final dividend declared on 20 January 20X8 10c per share
Required
Show the movement in retained earnings for ABC Co for the year ended 31 December 20X7.

Solution
$ $
Retained earnings at beginning of year
Profit for the period
Dividends – Preference
– Ordinary

Retained earnings at end of year

394
20: INTRODUCTION TO COMPANY ACCOUNTING

7 Long term borrowings


7.1 A company may choose to raise finance by issuing shares (equity).
Alternatively it can raise funds by issuing debt.

7.2 One way of raising long term finance is for a company to issue loan notes (also called loan
stock or debentures).
These loans usually carry a fixed rate of interest and have a pre-determined redemption
date, for example, $50,000 10% debentures 2012. This means the company will pay interest
at 10% on the $50,000 borrowed each year. The capital amount of $50,000 will be repaid in
2012.

8 Finance costs
8.1 The interest expense incurred on long term borrowings will be shown as an expense called
'finance costs' in the income statement.

8.2 It will be accounted for as follows:


Dr Finance costs (I/S)
Cr Bank

9 Income taxes
9.1 Companies must pay income tax on their profits. This tax is payable after the end of the
financial year and so the financial statements will include an accrual for the directors' best
estimate of the tax due on the profit for the period.

9.2 The tax is shown as an expense in the income statement and a current liability in the
statement of financial position and will be accounted for as follows:
Dr Income tax expense (I/S)
Cr Current tax payable (SOFP)

9.3 Often the actual amount of tax paid will be different from the amount that was recorded in
the financial statements.
This over or under provision is simply adjusted in the next financial statements.

395
20: INTRODUCTION TO COMPANY ACCOUNTING

Lecture example 5 Preparation question

Lauren Ltd has a year end of December.


When preparing its financial statements for the year ended 31 December 20X5, Lauren Ltd
estimated that its income tax payable would be $62,000.
Lauren Ltd settled this tax liability on 30 September 20X6, paying $65,000. The tax estimate for the
year ended 31 December 20X6 is $43,000.
Required
(1) Record the tax entries for the years ended 31 December 20X5 and 20X6 in the ledger
accounts.
(2) Prepare the tax note which relates to the income statement for the year ended 31 December
20X6.

Solution
(1) Income tax expense (I/S)

$ $

Current tax payable (SOFP)

$ $

(2) Tax note for the year ended 31 December 20X6

396
20: INTRODUCTION TO COMPANY ACCOUNTING

10 Comparison
The following table shows a comparison between a sole trader and a limited liability
company.

Sole trader Company


Ownership The proprietor owns the business. There are often a large number of
owners, who are called
shareholders or members.
Liability The proprietor has unlimited legal Members/shareholders have
liability regarding the business. limited liability. This means that
they are only liable to the extent of
their investment in the business.
Legal status The business and the proprietor A company is a separate legal
share legal identity (although the entity.
business is a separate business
entity for reporting purposes).
Management The proprietor usually owns and Members/shareholders do not
manages the business. usually manage the business, but
appoint a Board of Directors to run
the company on their behalf.
Profits The proprietor takes 'drawings' out Members/shareholders receive
of the business. profits in the form of dividends.
Any cash amounts taken as a The remainder of the profits are
salary are not an expense of the retained in the company. The
business but drawings. directors receive a salary from the
company and this is an expense in
the income statement.
Taxation Business profits are taxed in the Income tax is paid on the
hands of the proprietor, using company profits.
individual's tax rates.
Statement of The middle of the statement of The middle of the statement of
financial financial position is split into financial position is split into 'share
position 'opening capital', 'profits' and capital' and 'reserves'.
'drawings'.
Legal There are no legal requirements There are extensive legal
requirements specific to a sole trader. requirements governing limited
companies.
Other The business is closed to outside Investors can invest in a company.
investors.

397
20: INTRODUCTION TO COMPANY ACCOUNTING

11 Chapter summary
Section Topic Summary
Quick Quiz
1 Introduction Companies use the same method of bookkeeping to
record transactions. There are however some
differences in the terminology and the formats used.
2 Proforma financial The format in which companies must produce their
statements financial statements is prescribed by the accounting
standard IAS 1.
3 Share capital An entity may issue two main types of shares. Ordinary
or equity shareholders have voting rights and therefore
have control over the company. Preference
shareholders are really just providers of finance to the
business and have limited rights.
4 Share capital: In a limited liability company the shareholders own the
accounting treatment business. A company may raise finance by issuing new
share capital. Where shares are issued at a premium
to their nominal value, the premium is recorded in the
share premium account.
A bonus issue is where the company issues shares for
no cash consideration. With a rights issue, shares are
issued for cash but the price charged is slightly lower
than the current market price.
5 Reserves A company may have several different types of reserve
such as a share premium account, a revaluation
reserve and retained earnings.
6 Dividends Shareholders may receive a dividend as a return on
their investment; these are accounted for as a
deduction to retained earnings.
7 Long term borrowings A company may also raise finance by issuing debt such
as loan notes or debentures.
8 Finance costs It will have to pay interest on any debt that it issues
and this will be shown as 'finance costs' in the
income statement.
9 Current tax Companies pay corporation tax on their profits.
10 Comparison Sole traders and partnerships are very similar in their
nature whilst companies are quite different. You must
ensure that you are happy with both the differences and
similarities.

398
20: INTRODUCTION TO COMPANY ACCOUNTING

12 Double Entry Summary for Chapter 20


12.1 Adjustment to record dividends:

Dr Retained earnings (SOFP)


Cr Dividends payable (SOFP)

12.2 Adjustment to record finance costs:

Dr Finance costs (I/S)


Cr Bank (SOFP)

12.3 Adjustment to record the income tax expense:

Dr Income tax expense (I/S)


Cr Current tax payable (SOFP)

399
20: INTRODUCTION TO COMPANY ACCOUNTING

Chapter Summary
Finance costs

• Interest is shown as a finance cost in the income statement

Reserves Long term borrowings Income taxes

• Debt finance, for example: • Expense in the income statement and a


• Share premium account - debentures liability at the year end
• Revaluation reserve - loan notes • Dr Income tax expense
• Other reserves Cr Current tax payable
• Retained earnings: • Any under/ over provision is adjusted in
- cumulative undistributed profits the next year's financial statements

Introduction to
company accounting
Four main types of share capital: • Ordinary shares:
- authorised - equity share
- issued Shares - voting rights
- called up - no right to a dividend
- paid up • Preference shares:
- receive a fixed rate of dividends
Accounting treatment - no voting rights

Issue at a premium Bonus issue Rights issue

'When shares are issued at a premium 'Shares are issued for no cash consideration' 'Shares issued to existing shareholders for cash'
to their nominal value, the excess • Always done at nominal value • Issued at rights price which is below current
should be credited to the share • Dr Reserves (SPA) market price
premium account' Cr Share capital • Dr Cash
• Dr Cash • Advantages Cr Share capital
Cr Share capital - enables company to use the share premium Cr Share premium account
Cr Share premium account • Advantages
- price of shares will fall making them more - cost effective way for company to raise finance
affordable to new investors - if all rights are taken up shareholders will
• Disadvantage: maintain their percentage shareholding
- rationale is not always understood by • Disadvantages
shareholders - 'bad press' for the company if all rights are not
taken up
Dividends - effect on future dividend policy

• Dividends on ordinary shares and most preference shares are an appropriation of profits and are debited to retained earnings
• Dividends relating to redeemable preference shares are a finance cost in the income statement

400
Chapter 20: Questions

401
20: QUESTIONS

20.1 A company has an authorised share capital of 1,000,000 50c ordinary shares and an issued share capital
of 800,000 50c ordinary shares.
If an ordinary dividend of 5% is declared what is the amount payable to shareholders? $
(1 mark)

20.2 If a shareholder in a limited liability company sells his shares to another private investor, for less than he
paid for them, the share capital of the company will
A Remain unchanged
B Increase by the nominal value of the shares
C Increase by the amount received for the shares
D Decrease by the nominal value of the shares (2 marks)

20.3 A company’s issued share capital consists of $100,000 in 6% $1 preference shares and $50,000 in 50c
ordinary shares. The directors wish to pay an ordinary dividend for the year of 5 cents per share. What is
the company’s total dividend for the year?
A $8,500
B $11,000
C $5,000
D $17,000 (2 marks)

402
Chapter 20: Answers

403
20: ANSWERS

20.1 $20,000
5% × (800,000 × 0.50)

20.2 A

20.3 B ($100,000 × 6%) + ($50,000 × 2 × 5c)

END OF CHAPTER

404
21
Preparation of
financial statements for
companies

Syllabus Guide Detailed Outcomes


Having studied this chapter you will be able to:
• Recognise how the accounting equation and business entity convention underlie the statement of financial
position.
• Understand the nature of reserves and report them in a company statement of financial position.
• Prepare extracts of a statement of financial position from given information.
• Understand why the heading 'retained earnings' appears in a company statement of financial position.
• Prepare extracts of an income statement from given information.
• Understand how accounting concepts apply to revenue and expenses.
• Calculate revenue, cost of sales, gross profit and net profit from given information and disclose items of income
and expenditure in the income statement.
• Record income taxes in the income statement of a company.
• Understand the inter-relationship between the statement of financial position and income statement.
• Identify items requiring separate disclosure on the face of the income statement.
• Identify the components of the statement of changes in equity.

Exam Context
Whilst you will not be required to produce an entire income statement, statement of financial position or statement of
changes in equity you may be asked to calculate individual elements of each statement. A question on the Pilot Paper
required you to demonstrate understanding of what was included in the statement of changes in equity.

Qualification Context
The topics covered in this chapter are developed further in the Fundamentals level paper Financial Reporting (F7). Here
you will need to produce financial statements using the format specified by IAS 1. You will also learn how accounting
standards such as IFRS 5 affect the presentation of the financial statements if, for example, a company discontinues
part of its operations.

Business Context
Financial statements are used by a wide range of user groups to make decisions, for example whether or not to buy
shares in a company. Financial statements need to be prepared in a consistent way in order for users to be able to
compare different companies. The notes to the accounts will also provide a lot more detail on the headline figures shown
in the income statement and statement of financial position.

405
21: PREPARATION OF FINANCIAL STATEMENTS FOR COMPANIES

Overview
Statement of financial
Income statement position

Preparation of financial statements


for companies

Statement of Notes to the accounts


changes in equity

406
21: PREPARATION OF FINANCIAL STATEMENTS FOR COMPANIES

1 Introduction
1.1 As stated in Chapter 20 the financial statements of a limited liability company are subject to
regulation and must follow a prescribed format.

1.2 Much of the prescribed format is determined by IAS 1 (revised). This accounting standard
states what should be included in a set of financial statements and how they should be
presented.
A complete set of financial statements in accordance with IAS 1 (revised) comprises:
(a) a statement of financial position
(b) an income statement
(c) a statement of comprehensive income
(d) a statement of changes in equity
(e) a statement of cash flows; and
(f) notes, comprising a summary of significant accounting policies and other explanatory
notes.

2 Proforma financial statements


2.1 Income statement for the year ended 31 March 20X7
$'000
Revenue X
Cost of sales (X)
Gross profit X
Other income X
Distribution costs (X)
Administrative expenses (X)
Finance costs (X)
Profit before tax X
Income tax expense (X)
Profit for the year X

407
21: PREPARATION OF FINANCIAL STATEMENTS FOR COMPANIES

2.2 Statement of financial position as at 31 March 20X7


$'000
ASSETS
Non-current assets
Property, plant and equipment X
Other intangible assets X
X
Current assets
Inventories X
Trade receivables X
Other current assets X
Cash and cash equivalents X
X
Total assets X

EQUITY AND LIABILITIES


Equity
Share capital X
Share premium account X
Revaluation reserve X
Retained earnings X
X
Non-current liabilities
Long term borrowings X
Long term provisions X

Current liabilities
Trade payables X
Short term borrowings X
Current tax payable X
Short term provisions X
Total equity and liabilities X

408
21: PREPARATION OF FINANCIAL STATEMENTS FOR COMPANIES

2.3 Illustration
Below are the income statement and statement of financial position for Arrow Co for the
year ended 30 September 20X6
Arrow Co
Income statement for the year ended 30 September 20X6
$'000
Revenue 12,740
Cost of sales (7,040)
Gross profit 5,700
Distribution costs (2,060)
Administrative expenses (2,375)
Finance costs (72)
Profit before tax 1,193
Income tax expense (270)
Profit for the year 923
Arrow Co
Statement of financial position as at 30 September 20X6
$'000
ASSETS
Non-current assets
Property, plant and equipment 5,000
5,000
Current assets
Inventories 610
Trade receivables 1,000
Cash and cash equivalents 1,170
2,780
Total assets 7,780

EQUITY AND LIABILITIES


EQUITY
Share capital 1,750
Share premium account 585
Revaluation reserve 1,400
Retained earnings 1,873
5,608
Non-current liabilities
Long term borrowings 1,200
1,200
Current liabilities
Trade payables 550
Other payables 72
Current tax payable 270
Short term provisions 80
972
Total equity and liabilities 7,780

409
21: PREPARATION OF FINANCIAL STATEMENTS FOR COMPANIES

The following information was accounted for when the above financial statements were
produced:
(1) During the year the company made a rights issue on a 1 for 6 basis. The issue was
fully subscribed and the rights price was $1.27. Prior to the rights issue Arrow Co had
3,000,000 50c ordinary shares in issue.
(2) The property, plant and equipment were revalued by $600,000 during the year.
(3) A dividend of $300,000 was paid during the year.

2.4 Statement of comprehensive income


Financial statements have always included an income statement and a statement of
financial position
Following a revision to IAS 1, financial statements should also now include a statement of
comprehensive income.
This statement shows all of the realised gains and losses from the income statement and
the unrealised gains and losses from the statement of financial position in one statement
of performance.

Income statement Statement of financial position

Realised Unrealised
gains and losses gains and losses

e.g. profit for the year e.g. revaluation gains/losses

Statement of comprehensive income

The statement can be presented in one of two ways:


• as one single statement (proforma 1)
• as two separate statements (proforma 2)

410
21: PREPARATION OF FINANCIAL STATEMENTS FOR COMPANIES

Proforma 1 – one single statement

Statement of comprehensive income for the year ended 31 March 20X7


20X7 20X6
$’000 $’000
Revenue X X
Cost of sales (X) (X)
Gross profit X X
Other income X X
Distribution costs (X) (X)
Administrative expenses (X) (X)
Finance costs (X) (X)
Investment income X X
Profit before tax X X
Income tax expense (X) (X)
Profit for the year X X
Other comprehensive income:
Gains on property revaluation X X
Total comprehensive income for the year X X

Proforma 2 – two separate statements

Income statement for the year ended 31 March 20X7


20X7 20X6
$’000 $’000
Revenue X X
Cost of sales (X) (X)
Gross profit X X
Other income X X
Distribution costs (X) (X)
Administrative expenses (X) (X)
Finance costs (X) (X)
Investment income X X
Profit before tax X X
Income tax expense (X) (X)
Profit for the year X X

Statement of comprehensive income for the year ended 31 March 20X7

20X7 20X6
$’000 $’000
Profit for the year X X
Other comprehensive income:
Gains on property revaluation X X
Total comprehensive income for the year X X

411
21: PREPARATION OF FINANCIAL STATEMENTS FOR COMPANIES

Lecture example 1 Technique demonstration

Using the illustration in Section 2.3, prepare the statement of comprehensive income for the year
ended 30 September 20X6:
(a) showing the statement as one statement
(b) showing the statement as two separate statements.

Solution
(a) One single statement
Statement of comprehensive income for the year ended 30 September 20X6

$’000
Revenue 12,740
Cost of sales (7,040)
Gross profit 5,700
Distribution costs (2,060)
Administrative expenses (2,375)
Finance costs (72)
Profit before tax 1,193
Income tax expense (270)
Profit for the year 923
Other comprehensive income:
Gains on property revaluation
Total comprehensive income for the year

(b) Two separate statements


Income statement for the year ended 30 September 20X6

$’000
Revenue 12,740
Cost of sales (7,040)
Gross profit 5,700
Distribution costs (2,060)
Administrative expenses (2,375)
Finance costs (72)
Profit before tax 1,193
Income tax expense (270)
Profit for the year 923

412
21: PREPARATION OF FINANCIAL STATEMENTS FOR COMPANIES

Statement of comprehensive income for the year ended 30 September 20X6

$’000
Profit for the year 923
Other comprehensive income:
Gains on property revaluation

Total comprehensive income for the year

2.5 Statement of changes in equity


Proforma
Share Share Reval- Retained Total
capital premium uation earnings equity
account reserve
$’000 $’000 $’000 $’000 $’000
Balance at 31 March 20X6 X X X X X
Changes in accounting policy _ _ _ X X
Restated balance X X X X X
Issue of share capital X X X
Dividends (X) (X)
Total comprehensive income _ _ X X X
Balance at 31 March 20X7 X X X X X

Lecture example 2 Technique demonstration

Arrow had the following equity balances at 1 October 20X5 (the beginning of the year):
$'000
Share capital – 50c ordinary shares 1,500
Share premium account 200
Revaluation reserve 800
Retained earnings 1,250
3,750
Required
Using the information from the illustration in Section 2.3, produce a statement of changes in equity
for Arrow for the year ended 30 September 20X6.

413
21: PREPARATION OF FINANCIAL STATEMENTS FOR COMPANIES

Solution
Share Share Reval- Retained Total
capital premium uation earnings equity
account reserve
$’000 $’000 $’000 $’000 $’000
Balance at 30 September 20X5

Issue of share capital

Dividends

Total comprehensive income

Balance at 30 September 20X6

Working

414
21: PREPARATION OF FINANCIAL STATEMENTS FOR COMPANIES

3 Notes to the accounts


Notes are included in a set of financial statements to give users extra information. You
should be aware of the following notes:

3.1 Property, plant and equipment (Chapter 9)


Land and Machinery Office Total
buildings equipment
$ $ $ $
Net book value at 1 April 20X6 X X X X
Additions X X X X
Revaluation surplus X – – X
Depreciation charge (X) (X) (X) (X)
Disposals (X) (X) (X) (X)
Net book value at 31 March 20X7 X X X X

At 31 March 20X7
Cost or valuation X X X X
Accumulated depreciation (X) (X) (X) (X)
Net book value X X X X

At 31 March 20X6
Cost or valuation X X X X
Accumulated depreciation (X) (X) (X) (X)
Net book value X X X X

3.2 Intangible non-current assets (Chapter 10)


Development
expenditure
$
Net book value at 1 April 20X6 X
Additions X
Amortisation charge (X)
Disposals (X)
Net book value at 31 March 20X7 X

At 31 March 20X7
Cost X
Accumulated amortisation (X)
Net book value X

At 31 March 20X6
Cost X
Accumulated amortisation (X)
Net book value X

415
21: PREPARATION OF FINANCIAL STATEMENTS FOR COMPANIES

3.3 Provisions (Chapter 13)


$
At 1 April 20X6 X
Increase in period X
Released in period (X)
At 31 March 20X7 X

3.4 Contingent liabilities (Chapter 13)


Unless remote, disclose for each contingent liability:
(a) a brief description of its nature; and where practicable
(b) an estimate of the financial effect
(c) an indication of the uncertainties relating to the amount or timing of any outflow; and
(d) the possibility of any reimbursement

3.5 Contingent assets (Chapter 13)


Where an inflow of economic benefits is probable, an entity should disclose
(a) a brief description of its nature; and where practicable
(b) an estimate of the financial effect

3.6 Events after the reporting period (Chapter 22)


In respect of non-adjusting events after the reporting period disclose
(a) the nature of the event
(b) an estimate of its financial effect (or a statement that an estimate cannot be made).

416
21: PREPARATION OF FINANCIAL STATEMENTS FOR COMPANIES

4 Chapter summary
Section Topic Summary
1 Introduction The financial statements published by a company need
to follow the format prescribed by IAS 1 (revised).
2 Proforma financial You will not be required to produce an income
statements statement or statement of financial position but should
be aware of their contents.
The statement of comprehensive income is a new
performance statement which brings together the
realised gains and losses from the income statement
and the unrealised gains and losses from the statement
of financial position.
The statement of changes in equity shows the
movements on each of the accounts in the equity
section of the statement of financial position in a
separate statement.
3 Notes to the accounts The purpose of the notes to the accounts is to provide
additional information of key financial statement figures.

417
21: PREPARATION OF FINANCIAL STATEMENTS FOR COMPANIES

Chapter Summary
Statement of financial
Income statement position

'Shows the income and expenses for a period


under specific headings' 'Shows the assets and liabilities of a business at
• Points to note: a point in time'
- distribution costs: delivery costs
- administrative expenses: general costs
that do not 'fit' under the other captions
- finance costs: bank interest,
debenture/loan note interest
- income tax expense: estimate of income
tax due on the profits for the period
plus/minus any under/over provision in
respect of prior periods

Preparation of financial statements


for companies

Statement of Notes to the accounts


changes in equity

'Explains the movements between the equity • Examinable notes:


section of the statement of financial position at – property, plant and equipment
the beginning and the end of the year' – intangible non-current assets
• Key components: – provisions
– issue of share capital – contingent liabilities
– dividends (on ordinary shares) – contingent assets
– total comprehensive income for the year – events after the reporting period
• profit for the year
• revaluation surplus on non-current
assets

418
Chapter 21: Questions

419
21: QUESTIONS

21.1 Which of the following items impact on the Statement of Changes in Equity?
(i) Issue of ordinary shares
(ii) Revaluation of a building
(iii) Profit for the period
(iv) Revaluation of a non-current asset investment
A (i)
B (i), (iii)
C (ii), (iii)
D All of the above (2 marks)
21.2 Spend Co
The following balances remain in the books of Spend Co at 30 June 20X8 after the preparation of the
trading account.
$
Share capital
80,000 $1 ordinary shares 80,000
40,000 8% $1 preference shares 40,000
Share premium account 10,000
Revaluation reserve 30,000
Inventories at 30 June 20X8 83,852
Trade receivables and prepayments 27,200
Trade payables and accruals 13,722
Bank balance 7,796
10% debentures 16,000
General reserve 28,000
Irrecoverable debts 340
Gross profit for the period 81,508
Wages and salaries 28,200
Insurance 1,410
Postage and telephone 620
Light and heat 1,216
Debenture interest (½ year to 31 December 20X7) 800
Directors fees 2,500
General expenses 3,108
Vehicles (cost $19,400) 6,800
Office furniture and equipment (cost $44,640) 27,440
Land and buildings at valuation 132,200
Retained earnings at 1 July 20X7 24,252
The following information is also available:
(1) The land and buildings are to be revalued at $150,000;
(2) Office furniture and equipment is to be depreciated at 15% on cost, and vehicles at 20% on cost;
(3) A bill for $348 in respect of electricity consumed up to 30 June 20X8 has not been entered in the
ledger;
(4) The amount for insurance includes a premium of $300 paid in December 20X7 to cover the
company against fire loss for the year 1 January 20X8 to 31 December 20X8;

420
21: QUESTIONS

(5) Provisions are to be made for:


$
Directors’ fees 5,000
Audit fee 1,200
The outstanding debenture interest.
(6) The directors made the following recommendations prior to the year end which have not yet been
adjusted for:
(i) $12,000 should be transferred to a general reserve;
(ii) the preference dividend should be accrued for payment;
Required
Prepare the income statement from the gross profit line downwards for the period ended 30 June 20X8
and a statement of financial position as at that date (ignore income tax).

421
21: QUESTIONS

422
Chapter 21: Answers

423
21: ANSWERS

21.1 D
21.2 Spend Co
Spend Co
Income statement for the period ended 30 June 20X8
$ $
Gross profit for the period 81,508
Less expenses:
Irrecoverable debts 340
Wages and salaries 28,200
Insurance (1,410 – (300 x 6/12)) 1,260
Postage and telephone 620
Light and heat (1,216 + 348) 1,564
Debenture interest (800 + 800) 1,600
Directors’ fees (2,500 + 5,000) 7,500
Audit fee 1,200
General expenses 3,108
Depreciation: Office furniture and equipment 6,696
Vehicles 3,880
55,968
Profit for the period 25,540

Spend Co
Statement of financial position as at 30 June 20X8
Cost or Acc. NBV
Valuation Dep'n
$ $ $
NON-CURRENT ASSETS
Land and buildings 150,000 – 150,000
Furniture and equipment 44,640 23,896 20,744
Motor vehicles 19,400 16,480 2,920
214,040 40,376 173,664
CURRENT ASSETS
Inventories 83,852
Trade receivables and prepayments (27,200 + (300 × 6/12)) 27,350
Cash and cash equivalents 7,796
118,998
292,662

EQUITY
Share capital
80,000 $1 ordinary shares 80,000
40,000 8% $1 preference shares 40,000
Share premium account 10,000
Revaluation reserve (30,000 + 17,800) 47,800
General reserve (28,000 + 12,000) 40,000
Retained earnings (Working) 34,592
252,392
NON-CURRENT LIABILITIES
10% debentures 16,000

CURRENT LIABILITIES
Trade payables and accruals (13,722 + 5,000 + 1,200 + 800 + 348) 21,070
Dividends payable 3,200
24,270
292,662

424
21: ANSWERS

Working

Retained earnings $
Retained earnings at 1 July 20X7 24,252
Profit for the period 25,540
Dividends declared
8% preference dividend (3,200)
Transfer to general reserve (12,000)
Retained earnings at 30 June 20X8 34,592

425
21: ANSWERS

END OF CHAPTER

426
Events after the
reporting period 22
Syllabus Guide Detailed Outcomes
Having studied this chapter you will be able to:
• Define an event after the reporting period in accordance with International Financial Reporting Standards.
• Classify events as adjusting or non-adjusting.
• Distinguish between how adjusting and non-adjusting events are reported in the financial statements.

Exam Context
Questions on this topic are likely to require you to identify adjusting and non-adjusting events from a list of options and
the appropriate accounting treatment of each event. Both these types of questions were tested in the Pilot Paper.

Qualification Context
The knowledge in this chapter is tested again at the Professional level paper, Corporate Reporting (P2) where you will
be expected to consider how events after the reporting period may impact the way in which transactions are reported.

427
22: EVENTS AFTER THE REPORTING PERIOD

Overview

Definition

Events after the


reporting period

Adjusting events Non-adjusting events

428
22: EVENTS AFTER THE REPORTING PERIOD

1 Definition
1.1 Events after the reporting period: events, both favourable and unfavourable, that occur
between the end of the reporting period and the date when the financial statements are
authorised for issue.

1.2 There are two types of event after the balance sheet date.

2 Adjusting and non-adjusting events


2.1
Adjusting events Non-adjusting events

• Events which provide evidence of • Events that relate to conditions which


conditions which existed at the end of the arose after the end of the reporting
reporting period. period

• Examples: • Examples:
(1) resolution of a court case (1) destruction of major asset, eg by
(2) bankruptcy of a major customer flood or fire
(3) evidence of NRV of inventories (2) major share transactions
(4) discovery of fraud or errors that show (3) announcement of a plan to close part
the financial statements were incorrect of a business

• Accounting treatment: • Accounting treatment:


Change the amounts in the financial Disclose non-adjusting event in a note to
statements the financial statements

2.2 (a) Dividends proposed or declared after the end of reporting period but before the
financial statements are approved should be disclosed in a note to the financial
statements.
(b) A non-adjusting event that affects going concern becomes an adjusting event.

429
22: EVENTS AFTER THE REPORTING PERIOD

Lecture example 1 Exam standard for 2 marks

Which of the following events after the reporting period would normally qualify as a non-adjusting
event?
1 A fall in the market price of shares held by the entity as investments.
2 Insolvency of a trade receivable with a balance of $200,000 outstanding at the end of the
reporting period.
3 Declaration of the year-end dividend by the directors.
4 Confirmation of the amount of damages awarded to an employee who sued for unfair
dismissal after being sacked two months before the year end.
A 2 only
B 1 and 3
C 1, 3 and 4
D 2 and 4

Solution

3 Chapter summary
Quick Quiz
Section Topic Summary
1 Definition Events after the end of the reporting period are events
which occur between the end of the reporting period
and the date the financial statements are approved for
issue.
2 Adjusting and non- There are two types: adjusting and non-adjusting.
adjusting events
Adjusting events provide evidence of conditions that
existed at the end of the reporting period. The financial
statements should be changed to include this
information.
Non-adjusting events relate to conditions which
arose after the end of the reporting period. These
should be disclosed as a note to the financial
statements.

430
22: EVENTS AFTER THE REPORTING PERIOD

Chapter Summary

Definition

'Events, both favourable and unfavourable, that occur between the end of the
reporting period and the date when the financial statements are authorised for

Events after the


reporting period

Adjusting events Non-adjusting events

'Events which provide evidence of conditions 'Events that relate to conditions which arose
which existed at the end of the reporting after the end of the reporting period'
period'
• Disclose in a note to the financial
• Include in the financial statements statements
• Examples: • Examples:

– Resolution of a court case - Destruction of a major asset by flood


or fire
– Bankruptcy of a major customer
- Major share transactions
– Evidence of the NRV of inventories
- Announcement of a plant to close part
– Discovery of fraud or errors
of a business
- Dividends proposed/declared after the
end of the reporting period

431
22: EVENTS AFTER THE REPORTING PERIOD

432
Chapter 22: Questions

433
22: QUESTIONS

22.1 The following are examples of events which might occur between the end of the reporting period and the
date on which the financial statements are authorised for issue:
(1) Losses on inventories as a result of a catastrophe such as a fire or flood after the year end
(2) The discovery of fraud which shows that the financial statements were incorrect
(3) Revaluations of property which provide evidence of an impairment in value
Which of the examples given should normally be classified as an adjusting event?
A (1), (2) and (3)
B (1) and (2)
C (1) and (3)
D (2) and (3) (2 marks)

22.2 Robin Co has a year end of 31 December 20X8, the directors were informed on 27 February 20X9 that a
serious fire at one of the company's factories would stop production there for at least six months to come.
On 3 March 20X9 the directors of Robin Co were informed that a major customer had gone into
liquidation. The liquidator was pessimistic about the prospect of recovering anything for unsecured
creditors. The financial statements for the year ended 31 December 20X8 were approved on 20 March
20X9.
In accordance with IAS 10, Events after the reporting period, how should the two events be treated in the
financial statements?
Fire Liquidation
A Accounts adjusted Disclosed in notes
B Disclosed in notes Disclosed in notes
C Accounts adjusted Accounts adjusted
D Disclosed in notes Accounts adjusted (2 marks)

22.3 A Co has a year end of 31 December 20X7. During the preparation of the financial statements in March
20X8 the following issues arose:
(1) Sales of a particular inventory line were poor during the second half of 20X7. The directors had
hoped that sales would pick up in 20X8 but it is now apparent that the inventory will need to be
marked down below their original cost in order to sell them.
(2) On 12 February 20X8 one of the company's production plants was struck by lightening. The
company will suffer a net loss of $55,000 as a result of this.
(3) Sporran Co is a valued customer which owed A Co $34,000 at the balance sheet date, although
they were behind with their payments. Since the year end sales to Sporran Co were $12,000. The
directors have just received notification that Sporran Co has gone into liquidation.
How should the above events be classified according to IAS 10 Events after the reporting period?
Adjusting Non-adjusting
event event
A 2,3 1
B 1, 2 3
C 1,3 2
D 1, 2, 3 (2 marks)

434
Chapter 22: Answers

435
22: ANSWERS

22.1 D (1) After the end of the reporting period and therefore non-adjusting.
(2) The financial statements are incorrect, therefore clearly we must adjust.
(3) The impairment is assumed to have taken place by the end of the reporting period. We
simply did not find out until later.

22.2 D The fire is a non-adjusting event as it does not affect the value of the building at 31 December
20X8. It is therefore only disclosed in a note to the financial statements unless it threatens the
company's going concern in which case it would become an adjusting event.
The customer is assumed to be insolvent at 31 December 20X8. We simply did not know this and
therefore it is an adjusting event and it should be adjusted for.

22.3 C

END OF CHAPTER

436
Statements of cash flows 23
Syllabus Guide Detailed Outcomes
Having studied this chapter you will be able to:
• Differentiate between profit and cash flows and understand the need for management to control cash flow.
• Recognise the benefits and drawbacks to users of the financial statements of a statement of cash flows.
• Classify the effect of transactions on cash flows and how they should be treated in a company's statement of
cash flows.
• Calculate the figures needed for the statement of cash flows including cash flows from operating, investing and
financing activities.
• Calculate the cash flow from operating activities using the direct and indirect method.
• Prepare extracts from statements of cash flows from given information.

Exam Context
Questions on this chapter are likely to focus on whether you can identify which items should and should not go into the
statement of cash flows and also on performing basic calculations. For example, you may be asked to calculate figures
such as the cash generated from operations from given information or the cash paid to acquire property, plant and
equipment.

Qualification Context
The knowledge covered in this chapter is developed in the Fundamentals level paper Financial Reporting (F7) where
you will have to produce a statement of cash flows in full. This is likely to involve more complex areas such as cash
flows related to non-current assets held on finance leases. You will also need to be able to interpret a cash flow. Group
statements of cash flows are examined in the Professional level paper Corporate Reporting (P2).

Business Context
The ability to generate cash is key to the survival of an entity. Whilst directors may use cash budgets to estimate future
cash flows, the statement of cash flows shows an historic record of how cash has been generated and where it was
spent.
Cash is not subject to manipulation through an entity's choice of accounting policies. It is therefore a reliable measure of
performance that is relevant to users of the financial statements.

437
23: STATEMENTS OF CASH FLOWS

Overview
Cash Cash equivalents

Cash flows

Statements of
cash flows

IAS 7

Cash flows from Cash flows from Cash flows from


operating activities investing activities financing activities

Indirect method Direct method

438
23: STATEMENTS OF CASH FLOWS

1 Purpose
1.1 To show the effect of a company’s commercial transactions on its cash balance.
It is thought that users of accounts can readily understand cash flows, as opposed to
income statements and statements of financial position which are subject to manipulation by
the use of different accounting policies.
Cash flows are used as an investment appraisal method such as net present value and
hence a cash flow statement gives potential investors a method with which to evaluate a
business.

2 IAS 7: Statements of cash flows


2.1 IAS 7 splits cash flows into the following headings:
• Cash flows from operating activities
• Cash flows from investing activities
• Cash flows from financing activities

Definitions
2.2 (a) Cash (b) Cash equivalents

• cash on hand • short term, highly liquid


investments
• demand deposits • readily convertible to
known amounts of cash
• insignificant risk of
changes in value

e.g. treasury bills

(c)

Cash flows

• inflows and outflows of cash and cash equivalents

439
23: STATEMENTS OF CASH FLOWS

2.3 XYZ CO
Statement of cash flows for the year ended 31 December 20X7 (indirect method)
$000 $000
Cash flows from operating activities
Profit before taxation 3,390
Adjustment for:
Depreciation 450
Investment income (500)
Interest expense 400
3,740
Increase in trade and other receivables (500)
Decrease in inventories 1,050
Decrease in trade payables (1,740)
Cash generated from operations 2,550
Interest paid (270)
Income taxes paid (900)

Net cash from operating activities 1,380

Cash flows from investing activities


Purchase of property, plant and equipment (900)
Proceeds from sale of equipment 20
Interest received 200
Dividends received 200

Net cash used in investing activities (480)

Cash flows from financing activities


Proceeds from issue of share capital 250
Proceeds from long-term borrowings 250
Dividends paid* (1,290)

Net cash used in financing activities (790)

Net increase in cash and cash equivalents 110


Cash and cash equivalents at beginning of period 120
Cash and cash equivalents at end of period 230

* This could also be shown as an operating cash flow.

440
23: STATEMENTS OF CASH FLOWS

3 Cash flows from operating activities


3.1 These represent cash flows derived from operating or trading activities.
An entity should report cash flows from operating activities using either:
Section 1.7.1
(a) The direct method, whereby major classes of gross cash receipts and payments are
disclosed (preferred method per IAS 7 – see Section 6.1), or
(b) The indirect method (as above), whereby reported profit or loss is adjusted for the
effects of transactions of a non cash nature, any accruals or prepayments of
operating expenses, and items relating to investing or financing cash flows.

Income taxes paid


3.2 Income taxes paid may need to be calculated from other data given to you. This is best
achieved by putting the relevant figures into a 'T' account working.

Lecture example 1 Preparation question

In the statements of financial position of Tacks Co as at 31 December 20X9 and 31 December


20X8 were the following amounts for income tax payable.
31 December
20X9 20X8
$ $
Income tax payable 156,000 168,000
The income statement tax charge for 20X9 amounted to $104,000.
Required
What is the amount of income taxes paid during the year? $

Workings
Income tax payable
$'000 $'000

441
23: STATEMENTS OF CASH FLOWS

4 Cash flows from investing activities


4.1 The cash flows included in this section are those related to the acquisition or disposal of any
non-current assets or investments together with returns received in cash from investments,
i.e. dividends and interest. This section shows the extent to which expenditures have been
Section 1.7.2
made for resources intended to generate future income and cash flows.

Lecture example 2 Preparation question

On 31 December 20X8 the value of plant and equipment in the books of Erosion Co was as
follows:
$
Plant and equipment at cost 200,000
Accumulated depreciation 80,000
Plant and equipment at net book value 120,000
On 1 January 20X9 an item of plant was sold for $8,000 which had originally cost $20,000 when
new, but had a net book value of $11,000 at the time of sale. (The statement of financial position
values shown above do not show that this sale has taken place.)
On 31 December 20X9 the value of plant and equipment in the statement of financial position was:
$
Plant and equipment at cost 280,000
Accumulated depreciation 111,000
Plant and equipment at net book value 169,000
Required
Show the relevant entries for property, plant and equipment which would appear in a statement of
cash flows for Erosion Co in 20X9.

Solution
Workings
Plant & equipment – cost
$'000 $'000

Accumulated depreciation
$'000 $'000

442
23: STATEMENTS OF CASH FLOWS

5 Cash flows from financing activities


5.1 Financing cash flows comprise receipts from or repayments to external providers of finance
in respect of principal amounts of finance. Examples of financing cash flows are:
Section 1.7.3 • Cash proceeds from issuing shares
• Cash proceeds from issuing debentures, loans, notes, bonds, mortgages and other
short or long term borrowings
• Cash repayments of amounts borrowed
• Dividends paid to shareholders
In order to calculate such figures the closing statement of financial position figure for debt or
share capital and share premium is compared with the opening position for the same items.

Dividends paid
5.2 The cash outflows included in dividends paid are dividends paid on the reporting company's
equity shares.

Lecture example 3 Preparation question

Distribution Co statement of financial position extract for the year ended 31 December 20X9
20X9 20X8
$'000 $'000
Dividends payable 45 35
Dividends charged to retained earnings were $60,000.
Required
What are the dividends paid during the year ended 31 December 20X9? $
Workings
Dividends payable
$'000 $'000

443
23: STATEMENTS OF CASH FLOWS

Lecture example 4 Technique question

The summarised accounts of the Emma Co for the year ended 31 December 20X8 are as follows:
Statement of financial position as at 31 December
20X8 20X7
$'000 $'000
Non-current assets
Property, plant and equipment 628 514

Current assets:
Inventories 214 210
Trade receivables 168 147
Cash 7 –
389 357
1,017 871

Equity
Share capital ($1 ordinary shares) 250 200
Share premium account 70 60
Revaluation reserve 110 100
Retained earnings 314 282
744 642
Non-current liabilities
10% debentures 80 50

Current liabilities
Trade payables 136 121
Income tax payable 39 28
Dividends payable 18 16
Overdraft – 14
193 179
1,017 871

Income statement for the year ended 31 December 20X8


$'000
Revenue 600
Cost of sales 319
Gross profit 281
Other expenses (including depreciation of $42,000) 186
Finance costs (interest paid) 8
Profit before tax 87
Income tax expense 31
Profit for the period 56

444
23: STATEMENTS OF CASH FLOWS

Movement of retained earnings


$000
Balance at 31 December 20X7 282
Profit for the period 56
Dividends (24)
Balance at 31 December 20X8 314

You are additionally informed that there have been no disposals of property, plant and equipment
during the year. The new debentures were issued on 1 January 20X8.
Required
Produce a statement of cash flows for Emma Co for the year ended 31 December 20X8.

Solution
EMMA CO
Statement of cash flows for the year ended 31 December 20X8
$’000 $’000
Cash flows from operating activities
Profit before taxation
Adjustments for:
Depreciation
Interest expense

Increase in trade receivables


Increase in inventories
Increase in trade payables
Cash generated from operations
Interest paid
Income taxes paid
Net cash from operating activities

Cash flows from investing activities


Purchase of property, plant and equipment
Net cash used in investing activities

Cash flows from financing activities


Proceeds from issue of shares
Proceeds from issue of debentures
Dividends paid
Net cash from financing activities

Net increase in cash and cash equivalents


Cash and cash equivalents at beginning of year
Cash and cash equivalents at end of year

445
23: STATEMENTS OF CASH FLOWS

Workings

446
23: STATEMENTS OF CASH FLOWS

6 Cash flows from operating activities using the direct


method
6.1 As noted in Section 3.1, IAS 7 has two methods available under which the statement of cash
flows can be prepared:
• indirect method (seen previously)
• direct method

6.2 The only difference is the direct method derives the 'cash generated from operations' figure
in a different way. The operating element of the statement of cash flows should be shown as
follows:
$000 $000
Cash flows from operating activities
Cash receipts from customers 30,150
Cash payments to suppliers and employees (27,600)
Cash generated from operations 2,550
Interest paid (270)
Income taxes paid (900)

Net cash from operating activities 1,380

Cash received from customers


6.3 This represents cash flows received during the accounting period in respect of sales.

Cash payments to suppliers and employees


6.4 This represents cash flows made during the accounting period in respect of goods and
services and amounts paid to employees.

447
23: STATEMENTS OF CASH FLOWS

Lecture example 5 Technique question

Required
Using the information in Lecture example 4 produce the 'cash flows from operating activities'
section of the cash flow statement using the direct method.

Solution
EMMA CO
Statement of cash flows for year ended 31 December 20X8 (extract)
$ $
Cash flows from operating activities
Cash receipts from customers
Cash payments to suppliers and employees
Cash generated from operations
Interest paid
Income taxes paid
Net cash used in operating activities

448
23: STATEMENTS OF CASH FLOWS

7 Chapter summary
Section Topic Summary
Quick Quiz
1 Purpose The statement of cash flows shows the movement
between a company’s cash and cash equivalents at the
beginning and the end of the year.
2 Statements of cash Cash comprises cash on hand and on demand
flows deposits, less bank overdrafts.
(IAS 7)
Cash equivalents are short term, highly liquid
investments such as current asset investments (shares)
which can be converted in to known amounts of cash
relatively quickly without having a major impact on the
entity’s activities.
3 Cash flows from This section of the statement of cash flows shows the
operating activities cash and cash equivalents generated by and used in
the entity’s main trading activities.
4 Cash flows from This section shows the cash flows related to the
investing activities acquisition and disposal of non-current assets and
returns on investments such as interest and dividends
received.
5 Cash flows from Cash flows from financing activities include the monies
financing activities raised from issuing shares and loans and the cash used
in the repayment of loans and the payment of
dividends.
6 Cash flow from The statement of cash flows can be produced using
operating activities one of two methods: the indirect or the direct method.
using the direct method
The direct method provides exactly the same cash flow
information but calculates the cash flow from operating
activities using a slightly different calculation from the
indirect method.

449
23: STATEMENTS OF CASH FLOWS

Chapter Summary
Cash Cash equivalents

'Cash on hand and demand 'Short-term, highly liquid investments'


deposits' • Example:
– current asset investments

Cash flows

'Inflows and outflows of cash and cash equivalents'

Statements of
cash flows

IAS 7

'Requires that a company show the movement in cash and cash equivalents
between the beginning and the end of the year under three headings'
• cash flows from operating activities
• cash flows from investing activities
• cash flows from financing activities

Cash flows from Cash flows from Cash flows from


operating activities investing activities financing activities

'Cash flows from trading activities' 'Cash flows relating to the acquisition or 'Cash flows relating to the issue or repayment
disposal of non-current assets and the of long term finance'
returns on investments' • Includes
Indirect method Direct method • Includes – proceeds from share capital/debenture
– purchase of non-current assets issue
– proceeds from sale of non-current – repayment of loans
• Cash generated from operations assets – ordinary dividends paid
• Adjust profit before tax figure for: – interest/dividends received
– non-cash items
– items shown elsewhere in the
cash flow • Cash generated from operations:
– movements in working capital • Derived by calculating:
• Then deduct interest and income – cash receipts from customers
taxes paid – cash payments to suppliers and employees
• Then deduct interest and income taxes paid

450
Chapter 23: Questions

451
23: QUESTIONS

23.1 In a statement of cash flows which of the items below would not appear as an outflow of cash?
A The nominal value of debenture redeemed at par during the year
B The dividends paid to preference shareholders during the year
C The income statement charge for tax for the year (1 mark)

Data for Questions 23.2 and 23.3


Extracts from a company’s statements of financial position show the following items of property, plant and
equipment at net book value:
30 June
20X7 20X6
$ $
Property, plant and equipment
Freehold property 1,230,000 750,000
Plant and equipment 465,000 380,000
Furniture and fixtures 90,000 105,000
The building element of the freehold property was depreciated by $6,000 and then revalued on 30 June 20X7 by
$95,000. Plant and equipment, which had cost $49,000 when purchased in January 20X2 on which $35,000 of
depreciation had been charged, was disposed of in November 20X6 for $8,000. Depreciation on the plant and
equipment for the year amounted to $37,000. Depreciation of $55,000 has been charged on furniture and
fixtures.

23.2 What is the total figure to be adjusted for in ‘cash flows from operating activities’ in respect of property,
plant and equipment? $ (2 marks)

23.3 What is the total expenditure on property, plant and equipment included under ‘cash flows from investing
activities’? $ (2 marks)

23.4 In a statement of cash flows, a decrease in loan stock would be shown as a cash inflow under 'cash flows
from financing activities'.
A True
B False (1 mark)

23.5 These extracts have been taken from the accounts of Jeanne Co.
Statement of financial position (extracts) 31 October 31 October
20X7 20X6
Current liabilities
Dividends payable 9,750 5,750
Dividends charged to retained earnings during the year were $15,500.
What will appear as “dividends paid” in the statement of cash flows for the year ended 31 October 20X7?
A $5,750
B $11,500
C $15,500
D $21,250 (2 marks)

452
23: QUESTIONS

23.6 Jane Co
Income statement for the year ended 31 December 20X7
$’000
Revenue 2,553
Cost of sales 1,814
Gross profit 739
Distribution costs 125
Administrative expenses 264
Investment income 25
Finance costs 75
Profit before tax 300
Income tax expense 140
Profit for the period 160

Statement of financial position as at 31 December 20X7 20X6


$’000 $’000
Non-current assets
Property, plant and equipment 380 305
Development expenditure 250 200
Investments – 25
630 530
Current assets
Inventories 150 102
Trade receivables 390 315
Short-term investments 50 –
Cash in hand 2 1
592 418
Total assets 1,222 948

Equity
Share capital ($1 ordinary shares) 200 150
Share premium account 160 150
Revaluation reserve 100 91
Retained earnings 160 100
620 491
Non-current liabilities
Long term loan 100 –

Current liabilities
Trade payables 127 119
Bank overdraft 85 98
Income tax payable 190 160
Dividends payable 100 80
502 457
Total equity and liabilities 1,222 948

453
23: QUESTIONS

The following information is available:


(a) The proceeds of the sale of non-current asset investments amounted to $30,000;
(b) Furniture and fixtures, with an original cost of $85,000 and a net book value of $45,000, were sold
for $32,000 during the year;
(c) The current asset investments fall within the definition of cash equivalents under IAS 7;
(d) The following information relates to property, plant and equipment:
20X7 20X6
$’000 $’000
Cost 720 595
Accumulated depreciation 340 290
Net book value 380 305
(e) 50,000 $1 ordinary shares were issued during the year at a premium of 20c per share;
(f) Dividends charged to retained earnings were $100,000 in 20X7;
(g) Development expenditure has not yet started being amortised.
Required
Prepare a statement of cash flows for the year to 31 December 20X7.

454
Chapter 23: Answers

455
23: ANSWERS

23.1 C Income tax paid is a cash flow not the income statement tax charge.

23.2 $104,000
Property, plant and equipment

$’000 $’000
Bal b/d Freehold property 750
Plant & Equipment 380 Disposal – Plant & Equipment
Furniture & Fixtures 105 (49 – 35) 14
Depreciation
Freehold property 6
Plant & Equipment 37
Furniture & Fixtures 55
Revaluation Freehold property 95 98
∴ Acquisitions 567 Bal c/d Freehold property 1,230
Plant & Equipment 465
Furniture & Fixtures 90
1,897 1,897

Total adjustments in the reconciliation:


$’000
Depreciation 98
Loss on disposal of plant and equipment (8 – 14) 6
104

23.3 $567,000
See previous calculation

23.4 B

23.5 B Dividends payable

$ $
Balance b/d 5,750
∴ Paid 11,500
Retained earnings 15,500
Balance c/d 9,750
21,250 21,250

456
23: ANSWERS

23.6 Jane Co
Statement of cash flows for the year ended 31 December 20X7
$’000 $’000
Cash flows from operating activities
Profit before taxation 300
Adjustments for:
Depreciation (W2) 90
Loss on sale of property, plant and equipment (45 – 32) 13
Profit on sale of non-current asset investments (30 – 25) (5)
Investment income (25)
Finance costs 75
448
Increase in trade receivables (390 – 315) (75)
Increase in inventories (150 – 102) (48)
Increase in trade payables (127 – 119) 8
Cash generated from operations 333
Interest received 25
Interest paid (75)
Income taxes paid (W4) (110)
Net cash from operating activities 173

Cash flows from investing activities


Purchase of property, plant and equipment (W1) (201)
Proceeds from sale of property, plant and equipment 32
Proceeds from sale of non-current asset investments 30
Payments for development expenditure (W3) (50)
Net cash used in investing activities (189)

Cash flows from financing activities


Proceeds from issue of ordinary share capital 60
Proceeds from long term loan 100
Dividends paid (W5) (80)
Net cash from financing activities 80

Increase in cash and cash equivalents 64


Cash and cash equivalents at beginning of period (1 – 98) (97)
Cash and cash equivalents at end of period (50 + 2 – 85) (33)
Workings
(W1)
Property, plant and equipment – cost

$’000 $’000
Balance b/d 595 Disposals 85
Revaluation (100 – 91) 9
Additions (bal fig) 201 Balance c/d 720
805 805

(W2)
Property, plant and equipment - Accumulated depreciation

$’000 $’000
Disposals (85 – 45) 40 Balance b/d 290
Balance c/d 340 ∴Depreciation charge 90
380 380

457
23: ANSWERS

(W3)
Development expenditure

$’000 $’000
Balance b/d 200
∴ additions 50 Balance c/d 250
250 250

(W4)
Income tax payable
$’000 $’000
∴ Income tax paid 110 Balance b/d 160
Balance c/d 190 Income statement 140
300 300

(W5)
Dividends payable

$’000 $’000
∴ Dividends paid 80 Balance b/d 80
Balance c/d 100 Retained earnings 100
180 180

END OF CHAPTER

458
Home study chapter –
Information technology 24
Syllabus Guide Detailed Outcomes
Having studied this chapter you will be able to:
• Understanding the basic function and form of accounting records in manual and computerised systems.
• Compare manual and computerised systems and identify advantages and disadvantages of computerised
accounting systems.
• Understand the uses of integrated accounting software packages.
• Understand business use of computers and the nature and purpose of spreadsheets and database systems.

Exam Context
Questions on this topic are likely to focus on the advantages and disadvantages of using a computerised system and the
differences between a manual and a computerised system.

Qualification Context
The importance of accounting systems and internal controls is tested in the Fundamentals level paper, Accountant in
Business (F1).

459
24: HOME STUDY CHAPTER – INFORMATION TECHNOLOGY

Overview

Computerised accounting
packages Integrated software

Accounting modules

Information
technology

Databases Spreadsheets

460
24: HOME STUDY CHAPTER – INFORMATION TECHNOLOGY

1 Introduction
1.1 In today's world most businesses use accounting systems which are computerised, although
some smaller businesses may keep manual records.

1.2 The same principles of double entry are used regardless of whether an accounting system is
manual or computerised.

2 Accounting packages
2.1 There are two main types of computerised accounting packages:
(a) Dedicated accounting packages, for example SAGE.
(b) General software, for example spreadsheets which can be used to keep accounting
records.

2.2 Advantages and disadvantages of computerised accounting packages.

Advantages Disadvantages
(1) Large amounts of data can be (1) Time and cost in setting up the system
processed very quickly and staff training
(2) Computerised systems are more (2) Need for internal controls and security
accurate checks to ensure the accuracy of data
(3) Large volumes of data can be (3) Lack of 'audit trail'
processed
(4) Little training is required (4) Staff may resist the introduction of a
computerised system
(5) Computer can analyse data into
tailored reports

3 Accounting modules
Definition
3.1 Accounting module – a program which deals with one part of a business' accounting
system

3.2 Examples of modules include:


(a) Invoicing
(b) Receivables ledger
(c) Nominal ledger
(d) Payroll
(e) Cash book
(f) Non-current asset register

461
24: HOME STUDY CHAPTER – INFORMATION TECHNOLOGY

Integrated software
3.3 Each module may be integrated with other modules so that when information is recorded in
one module it is automatically updated in another module.
Section 1.5 Examples:
(a) The payroll module may be integrated with the nominal ledger module so that once
the payroll information is determined the associated wages expense is updated in the
nominal ledger.
(b) The invoicing module may be integrated with the inventory, receivables ledger and
nominal ledger modules so that once an invoice is sent the inventory levels are
updated as is the customer's account in the receivables ledger.

3.4 Advantages and disadvantages of integrated software.

Advantages Disadvantages
(1) An entry in one module (1) These systems require more memory than
automatically updates all the a stand-alone system so there is less space
others to store actual data
(2) Reports generated by the (2) Each module may be limited to fewer
system can draw information functions than a specialised module
from all relevant modules (because one program is doing everything)
(3) Reduction in clerical time used (3) An error in one part of the system will flow
to input information and errors through to all areas

4 Databases
Definition
4.1 A database is a 'pool of data' which can be used by any number of applications.

4.2 Examples:
(a) Non-current asset register
(b) List of customers/suppliers
(c) Price lists

462
24: HOME STUDY CHAPTER – INFORMATION TECHNOLOGY

Lecture example 1 Idea generation

What sort of information might be contained in a database file for a non-current asset register?

Solution

4.3 A database should have four major objectives.


(a) It should be shared – different individuals should be able to access the
same information
(b) Its integrity must be preserved – only valid alterations to information should be
made
(c) It should meet the needs of different users.
For example, the accounts department may be interested predominantly in the net book
value of the non-current assets but the production manager will need to know their
whereabouts in order to schedule jobs efficiently
(d) The database must be able to grow and develop according to the needs of the business

5 Spreadsheets
5.1 Spreadsheets are essentially an electronic piece of paper. They are used in all parts of a
business, predominantly to perform numerical calculations.

5.2 Uses of spreadsheets by the accounting function:


(a) To maintain accounting records, for example a cash book
(b) To produce financial statements
(c) To produce budgets/forecasts
(d) To conduct variance analysis

463
24: HOME STUDY CHAPTER – INFORMATION TECHNOLOGY

6 Chapter summary
Section Topic Summary
Quick Quiz
1 Introduction Nowadays most businesses use a computerised
accounting system. However the mechanics of double
entry bookkeeping are the same regardless of whether
a manual or a computerised accounting system is in
place.
2 Accounting packages There are two main types of accounting packages:
dedicated packages such as SAGE and general
software such as spreadsheets.
3 Accounting modules An accounting module is a program which deals with
one part of a business’ accounting system.
These modules may or may not be integrated with other
modules.
4&5 Databases and Databases and spreadsheets are electronic ways of
spreadsheets holding and manipulating information.

464
24: HOME STUDY CHAPTER – INFORMATION TECHNOLOGY

Chapter Summary
• Different accounting modules may be integrated
• Advantages:
- one entry updates all modules
- reduced input time
• Disadvantages:
- an error in one part of the system will flow through to all areas
Computerised accounting
packages Integrated software

• Dedicated accounting packages:


- off the shelf, for example SAGE
- customised
• General software
Accounting modules
- for example: spreadsheets
• Advantages
- speed
- accuracy 'A program which deals with one part
- volume of data of a business' accounting systems'
- computer analysis for reports • Examples:
• Disadvantages - invoicing
- time and cost - receivables ledger
- need for internal controls - nominal ledger
- lack of audit trail - payroll
- non-current asset register

Information
technology

Databases Spreadsheets

'A pool of data which can be used by any 'An electronic piece of paper'
number of applications'. • Used
• Examples - to maintain accounting records
- non-current asset register - to produce budgets/forecasts
- customer/supplier lists - to conduct variance analysis
- price lists

465
24: HOME STUDY CHAPTER – INFORMATION TECHNOLOGY

466
Chapter 24: Questions

467
24: QUESTIONS

24.1 All businesses will apply the same principles of double entry bookkeeping regardless of whether they
operate a manual or a computerised system.
Is this statement true or false?
A True
B False (1 mark)

24.2 If a database is to contain accurate and valid information it should only be amended by authorised
personnel.
Is this statement true or false?
A True
B False (1 mark)

468
Chapter 24: Answers

469
24: ANSWERS

24.1 A

24.2 A

END OF CHAPTER

470
Answers to
Lecture Examples

471
25: ANSWERS TO LECTURE EXAMPLES

Chapter 1
Answer to Lecture Example 1
Users of financial information
(a) Investors
– Profitability
– Future prospects
– Likely risk and return
– Chance of capital growth
– Ability to pay dividends
(b) Employees
– Profitability
– Long-term growth
– Security of their job
– Likelihood of bonus
– Number of employees
– Ability to pay retirement benefits
(c) Lenders
– Whether return on finance will continue to be met
– Other providers and security of their debt
– Likelihood of repayment of capital amount
(d) Suppliers
– Likelihood of payment on time
– Likelihood of payment at all
– Whether they should continue to supply
(e) Customers
– Ability of entity to continue supplying
– Profitability as a measure of value for money of goods bought
(f) Government and their agencies
– Statistics
– Size of company
– Growth rates
– Average payment periods
– Foreign trade
– Profits made
– Corporate income tax liability
– Sales tax liability
(g) Public
– Contribution to local economy
– Information about trends in the prosperity of the entity
– Range of activities provided

472
25: ANSWERS TO LECTURE EXAMPLES

Chapter 2
Answer to Lecture Example 1
A The IASCF appoints members to the IASB, IFRIC and SAC. The SAC advises the IASB on its
agenda.

Answer to Lecture Example 2


A

Chapter 3
Answer to Lecture Example 1
Advantages of historic cost
(1) The transaction cost of $1 million is a very reliable figure which was quantified at the date of
acquisition.
(2) Using current market values for the building may lead to volatility in asset values due to changing
market prices.
(3) Any change in the asset's value will affect the amount of depreciation charged and therefore the
entity's profits. This makes comparability more difficult.
Disadvantages of historic cost
(1) Asset values generally appreciate over time and so using historic cost will mean that the financial
statements contain information which is out of date and therefore less useful for decision making.
(2) Sales revenue and costs will be shown at current prices but depreciation will be based on historic
cost and therefore too low a figure. Profits will therefore look artificially high.

Answer to Lecture Example 2


(a) Historic cost is $1,000
(b) Net realisable value is
$
Selling price (100 × $11) 1,100
Less: completion costs (100 × $2) (200)
900
(c) Show inventory at the lower of cost and net realisable value = $900.

473
25: ANSWERS TO LECTURE EXAMPLES

Chapter 4
Answer to Lecture Example 1
Own
Examples:
(i) House
(ii) Car
(iii) Cash
Owe
Examples:
(i) Mortgage
(ii) Car loan
(iii) Credit card

Chapter 5
Answer to Lecture Example 1
Transaction Debit Credit

(a) Sales for cash Cash Sales


increase asset income

(b) Sales on credit Receivables Sales


increase asset income

(c) Purchases for cash Purchases Cash


expense decrease asset

(d) Purchases on credit Purchases Payables


expense increase liability

(e) Pay electricity bill Electricity Cash


expense decrease asset

(f) Receive cash from a credit customer Cash Receivables


increase assets decrease assets

(g) Pay cash to a credit supplier Payables Cash


decrease liability decrease asset

(h) Borrow money from the bank Cash Loan


increase asset increase liability

474
25: ANSWERS TO LECTURE EXAMPLES

Answer to Lecture Example 2


Cash
$ $
Capital 5,000 Rent 500
Sales 2,100 Electricity 200
Car 1,000
Drawings 300
Capital
$ $
Cash 5,000
Trade payables
$ $
Purchases 2,000
Purchases
$ $
Trade payables 2,000
Rent
$ $
Cash 500
Electricity
$ $
Cash 200
Car
$ $
Cash 1,000
Drawings
$ $
Cash 300
Trade receivables
$ $
Sales 1,750
Sales
$ $
Trade receivables 1,750
Cash 2,100

Answer to Lecture Example 3


Dr Cash Cr
$ $
2/1 Sales 500 1/1 Purchases 300
10/1 Sales 500 25/1 Telephone 50
Bal c/d 650
1,000 1,000
Bal b/d 650

475
25: ANSWERS TO LECTURE EXAMPLES

Answer to Lecture Example 4


Cash
$ $
Capital 5,000 Rent 500
Sales 2,100 Electricity 200
Car 1,000
Drawings 300
Bal c/d 5,100
7,100 7,100
Bal b/d 5,100
Capital
$ $
Bal c/d 5,000 Cash 5,000
5,000 5,000
Bal b/d 5,000
Trade payables
$ $
Bal c/d 2,000 Purchases 2,000
2,000 2,000
Bal b/d 2,000
Purchases
$ $
Trade Payables 2,000 Bal c/d 2,000

Bal b/d 2,000


Rent
$ $
Cash 500 Bal c/d 500

Bal b/d 500


Electricity
$ $
Cash 200 Bal c/d 200

Bal b/d 200


Car
$ $
Cash 1,000 Bal c/d 1,000

Bal b/d 1,000


Drawings
$ $
Cash 300 Bal c/d 300

Bal b/d 300


Trade receivables
$ $
Sales 1,750 Bal c/d 1,750

Bal b/d 1,750

476
25: ANSWERS TO LECTURE EXAMPLES

Sales
$ $
Bal c/d 3,850 Trade receivables 1,750
Cash 2,100
3,850 3,850
Bal b/d 3,850

Chapter 6
Answer to Lecture Example 1
Trial Balance
Debit Credit
$ $
Cash 5,100
Capital 5,000
Trade payables 2,000
Purchases 2,000
Rent 500
Electricity 200
Car 1,000
Drawings 300
Trade receivables 1,750
Sales 3,850
10,850 10,850
Purchases
$ $
Creditors 2,000 Bal c/d 2,000

Bal b/d 2,000 Income statement 2,000


Rent
$ $
Cash 500 Bal c/d 500

Bal b/d 500 Income statement 500


Electricity
$ $
Cash 200 Bal c/d 200

Bal b/d 200 Income statement 200


Sales
$ $
Bal c/d 3,850 Trade receivables 1,750
Cash 2,100
3,850 3,850

Income statement 3,850 Bal b/d 3,850

477
25: ANSWERS TO LECTURE EXAMPLES

Answer to Lecture Example 2


Colin
Income Statement
$ $
Sales (20 telephones) 600
Cost of sales
Purchases (50 telephones) 1,000
Less: closing inventories (30 telephones) (600)
400
Gross profit 200

Answer to Lecture Example 3


Income Statement
$ $
Purchases 2,000 Sales 3,850
Gross profit c/d 2,100 Closing inventory 250
4,100 4,100
Rent 500 Gross profit b/d 2,100
Electricity 200
Net profit c/d 1,400
2,100 2,100

Net profit b/d 1,400

Answer to Lecture Example 4


DOUGLAS
INCOME STATEMENT FOR THE MONTH OF JANUARY
$ $
Sales 3,850
Less cost of sales:
Purchases 2,000
Less: closing inventories ( 250)
1,750
Gross profit 2,100
Less expenses:
Rent 500
Electricity 200
(700)
Net profit 1,400

478
25: ANSWERS TO LECTURE EXAMPLES

DOUGLAS
STATEMENT OF FINANCIAL POSITION AS AT 31 JANUARY
NON-CURRENT ASSET $ $
Motor vehicle 1,000
CURRENT ASSETS
Inventories 250
Trade receivables 1,750
Cash 5,100
7,100
8,100
PROPRIETOR’S INTEREST $ $
Capital introduced on 1 January 5,000
Profit for the year 1,400
Less: drawings (300)
Balance 31 January 6,100
CURRENT LIABILITIES
Trade payables 2,000
8,100

Answer to Lecture Example 5


Drawings
$ $
Cash 300 Bal c/d 300
Bal b/d 300 Capital 300
Income statement
$ $
Purchases 2,000 Sales 3,850
Gross profit c/d 2,100 Closing inventory 250
4,100 4,100
Rent 500 Gross profit b/d 2,100
Electricity 200
Net profit c/d 1,400
2,100 2,100
Capital 1,400 Net profit b/d 1,400
Capital
$ $
Balance c/d 5,000 Cash 5,000

Drawings 300 Balance b/d 5,000


Balance c/d 6,100 Net profit 1,400
6,400 6,400

Balance b/d 6,100

Answer to Lecture Example 6


Assets = capital + (profit – drawings) + payables
8,100 = 5,000 + (1,400 – 300) + 2,000

479
25: ANSWERS TO LECTURE EXAMPLES

Chapter 7
Answer to Exercise
Net Sales tax Gross
$ $ $
(1) Factory buys raw material 100 15.00 115.00

(2) Manufactures goods and sells to wholesaler 250 37.50 287.50


22.50 Due to sales tax authority

Answer to Lecture Example 1


Purchases Trade payables
$ $
Trade payables 1,000 Purchases 1,150

Trade receivables Sales tax control a/c


$ $ $
Sales 1,725 Trade payables 150 Trade rec. 225

Sales
$ $
Trade rec. 1,500

Chapter 8
Answer to Lecture Example 1
C Transport costs to deliver goods to customers are an example of carriage outwards and should
not be included. Administrative overheads do not relate to production and cannot therefore be
included.
The depreciation of the factory machine is a production overhead and should be included.

Answer to Lecture Example 2


Net realisable value is:
$
Estimated selling price 35
Less: costs of completion (12)
Less: selling costs (1)
22

480
25: ANSWERS TO LECTURE EXAMPLES

Answer to Lecture Example 3


(a) Closing inventories (FIFO)
Purchases
Opening 10 Jan 20 Jan 25 Jan
inventories
200 300 350 250
Sales
14 Jan (200) (80)
21 Jan (220) (180)
28 Jan (80)
Nil Nil 90 250

@ $11.50 @ $13.00
= $1,035 = $3,250

$4,285

Cost of sales (FIFO)


$
Opening inventories (200 x $10) 2,000
Purchases 10,530
12,530
Less: closing inventories (4,285)
8,245
(b) Closing inventories and cost of sales (AVCO)
Average Total Cost of
Units Cost Unit Cost Cost Sales
$ $ $ $
1.1.X2 b/f 200 10.00 2,000

10.1.X2 Purchase 300 10.85 3,255


500 (W1) 10.51 5,255

14.1.X2 Sale (280) 10.51 (2,943) 2,943


220 2,312

20.1.X2 Purchase 350 11.50 4,025


570 (W2) 11.12 6,337

21.1.X2 Sale (400) 11.12 (4,448) 4,448


170 1,889

25.1.X2 Purchase 250 13.00 3,250


420 (W3) 12.24 5,139

28.1.X2 Sale (80) 12.24 (979) 979


340 4,160 8,370

$5,255 $6,337 $5,139


(W1) = $10.51 (W2) = $11.12 (W3) = $12.24
500 570 420

481
25: ANSWERS TO LECTURE EXAMPLES

Chapter 9
Answer to Lecture Example 1
Examples include:
(a) Land and buildings
(b) Plant and equipment
(c) Motor vehicles
(d) Furniture and fittings, computers

Answer to Lecture Example 2


B The cost capitalised should include the purchase price ($20,000) plus all directly attributable costs
(delivery and installation).
The cost of the maintenance contract should be shown as an expense in the income statement.

Answer to Lecture Example 3


Straight line method:
$2,500 - $250
(a) Depreciation charge =
3 years

= $750 per annum

(b) Year Cost Accumulated NBV


depreciation
$ $ $
1 2,500 750 1,750
2 2,500 1,500 1,000
3 2,500 2,250 250

Answer to Lecture Example 4


Reducing balance method:
NBV b/d Dep’n Dep’n Accumulated NBV c/d
rate expense depreciation
$ $ $
Year 1 (6,000 – 0) × 40% 2,400 2,400 3,600
Year 2 (6,000 – 2,400) × 40% 1,440 3,840 2,160
Year 3 (6,000 – 3,840) × 40% 864 4,704 1,296

Answer to Lecture Example 5


(a) Journal entry
Debit Credit
$ $
Depreciation expense 750

Accumulated depreciation 750

Being annual depreciation charged on machine

482
25: ANSWERS TO LECTURE EXAMPLES

(b) Accounting for depreciation:


Machine (SOFP)
$ $
Cash 2,500 Bal c/d 2,500
2,500 2,500
Bal b/d 2,500

Depreciation expense (I/S)


$ $
Year 1 Accumulated dep’n 750 Year 1 I/S 750

Year 2 Accumulated dep’n 750 Year 2 I/S 750

Year 3 Accumulated dep’n 750 Year 3 I/S 750

Accumulated depreciation (SOFP)


$ $
Bal c/d 750 Year 1 Depreciation expense 750

Bal c/d 1,500 Year 2 Bal b/d 750


Depreciation expense 750
1,500 1,500

Bal c/d 2,250 Year 3 Bal b/d 1,500


Depreciation expense 750
2,250 2,250

(c) Income statement (extracts):


Year 1 Year 2 Year 3
$ $ $
Expenses
Depreciation 750 750 750

Statement of financial position (extracts):


Cost Accumulated Net Book
Depreciation Value
$ $ $
(Year 1) Machine 2,500 (750) 1,750
(Year 2) Machine 2,500 (1,500) 1,000
(Year 3) Machine 2,500 (2,250) 250

483
25: ANSWERS TO LECTURE EXAMPLES

Answer to Lecture Example 6


(a)
$
Sales proceeds 3,000
NBV at end of year 2 (2,160)
840
(b)
Machine (SOFP)
$ $
Bal b/d 6,000 (a) Disposal account 6,000

Accumulated depreciation (SOFP)


$ $

(b) Disposal account 3,840 Bal b/d 3,840

Disposal account (I/S)


$ $
(a) Machine 6,000 (c) Cash 3,000

Balance = profit 840


on disposal (I/S) (b) Accumulated dep’n 3,840
6,840 6,840

Answer to Lecture Example 7


(a) The profit on disposal is still $840, the only difference is that the proceeds were not received in
cash, but in the form of a part exchange allowance.
(b) Cash paid for the new machine is $7,000 ($10,000 – $3,000)

Old machine (SOFP)


$ $
Bal b/d 6,000 (a) Disposal account 6,000

Accumulated depreciation (SOFP)


$ $
(b) Disposal account 3,840 Bal b/d 3,840

New machine (SOFP)


$ $
(c) Disposal account 3,000 Bal c/d 10,000
Cash 7,000
10,000 10,000
Bal b/d 10,000

484
25: ANSWERS TO LECTURE EXAMPLES

Disposal account (I/S)


$ $
(a) Machine 6,000 (c) New machine (part exchange) 3,000
Profit on disposal (I/S) 840 (b) Accumulated depreciation 3,840
6,840 6,840

Answer to Lecture Example 8


(a) The double entry is
$ $
Dr Non-current asset – building (150 – 100) 50,000
Dr Accumulated depreciation – building 20,000
Cr Revaluation reserve (β) 70,000

Building (SOFP)
$ $
Bal b/d 100,000
Revaluation reserve 50,000 Bal c/d 150,000
150,000 150,000
Bal b/d 150,000

Accumulated depreciation (SOFP)


$ $
Revaluation reserve 20,000 Bal b/d 20,000

Revaluation reserve (SOFP)


$ $
Building 50,000
Bal c/d 70,000 Accumulated depreciation 20,000
70,000 70,000
Bal b/d 70,000

$150,000
(b) Depreciation charge is = $3,750
40 years

Answer to Lecture Example 9


Review of useful life:
Year Depreciation Accumulated NBV
charge depreciation
$ $ $
20X1 40,000 = 8,000 8,000 32,000
5
20X2 40,000 = 8,000 16,000 24,000
5
20X3 24,000 = 12,000 28,000 12,000
2
20X4 24,000 = 12,000 40,000 0
2
40,000

485
25: ANSWERS TO LECTURE EXAMPLES

Answer to Lecture Example 10


Change in method of depreciation:
Depreciation Accumulated NBV
charge depreciation
$ $ $
20X1 40,000 × 25% 10,000 10,000 30,000
20X2 30,000 × 25% 7,500 17,500 22,500
20X3 22,500 - 1,500 7,000 24,500 15,500
3
20X4 7,000 31,500 8,500
20X5 7,000 38,500 1,500
38,500

Chapter 10
Answer to Lecture Example 1
(1) Market research would take place at an early stage in any development process. Its purpose is to
gather information about whether there may be interest in a potential product. At this point in time
an entity cannot be certain that the expenditure will lead to profits and so the costs are research
costs. $20,000 should be shown as an expense in the income statement.
(2) A machine is a tangible non-current asset and is accounted for under IAS 16 regardless of its use.
The $100,000 should be capitalised as a tangible non-current asset and depreciated over its
useful life of 10 years.
(3) Material costs and design and manufacture salaries are part of the development process. They
should be capitalised as an intangible non-current asset provided that all of the 'PIRATE' criteria
are met.
The costs should be amortised in 20X9 once the car is available to be sold on the market.

Answer to Lecture Example 2

Income statement extracts


Expenses X1 X2 X3 X4 X5
$ $ $ $ $
Research expenditure 35,000 – – – 38,000
Amortisation of
development expenditure – – 40,000 40,000 40,000

Statement of financial position extracts


Non-current assets X1 X2 X3 X4 X5
$ $ $ $ $
Development expenditure 55,000 120,000 120,000 120,000 120,000
Amortisation – – (40,000) (80,000) (120,000)
Net book value 55,000 120,000 80,000 40,000 –

486
25: ANSWERS TO LECTURE EXAMPLES

Chapter 11
Answer to Lecture Example 1
(a)
$
Electricity expense
Cash paid: 10.3.X7 96
12.6.X7 120
14.9.X7 104
10.12.X7 145
465
December expense missing ( 1 × $168) 56
3
521

$
Rent expense
Cash paid: 1.2.X7 375
6.4.X7 1,584
1,959
Less: expense relating to Jan – March × ( 3 × $1,584) (396)
12
1,563
(b) & (c)
Electricity accrual is $56
$ $
Dr Electricity expense (I/S) 56
Cr Accruals (SOFP) 56
Being: electricity expense accrued at 31 December 20X7.
Rent prepayment is $396
$ $
Dr Prepayments (SOFP) 396
Cr Rent expense (I/S) 396
Being: rent expense prepaid at 31 December 20X7.

Answer to Lecture Example 2


Electricity expense (I/S)
$ $
10.3.X7 Cash 96
12.6.X7 Cash 120
14.9.X7 Cash 104
10.12.X7 Cash 145
31.12.X7 Accruals 56 31.12.X7 Transfer to income
statement 521
521 521

487
25: ANSWERS TO LECTURE EXAMPLES

Rent expense (I/S)


$ $
1.2.X7 Cash 375
6.4.X7 Cash 1,584 31.12.X7 Transfer to income
statement 1,563
31.12.X7 Prepayments 396

1,959 1,959

Accruals (SOFP)
$ $
31.12.X7 Bal c/d 56 31.12.X7 Electricity 56
56 56
1.1.X8 Bal b/d 56

Prepayments (SOFP)
$ $
31.12.X7 Rent 396
31.12.X7 Bal c/d 396
396 396
1.1.X8 Bal b/d 396

Answer to Lecture Example 3


Working
Electricity expense (I/S)
$ $
12.3.X8 Cash 168 1.1.X8 Accrual reversed 56
9.6.X8 Cash 134 31.12.X8 To Income statement 585
12.9.X8 Cash 118
12.12.X8 Cash 158
31.12.X8 Accrual ( 13 × $189) 63
641
641

Accruals (SOFP)
$ $
1.1.X8 Accrual reversed 56 1.1.X8 Bal b/d 56
31.12.X8 Bal c/d 63 31.12.X8 Electricity accrual (W) 63
119 119
1.1.X9 Bal b/d 63

Answer to Lecture Example 4


B
$
Insurance expense
July X6 – August X6 ( 2 12 × $24,000) 4,000
Sept X6 – June X7 ( 1012 × $30,000) 25,000
29,000
Prepayment
1 June X7 paid ( 1 × $30,000)
4
7,500
Less: June X7 ( 1 × $7,500)
3
(2,500)
5,000

488
25: ANSWERS TO LECTURE EXAMPLES

Chapter 12
Answer to Lecture Example 1
(a) The balance c/d on the trade receivables account at the end of the year is $50,000.
(b) The bad debt expense shown in the I/S is $15,000
Workings
Trade receivables (SOFP)
$ $
31.12.X7 Bal b/d 65,000 31.12.X7 Bad debt expense
(Ali $7,000) 15,000
(Tyson $8,000)
31.12.X7 Bal c/d 50,000
65,000 65,000

Bad debt expense (I/S)


$ $

31.12.X7 Trade receivables 15,000 31.12.X7 To I/S 15,000

Answer to Lecture Example 2


Allowance for receivables:
(a) The allowance for receivables shown on the statement of financial position is $3,500
(b) The doubtful debts expense shown in the I/S is $3,500
Working
Allowance for receivables (SOFP)
$ $

Bal c/d 3,500 Doubtful debts expense 3,500

Doubtful debts expense (I/S)


$ $

Allowance for receivables 3,500 I/S 3,500

Income statement extract


$
Expenses
Bad debts (see Lecture Example 1) (15,000)
Doubtful debts expense (3,500)
Statement of financial position extract
$
Current assets
Trade receivables 50,000
Less: allowance for receivables (3,500)
46,500

489
25: ANSWERS TO LECTURE EXAMPLES

Answer to Lecture Example 3


(a) The allowance for receivables shown in the statement of financial position is $1,334
(b) The bad and doubtful debts expense shown in the income statement is $1,674.
Trade receivables (SOFP)
$ $
Bal b/d 47,440 Bad & doubtful debts expense 340
Bal c/d 47,100
47,440 47,100
Bal b/d 47,100

Allowance for receivables (SOFP)


$ $
Bal c/d Bad and doubtful debts expense 1,334
Specific 400
General (W) 934
1,334
1,334 1,334
Bal b/d 1,334

Bad and doubtful debts expense (I/S)


$ $
Trade receivables 340
Allowance for receivables 1,334 I/S 1,674
1,674 1,674

Working
(W) General allowance:
$
Trade receivables (net of bad debts written off) 47,100
Less: specific allowance (400)
46,700 × 2%
= $934

Answer to Lecture Example 4


Bad debts recovered:
Trade receivables (SOFP)
$ $

1.1.X8 Bal b/d 50,000

Bad debt expense (I/S)


$ $

I/S 7,000 Cash 7,000

490
25: ANSWERS TO LECTURE EXAMPLES

Cash (SOFP)
$

Bad debt expense 7,000

Answer to Lecture Example 5


Specific allowance recovered:
Trade receivables (SOFP)
$ $
Bal b/d 50,000 (a) Cash 3,500
Bal c/d 46,500
50,000 50,000

Allowance for receivables (SOFP)


$ $

(b) Doubtful debts expense 3,500 Bal b/d 3,500

Bad and doubtful debts expense (I/S)


$ $

I/S 3,500 (b) Allowance for doubtful debts 3,500

Answer to Lecture Example 6


$ $
Dr Allowance for receivables 3,500
Cr Trade receivables 3,500

Answer to Lecture Example 7


Changes in general allowance:
The doubtful debts expense in 20X8 is $500 [(30,000 x 5%) – (20,000 x 5%)]
Long method
Allowance for receivables (SOFP)
$ $
(a) Doubtful debts expense 1,000 1.1.X8 Bal b/d
(20,000 × 5%) ($20,000 × 5%) 1,000

Bal c/d 1,500 (ii) 31.12.X8 Doubtful debts expense


2,500 ($30,000 × 5%) 1,500
2,500

491
25: ANSWERS TO LECTURE EXAMPLES

Doubtful debts expense (I/S)


$ $
(ii) Allowance for receivables 1,500 (a) Allowance for receivables 1,000
I/S 500
1,500
1,500

Short method
Allowance for receivables (SOFP)
$ $
31.12.X7 Bal b/d
($20,000 × 5%) 1,000
31.12.X8 Bal c/d
($30,000 × 5%) 1,500 Doubtful debts expense 500
(increase in allowance)
1,500 1,500

Doubtful debts expense (I/S)


$ $

Allowance for receivables 500 I/S 500

Answer to Lecture Example 8


A $13,000
Allowance for Income
receivables statement
$ $
(1) Write off recovered (2,000)
(2) Write off in 20X8 18,000
(3) Change in allowance:
At 30.9.X7 24,000
At 30.9.X8 21,000
Decrease required 3,000 (3,000)
13,000

Chapter 13
Answer to Lecture Example 1
(a) A provision should be made using expected values:
($1m × 20%) + ($6m × 5%) = $0.5m
Dr Warranty cost expense (I/S) $0.5m
Cr Provisions (SOFP) $0.5m
(b) In 20X8 the provision needs to increase by $0.25m ($0.75m – $0.5m). Entry is:
Dr Warranty cost expense (I/S) $0.25m
Cr Provisions (SOFP) $0.25m

492
25: ANSWERS TO LECTURE EXAMPLES

(c) In 20X9 the provision needs to decrease by $0.45m ($0.75m – $0.3m). Entry is
Dr Provisions (SOFP) $0.45m
Cr Warranty cost expense (I/S) $0.45m

Chapter 14
Answer to Lecture Example 1
(1) Books of prime entry
Sales day book
Date Customer Amount
10 Jan X6 Customer A 150
10 Jan X6 Customer B 200

350
Purchase day book
Date Supplier Amount
15 Jan X6 Supplier Y 100
15 Jan X6 Supplier Z 1,300

1,400

Cash receipts book


Date Narrative Total Sales Receivables
21 Jan X6 Customer B 200 200

200 200
Cash payments book
Date Narrative Total Purchases Payables
21 Jan X6 Supplier Y 100 100

100 100
Memorandum ledgers
Receivables ledger
Customer A
$ $
10.1.X6 Sales 150
Bal c/d 150
150 150
Bal b/d 150
Customer B
$ $
10.1.X6 Sales 200 21.1.X6 Payment received 200

200 200

493
25: ANSWERS TO LECTURE EXAMPLES

Payables ledger
Supplier Y
$ $
21.1.X6 Payment made 100 15.1.X6 Purchases 100

100 100
Supplier Z
$ $
Bal c/d 1,300 15.1.X6 Purchases 1,300

1,300 1,300
(2)&(3) Nominal ledger
RLCA (SOFP)
$ $
31.1.X6 Sales 350 31.1.X6 Bank 200
Bal c/d 150
350 350
Bal b/d 150
PLCA (SOFP)
$ $
31.1.X6 Bank 100 31.1.X6 Purchases 1,400
Bal c/d 1,300
1,400 1,400
Bal b/d 1,300
Bank (SOFP)
$ $
31.1.X6 RLCA 200 31.1.X6 PLCA 100
Bal c/d 100
200 200
Bal b/d 100
Sales (I/S) Purchases (I/S)
$ $ $ $
31.1.X6 350 31.1.X6 1,400
RLCA PLCA
I/S 350 I/S 1,400
350 350 1,400 1,400
(4) Reconciliation
Balance per list of balances
$
Receivables ledger
Customer A 150
Customer B –
150

Balance per RLCA 150

Balance per list of balances


$
Payables ledger
Supplier Y –
Supplier Z 1,300
1,300

Balance per PLCA 1,300

494
25: ANSWERS TO LECTURE EXAMPLES

Answer to Lecture Example 2


(a)
Sales (I/S) RLCA (SOFP)
$ $ $ $
1.1.X7 RLCA 10,000 1.1.X7 Sales 10,000

(b)
Bank (SOFP) RLCA (SOFP)
$ $ $ $
4.1.X7 RLCA 9,000 1.1.X7 Sales 10,000 4.1.X7 Bank 9,000
Discounts 1,000
allowed

10,000 10,000

Discounts allowed (I/S)


$ $
4.1.X7 RLCA 1,000

(c)
Bank (SOFP) RLCA (SOFP)
$ $ $ $
4.1.X7 RLCA 10,000 1.1.X7 Sales 10,000 4.1.X7 Bank 10,000

10,000 10,000

Answer to Lecture Example 3


(a)
Purchases (I/S) PLCA (SOFP)
$ $ $ $
PLCA 5,000 Purchases 5,000

(b)

Bank (SOFP) PLCA (SOFP)


$ $ $ $
Bank 4,750 Bank 4,750 Purchases 5,000
Discounts
received 250

5,000 5,000

495
25: ANSWERS TO LECTURE EXAMPLES

Discounts received (I/S)


$ $
PLCA 250

(c)

Bank (SOFP) PLCA (SOFP)


$ $ $ $

PLCA 5,000 Bank 5,000 Purchases 5,000

Answer to Lecture Example 4


B
$
List price 50,000
Less: trade discount (12%) (6,000)
Record purchase at this value 44,000
Less: settlement discount (4%) (1,760) $50,336
Calculate sales tax on this value 42,240

Sales tax at 15% 6,336

Answer to Lecture Example 5


(a) RLCA
$ $
Balance b/d 614,000 Bank 311,000
Sales 302,600 Discounts allowed 3,400
Contras (PLCA) 8,650
Bad debts 32,000
Bal c/d 561,550
916,600 916,600
(b) Reconciliation
RLCA
$ $
Bal b/d (part (a)) 561,550

(i) Sales (SDB undercast) 3,600 Bal c/d 565,150


565,150 565,150

$
Balance per list of balances 563,900
(ii) Credit balance included as a debit (2 × $450) (900)
Customer balance omitted 2,150
1,250
565,150

496
25: ANSWERS TO LECTURE EXAMPLES

Chapter 15
Answer to Lecture Example 1
Adjustment of cash book balance
Cash account
$ $
Balance b/d 204 Standing order (3i) 35
Bank interest (3ii) 18 Bank charges (3iii) 14
Balance c/d 173
222 222
Bank reconciliation statement
$
Balance per bank statement at 31 March 20X8 2,618
Unrecorded lodgements 723
Outstanding cheques (3,168)
Balance per cash book at 31 March 20X8 173

Answer to Lecture Example 2


B (1) is a bank error, (4) is an outstanding cheque (2), (3) and (5) have all been processed correctly
by the bank but need recording in the cash account.

Chapter 16
Answer to Lecture Example 1
(a) Journal entries
Dr Cr
$ $
(1) Rent and rates 350
Trade receivables 350

(2) Discounts allowed 500


Trade receivables 500

(3) Trade receivables 2,620


Cash at bank 2,620

(4) Suspense account 1,900


Cash at bank 1,900

(5) Stationery and postage 1,460


Suspense account 1,460

(6) Capital 18,000


Suspense account 18,000
(b) Suspense account
$ $
Brought forward
(102,800 – 85,240) 17,560 Stationery and postage (5) 1,460
Cash at bank (4) 1,900 Capital (6) 18,000
19,460 19,460

497
25: ANSWERS TO LECTURE EXAMPLES

Answer to Lecture Example 2


Adjustment of profits statement for the year ended 30 April 20X7

Increases Decreases
$ $ $
Draft profit
Adjustments 12,300
Rent (1) 350
Discounts allowed (2) 500
Stationery (5) 1,460
Total adjustments 2,310 (2,310)
Revised profit 9,990

Answer to Lecture Example 3


B
Increases Decreases
$ $ $
Draft profit 112,400
Adjustments:
(1) sales returns (2 × $2,700) 5,400
(2) depreciation (W) 1,250
– 6,650 (6,650)
Adjusted profit 105,750
(W ) Depreciation charge was
33 13 % × ($15,000 × 2/4) = $2,500
Depreciation charge should have been
$15,000 ÷ 4 years = $3,750
Incremental depreciation to be charged $1,250

Chapter 17
Answer to Lecture Example 1
(a) (1)
Dr Cr
$ $
Dr Inventories (SOFP) 647
Cr Closing inventories (I/S) 647
Being: adjustment to record year end closing inventories.
(2)
$ $
Dr Drawings (12 × $10) 120
Cr Wages 120
Being: correction of cash drawings posted as wages.

498
25: ANSWERS TO LECTURE EXAMPLES

(3)
$ $
Dr Depreciation expense (I/S) 601
Cr Accumulated depreciation:
Motor vehicles ($1,740 × 25%) 435
Furniture and fittings ($829 × 20%) 166
Being: adjustment to record depreciation for the year
(4)
$ $
Dr Bad debt expense 37
Cr Trade receivables 37
Being: write off of irrecoverable customer balance.
(5)
$ $
Dr Bank (2 × $180) 360
Cr Suspense account 360
Being: adjustment to correct cash receipt from trade receivables.
(6)
$ $
Dr Drawings 63
Cr Purchases 63
Being: adjustment for goods drawn from business (removed at cost value)
(7)
$ $
Dr Rent expense (600 – 500) 100
Cr Accruals 100
Being: accrual of rent expense.
$ $
Dr Prepayments ($180 × 6/12) 90
Cr Electricity expense 90
Being: prepayment of electricity expense.
(8)
$ $
Dr Discounts allowed (I/S) 73
Cr Suspense account 73
Being: adjustment for discounts allowed omitted.
(b)
Suspense account
$ $
Bal b/d 433
(5) Bank 360
(8) Discounts allowed 73

433 433

499
25: ANSWERS TO LECTURE EXAMPLES

(c) Mugg
Income statement for the year ended 31 December 20X7
$ $
Sales 15,542
Less: cost of sales
Opening inventories 510
Purchases (9,876 – 63) 9,813
10,323
Less: closing inventories 647
9,676
Gross profit 5,866
Discounts received 129
5,995
Less expenses:
Rent (500 + 100) 600
Electricity (240 – ( 612 × 180)) 150
Insurance 120
Wages (1,634 – 120) 1,514
Repairs 635
Depreciation 601
Travel and entertaining 192
Bad debts 37
Discounts allowed 73
3,922
Profit for the period 2,073
Mugg
Statement of financial position as at 31 December 20X7
Accumulated NBV
Cost depreciation
$ $ $
Non-current assets
Motor vehicles 1,740 870 870
Furniture and fixtures 830 332 498
2,569 1,202 1,368
Current assets
Inventories 647
Trade receivables (672 – 37) 635
Prepayments 90
Cash and bank balances (5 + 762 + 360) 1,127
2,499
3,867

Capital $
Capital as at 1 January 20X7 2,377
Profit for the period 2,073
Less: drawings (1,200 + 63 + 120) (1,383)
3,067
Current liabilities
Trade payables 700
Accruals 100
800
3,867

500
25: ANSWERS TO LECTURE EXAMPLES

Chapter 18
Answer to Lecture Example 1
% $
Sales 100 476,000 x 60%

COS 60 285,600

GP 40 190,400

Answer to Lecture Example 2


% $
Sales 130 221,000 x 100/130

COS 100 170,000

GP 30 51,000

Purchases: $
Cost of sales
Opening inventory 43,000

+ Purchases 174,500

– Closing inventory 47,500


170,000

Answer to Lecture Example 3


B Cost structure: 25% mark up.
$
Sales = 125% = 985,000
∴ COS = 100% = 788,000
Gross profit 25% 197,000
Cost of sales
$
Opening inventories 620,000
Purchases 700,000
1,320,000
Less: cost of sales (788,000)
Closing inventories should be 532,000
Closing inventories is (180,000)
∴ inventory lost in fire 352,000

501
25: ANSWERS TO LECTURE EXAMPLES

Answer to Lecture Example 4


Trade payables
$ $
Bal b/d 38,450
Till 430
Bank 167,224
Balance c/d 43,825 Purchases* 173,029
211,479 211,479

* Dr Purchases (I/S) $173,029


Cr Trade payables $173,029

Answer to Lecture Example 5


Cash
$ $
Bal b/d 50
Receipts from General expenses 4,500
Trade receivables (1) 39,204 Drawings 6,250
Bankings 28,454
Bal c/d 50
39,254 39,254

Trade receivables
$ $
Bal b/d 1,447 Cash (deduced from
Sales* (2) 39,685 cash a/c) 39,204
Bal c/d 1,928
41,132 41,132

Answer to Lecture Example 6


$4,050
Cost structure:
$
Sales = 100% = 23,750
∴ COS = 80% = 19,000
Gross profit 20% 4,750

Cash
$ $
Balance b/d 1,000 Wages 5,200
Stationery 500
Sales 23,750 Electricity 1,200
Bankings 12,800
∴ drawings 4,050
Bal c/d 1,000
24,750 24,750

502
25: ANSWERS TO LECTURE EXAMPLES

Chapter 19
Answer to Lecture Example 1
(a)
(i)
Capital accounts
Tick Cast Balance Tick Cast Balance
$ $ $ $ $ $
Bal c/d 50,000 30,000 20,000 Bank 50,000 30,000 20,000

50,000 30,000 20,000 50,000 30,000 20,000


Bal b/d 50,000 30,000 20,000

(ii)
Appropriation account for the year ended 31 December 20X4
$ $ $
Salary – Balance 15,000 Profit b/d from the
income statement 50,000
Interest on capital (12%)
Tick 6,000
Cast 3,600
Balance 2,400
12,000
Profit share
Tick (5/10) 11,500
Cast (3/10) 6,900
Balance (2/10) 4,600
23,000
50,000 50,000

(iii)
Current accounts
Tick Cast Balance Tick Cast Balance
$ $ $ $ $ $
Drawings 6,000 4,000 8,800 Salary 15,000
Bal c/d 11,500 6,500 13,200 Interest on
capital 6,000 3,600 2,400
Profit share 11,500 6,900 4,600
17,500 10,500 22,000 17,500 10,500 22,000

503
25: ANSWERS TO LECTURE EXAMPLES

(iv)
TICK, CAST AND BALANCE
Statement of financial position as at 31 December 20X4 (extract)
$ $
Capital accounts
Tick 50,000
Cast 30,000
Balance 20,000
100,000

Current accounts
Tick 11,500
Cast 6,500
Balance 13,200
31,200
131,200
(b) Alternative answer to parts (ii) and (iii) (including interest on drawings)
(ii)
Appropriation account for the year ended 31 December 20X4
$ $ $
Salary – Balance 15,000 Profit b/d 50,000
Interest on drawings
Interest on capital (12%) Tick (6,000 x 10% x 6/12) 300
Tick 6,000 Cast (4,000 x 10% x 3/12) 100
Cast 3,600
Balance 2,400
12,000
Profit share
Tick (5/10) 11,700
Cast (3/10) 7,020
Balance (2/10) 4,680
23,400
50,400 50,400

(iii)
Current accounts
Tick Cast Balance Tick Cast Balance
$ $ $ $ $ $
Drawings 6,000 4,000 8,800 Salary 15,000
Interest on Interest on
drawings 300 100 – capital 6,000 3,600 2,400
Balance c/d 11,400 6,520 13,280 Profit share 11,700 7,020 4,680
17,700 10,620 22,080 17,700 10,620 22,080

Bal b/d 11,400 6,520 13,280

504
25: ANSWERS TO LECTURE EXAMPLES

Answer to Lecture Example 2


X Y Z Total
$ $ $ $
Salaries 15,000 – 8,000 23,000
Interest on capital 400 400 400 1,200
Profit share (6:3:1) 25,320 12,660 4,220 42,200 (β)
Total profit for the year (W) 40,720 13,060 12,620 66,400
(W)
$
Profit before loan interest 67,000
Loan interest ($10,000 × 12% × 6/12) (600)
66,400

Answer to Lecture Example 3


B M S A
$ $ $
Salary 20,000 10,000
PSR (1st half) 90,000 30,000 30,000
PSR (2nd half) 110,000 66,000 44,000
200,000 116,000 84,000

$
Profit for year 400,000
Add bank expense relating to first half of year 40,000
440,000
1st Half
$ $
Profit ($440,000 x 6/12) 220,000
Expense relevant to first half of year (40,000)
180,000
Salary S (40,000 x 6/12) (20,000)
A (20,000 x 6/12) (10,000)
150,000
PSR M 60% (90,000)
S 20% (30,000)
A 20% (30,000)
(150,000)

2nd Half
$ $
Profit ($440,000 x 6/12) 220,000
Salary –
220,000
PSR M 50% (110,000)
S 30% (66,000)
A 20% (44,000)
(220,000)

505
25: ANSWERS TO LECTURE EXAMPLES

Answer to Lecture Example 4


Capital account
A B C A B C
$'000 $'000 $'000 $'000 $'000 $'000
Bal b/d 80 97 –
Goodwill 20 20 20 Goodwill 25 35 –
(1:1:1) (5:7)
Bal c/d 85 112 80 Bank – – 100
105 132 100 105 132 100

Goodwill
$'000 $'000
Capital account (210 – 150) 60 Capital account 60

C has effectively paid in capital of $100,000 but $20,000 of this was used to pay for C's share of
the goodwill the partnership had built up. C's closing investment in the partnership is therefore
$80,000.

Chapter 20
Answer to Lecture Example 1
Rab Co
$ $
Dr Cash (200,000 × 80c) 160,000
Cr Share capital (200,000 × 50c) 100,000
Cr Share premium account (200,000 × 30c) 60,000

Statement of financial position (extract) as at 1 June 20X0


Equity $
Share capital – 50c ordinary shares (50,000 + 100,000) 150,000
Share premium account 60,000
210,000

506
25: ANSWERS TO LECTURE EXAMPLES

Answer to Lecture Example 2


Bonus Issue
300,000
New share capital: × 50c = $37,500
4
Double entry:
$ $
Dr Share premium account 37,500
Cr Share capital 37,500

Statement of financial position


$
Share capital – 50c ordinary shares (150,000 + 37,500) 187,500
Share premium account (60,000 – 37,500) 22,500
Retained earnings 200,000
410,000

Answer to Lecture Example 3


Rights Issue
$ $

375,000
New share capital: × 50c 37,500
5

375,000
Share premium: × $1 75,000
5

$ $
Dr Cash 112,500
Cr Share capital 37,500
Cr Share premium account 75,000

Rab Co
Statement of financial position (extract)
$
Share capital – 50c ordinary shares 225,000
Share premium account 97,500
Retained earnings 230,000
552,500

Answer to Lecture Example 4


ABC Co
Reconciliation of movement in retained earnings
for year ended 31 December 20X6
$ $
Retained earnings at beginning of year 125,000
Profit for the period 50,000
Dividends – preference 6,000
– ordinary 10,000
(16,000)
Retained earnings at end of year 159,000

507
25: ANSWERS TO LECTURE EXAMPLES

Answer to Lecture Example 5


(1)
Income tax expense (I/S)
$ $
31.12.X5 Current tax payable 62,000 31.12.X5 Income statement 62,000

30.9.X6 Current tax payable 3,000

31.12.X6 Current tax payable 43,000

Current tax payable (SOFP)


$ $
31.12.X5 Balance c/d 62,000 31.12.X5 Income tax expense 62,000
62,000 62,000

30.9.X6 Bank 65,000 1.1.X6 Balance b/d 62,000


30.9.X6 Income tax expense 3,000
31.12.X6 Bal c/d 43,000 31.12.X6 Income tax expense 43,000
108,000 108,000

1.1.X7 Balance b/d 43,000


(2) Tax note for the year ended 31 December 20X6
$
Tax charge for the year 43,000
Under provision in respect of prior periods 3,000
46,000

Chapter 21
Answer to Lecture Example 1
(a) One single statement
Statement of comprehensive income for the year ended 30 September 20X6

$’000
Revenue 12,740
Cost of sales (7,040)
Gross profit 5,700
Distribution costs (2,060)
Administrative expenses (2,375)
Finance costs (72)
Profit before tax 1,193
Income tax expense (270)
Profit for the year 923
Other comprehensive income:
Gains on property revaluation 600
Total comprehensive income for the year 1,523

508
25: ANSWERS TO LECTURE EXAMPLES

(b) Two separate statements


Income statement for the year ended 30 September 20X6

$’000
Revenue 12,740
Cost of sales (7,040)
Gross profit 5,700
Distribution costs (2,060)
Administrative expenses (2,375)
Finance costs (72)
Profit before tax 1,193
Income tax expense (270)
Profit for the year 923

Statement of comprehensive income for the year ended 30 September 20X6

$’000
Profit for the year 923
Other comprehensive income:
Gains on property revaluation 600
Total comprehensive income for the year 1,523

Answer to Lecture Example 2


Share Share Reval- Retained Total
capital premium uation earnings equity
account reserve
$’000 $’000 $’000 $’000 $’000
Balance at 30 September 20X5 1,500 200 800 1,250 3,750

Issue of share capital 250 385 635

Dividends (300) (300)

Total comprehensive income ___ ___ 600 923 1,523

Balance at 30 September 20X6 1,750 585 1,400 1,873 5,608

Working
Rights issue:
Issue is on a 1 for 6 basis, therefore issue 3,000,000 ÷ 6 = 500,000 shares at $1.27 each.
Record as:
Dr Bank (500,000 × $1.27) 635,000
Cr Share capital (500,000 × 50c) 250,000
Cr Share premium (500,000 × 77c) 385,000

509
25: ANSWERS TO LECTURE EXAMPLES

Chapter 22
Answer to Lecture Example 1
B 1 and 3 are non-adjusting events as the condition did not exist at the end of the reporting period.

Chapter 23
Answer to Lecture Example 1
Income taxes paid
Income tax payable
$'000 $'000
Income tax paid 116 Bal b/d 168
Bal c/d 156 I/S 104
272 272

Answer to Lecture Example 2


Property, plant and equipment
Plant and equipment – cost
$'000 $'000
Bal b/d 200 Disposal 20
Addition 100 Bal c/d 280
300 300

Accumulated depreciation
$'000 $'000
Disposal 9 Bal b/d 80
Bal c/d 111 ... Charge 40
120 120

Profit/loss on disposal:
$
Net book value of asset sold 11,000
Sales proceeds (8,000)
Loss on sale (3,000)
The entries in the statement of cash flows for 20X9 would be:
$
(i) Cash flows from operating activities (extract)
Adjustments for
Depreciation 40,000
Loss on sale of plant 3,000
43,000
(ii) Cash flows from investing activities (extract)
Purchase of property, plant and equipment (100,000)
Proceeds from sale of plant 8,000
(92,000)

510
25: ANSWERS TO LECTURE EXAMPLES

Answer to Lecture Example 3


Dividends paid
Dividends payable
$'000 $'000
Dividends paid 50 Bal b/d 35
Bal c/d 45 Retained earnings 60
95 95

Answer to Lecture Example 4


Emma Co
Statement of cash flows for the year ended 31 December 20X8
$’000 $’000
Cash flows from operating activities
Profit before taxation 87
Adjustments for:
Depreciation 42
Interest expense 8
137
Increase in trade receivables (168 – 147) (21)
Increase in inventories (214 – 210) (4)
Increase in trade payables (136 – 121) 15
Cash generated from operations 127
Interest paid (8)
Income taxes paid (W2) (20)
Net cash from operating activities 99

Cash flows from investing activities


Purchase of property, plant and equipment (W1) (146)
Net cash used in investing activities (146)

Cash flows from financing activities


Proceeds from issue of shares (250 + 70 – 200 – 60) 60
Proceeds from issue of debentures 30
Dividends paid (W3) (22)
Net cash from financing activities 68

Net increase in cash and cash equivalents 21


Cash and cash equivalents at beginning of year (14)
Cash and cash equivalents at end of year 7
Workings

(W1) Property, plant and equipment at NBV


$’000 $’000
Bal b/d 514 Depreciation 42
Revaluation during the year 10 Bal c/d 628
(110 – 100)
Additions 146
670 670

511
25: ANSWERS TO LECTURE EXAMPLES

(W2) Income tax payable


$’000 $’000
Income tax paid 20 Bal b/d 28
Bal c/d 39 Income tax expense (I/S) 31
59 59

(W3) Dividends payable


$’000 $’000
Dividends paid 22 Bal b/d 16
Bal c/d 18 Dividend for year 24
40 40

Answer to Lecture Example 5


Emma Co
Statement of cash flows for the year ended 31 December 20X8 (extract)
$'000 $'000
Cash flow from operating activities

Cash receipts from customers (W1) 579


Cash payments to suppliers and employees (W2) (452)
Cash generated from operations 127
Interest paid* (8)
Income taxes paid* (20)
Net cash from operating activities 99
* These amounts are the same amounts as in Lecture Example 4.
Workings
(W1) Trade receivables
$'000 $'000
Bal b/d 147 ∴ cash received 579
Revenue (I/S) 600
Bal c/d 168

747 747
(W2) Trade payables
$'000 $'000
Bal b/d 121
∴ cash paid 452 Expenses (W3) 467

Bal c/d 136


588 588
(W3)
$'000 $'000
Cost of sales 319
Add: closing inventories 214
Less: opening inventories (210)
Purchases 323
Other expenses 186
Less: depreciation (42)
144
467

512
25: ANSWERS TO LECTURE EXAMPLES

Chapter 24
Answer to Lecture Example 1
Information that may be included in a database file for a non-current asset register:
(1) Code/item number to identify asset
(2) Details of category of asset (motor vehicles, machine etc)
(3) Serial number of the asset
(4) Details of physical location of the asset
(5) Person responsible for the asset
(6) Cost of the asset
(7) Date of purchase
(8) Depreciation policy for the asset
(9) Accumulated depreciation charged to date
(10) Net book value of the asset
(11) Insurance details

513
25: ANSWERS TO LECTURE EXAMPLES

END OF ANSWERS TO LECTURE EXAMPLES

514

Вам также может понравиться